Menu Top
Applied Mathematics for Class 11th & 12th (Concepts and Questions)
11th Concepts Questions
12th Concepts Questions

Applied Maths Class 11th Chapters (Q & A)
1. Numbers and Quantification 2. Numbers Applications 3. Sets
4. Relations 5. Sequences and Series 6. Permutations and Combinations
7. Mathematical Reasoning 8. Calculus 9. Probability
10. Descriptive Statistics 11. Financial Mathematics 12. Coordinate Geometry

Content On This Page
Objective Type Questions Short Answer Type Questions Long Answer Type Questions


Chapter 11 Financial Mathematics (Q & A)

Welcome to this comprehensive Question and Answer resource page, meticulously designed to serve as an essential practice platform for Chapter 11: Financial Mathematics. This collection provides extensive opportunities to test, reinforce, and apply the crucial concepts and calculation techniques covered in this highly practical chapter of Applied Mathematics. Understanding how money grows over time, how loans are structured, how investments are valued, and how assets depreciate is fundamental not only for further studies in commerce and economics but also for navigating personal financial decisions effectively. This Q&A bank moves beyond theoretical understanding, challenging you to perform accurate calculations and solve realistic financial scenarios, thereby building essential quantitative literacy and problem-solving skills.

The questions featured here cover the entire spectrum of financial mathematics introduced at this level. You will find ample practice on the core concept of Interest. While problems involving Simple Interest ($SI = \frac{P \times R \times T}{100}$) might serve as a warm-up, the significant focus is on Compound Interest. Expect questions requiring you to calculate the future amount using $A = P(1 + i)^n$ and the compound interest ($CI = A - P$) under various scenarios. This includes dealing with different compounding frequencies (such as annually, semi-annually, quarterly, or even monthly), demanding careful calculation of the periodic interest rate ($i$) and the number of periods ($n$). Understanding and calculating the Effective Rate of Interest – the equivalent annual rate reflecting the effect of compounding – is another key area assessed, crucial for comparing different investment options fairly.

A major component of this Q&A collection revolves around Annuities – sequences of equal payments made at regular intervals. The focus is primarily on Ordinary Annuities (payments at the end of each period). Questions will rigorously test your ability to apply the standard formulas to calculate:

These problems often involve complex word scenarios requiring careful identification of the parameters ($R, i, n$) and selection of the correct formula (FV or PV).

Furthermore, the resource provides practice on the concept of Depreciation, the decrease in value of assets over time. Questions will require you to calculate the depreciated value or the amount of depreciation using common methods:

You might also encounter problems asking to find the rate of depreciation or the effective life of an asset. Depending on the syllabus scope, basic questions related to financial instruments like calculating dividends or yield on stocks/shares, or practical calculations involving taxes like Goods and Services Tax (GST), could also be included.

The question formats are diverse, including MCQs for quick checks, Fill-in-the-Blanks for specific values, True/False for conceptual understanding, and demanding Short/Long Answer word problems requiring multi-step calculations and comparisons. Crucially, the provided answers are detailed, offering step-by-step calculations, clear formula application, careful handling of time periods and rates, and logical structuring for complex problems. This Q&A resource is vital for developing the computational proficiency and practical financial understanding needed in today's world.



Objective Type Questions

Question 1. What is the main purpose of interest in financial transactions?

(A) To cover the cost of inflation.

(B) To compensate the lender for the risk and opportunity cost of lending money.

(C) To generate revenue for the government.

(D) To simplify financial calculations.

Answer:

Correct Option: (B) To compensate the lender for the risk and opportunity cost of lending money.

Explanation: Interest is charged primarily to compensate the lender for:

  • The risk involved in lending (possibility of default), and
  • The opportunity cost of forgoing other investment opportunities during the lending period.

Therefore, option (B) is the most appropriate choice.

Question 2. If you borrow $\textsf{₹}\,10,000$ at an interest rate of 8% per annum, what is the principal amount?

(A) $\textsf{₹}\,10,000$

(B) $\textsf{₹}\,800$

(C) 8%

(D) The total amount repaid.

Answer:

Correct Option: (A) $\textsf{₹}\,10,000$

Explanation:

The principal amount is the original sum of money borrowed or invested before any interest is added.

Here, it is clearly stated that you borrow $\textsf{₹}\,10,000$, which directly refers to the principal amount.

Therefore, the correct answer is: Option (A).

Question 3. A higher interest rate generally means:

(A) Lower cost for borrowers.

(B) Higher returns for lenders/investors.

(C) Less risk for lenders.

(D) Simpler calculations.

Answer:

Correct Option: (B) Higher returns for lenders/investors.

Explanation:

A higher interest rate implies that lenders or investors earn more from the money they lend or invest. It increases their income from interest, thus offering higher returns.

On the contrary:

  • (A) is incorrect because a higher interest rate increases the cost for borrowers.
  • (C) is incorrect because interest rate does not directly reduce lending risk.
  • (D) is incorrect because the complexity of calculation is not related to the interest rate level.

Therefore, option (B) is correct.

Question 4. Assertion (A): Interest rates influence borrowing and lending decisions in the economy.

Reason (R): High interest rates discourage borrowing and encourage saving.

(A) Both A and R are true and R is the correct explanation of A.

(B) Both A and R are true but R is not the correct explanation of A.

(C) A is true but R is false.

(D) A is false but R is true.

Answer:

Correct Option: (A) Both A and R are true and R is the correct explanation of A.


Explanation:

Assertion (A): Interest rates indeed influence borrowing and lending decisions. When rates change, they directly affect how much people borrow or save.

Reason (R): High interest rates make borrowing more expensive and saving more rewarding. Hence, individuals and businesses tend to borrow less and save more.

Since both statements are correct and Reason (R) clearly explains why Assertion (A) is true, option (A) is the correct choice.

Question 5. Case Study: Ram deposits $\textsf{₹}\,50,000$ in a fixed deposit account that offers an interest rate of 6% per annum.

What is the principal amount of Ram's deposit?

(A) $\textsf{₹}\,50,000$

(B) $\textsf{₹}\,3,000$

(C) 6%

(D) The total amount received after interest.

Answer:

Correct Option: (A) $\textsf{₹}\,50,000$


Explanation:

The principal amount is the original sum of money deposited or invested before any interest is added.

In this case, Ram deposits $\textsf{₹}\,50,000$, which is the amount on which interest will be calculated.

Therefore, the principal amount is $\textsf{₹}\,50,000$.

Question 6. Complete the statement: The interest rate is usually expressed as a percentage per ________.

(A) month

(B) year

(C) period

(D) transaction

Answer:

Correct Option: (B) year


Explanation:

The interest rate is generally expressed as a percentage per annum, i.e., per year. This is known as the annual interest rate.

Although interest can also be calculated monthly, quarterly, or for other time periods, the standard expression is per year unless otherwise specified.

Therefore, the correct answer is (B) year.

Question 7. In simple interest, the interest is calculated only on the ________ amount.

(A) principal

(B) accumulated

(C) interest

(D) discounted

Answer:

Correct Option: (A) principal


Explanation:

In simple interest, the interest is calculated only on the original sum of money borrowed or invested, which is called the principal.

Unlike compound interest, where interest is calculated on both the principal and the accumulated interest, simple interest does not consider interest on interest.

Hence, in simple interest, the interest is calculated only on the principal amount.

Question 8. If $\textsf{₹}\,1000$ is invested at a simple interest rate of 5% per annum for 3 years, what is the simple interest earned?

(A) $\textsf{₹}\,50$

(B) $\textsf{₹}\,150$

(C) $\textsf{₹}\,1050$

(D) $\textsf{₹}\,1150$

Answer:

Correct Option: (B) $\textsf{₹}\,150$


Explanation:

We use the formula for simple interest:

$SI = \dfrac{P \times R \times T}{100}$

…(i)

Where,

$P = \textsf{₹}\,1000$, $R = 5\%$ per annum, $T = 3$ years

Substituting in equation (i):

$SI = \dfrac{1000 \times 5 \times 3}{100}$

$SI = \dfrac{15000}{100} = \textsf{₹}\,150$

Therefore, the simple interest earned is $\textsf{₹}\,150$.

Question 9. In compound interest, the interest is calculated on the principal amount plus the ________ earned in previous periods.

(A) simple interest

(B) compound interest

(C) accumulated amount

(D) principal only

Answer:

Correct Option: (B) compound interest


Explanation:

In compound interest, the interest is calculated not just on the original principal, but also on the interest earned in the previous periods.

This means that each time period, the interest is added to the principal to form a new base amount on which the interest is calculated again.

Therefore, the interest is calculated on the principal amount plus the compound interest earned previously.

Question 10. If $\textsf{₹}\,1000$ is invested at a compound interest rate of 5% per annum, compounded annually for 2 years, what is the accumulated amount?

(A) $\textsf{₹}\,1050$

(B) $\textsf{₹}\,1100$

(C) $\textsf{₹}\,1102.50$

(D) $\textsf{₹}\,100$

Answer:

Correct Option: (C) $\textsf{₹}\,1102.50$


Given:

Principal (P) = $\textsf{₹}\,1000$

Rate (R) = 5% per annum

Time (T) = 2 years


Formula for compound amount:

$A = P \left(1 + \frac{R}{100} \right)^T$

…(i)

Substituting the values:

$A = 1000 \left(1 + \frac{5}{100} \right)^2$

[Using equation (i)]

$A = 1000 \left(1.05\right)^2$

$A = 1000 \times 1.1025$

$A = \textsf{₹}\,1102.50$


Therefore, the accumulated amount after 2 years is $\textsf{₹}\,1102.50$.

Question 11. The formula for the accumulated amount (A) under simple interest is: (P = Principal, R = Rate per period, T = Number of periods)

(A) $A = P(1 + RT)$

(B) $A = P(1 + R)^T$

(C) $A = PRT/100$

(D) $A = P + PRT/100$

Answer:

Correct Option: (D) $A = P + \dfrac{PRT}{100}$


Explanation:

In Simple Interest, the formula for interest is:

$I = \dfrac{PRT}{100}$

…(i)

Where:

  • P = Principal
  • R = Rate of Interest per period
  • T = Time (in same units as rate)

To find the accumulated amount (A) under simple interest, we add the interest to the principal:

$A = P + I$

…(ii)

Substituting equation (i) into equation (ii):

$A = P + \dfrac{PRT}{100}$


Hence, the correct formula for accumulated amount under simple interest is: $A = P + \dfrac{PRT}{100}$

Question 12. The formula for the accumulated amount (A) under compound interest is: (P = Principal, i = Interest rate per compounding period, n = Number of compounding periods)

(A) $A = P(1 + ni)$

(B) $A = P(1 + i)^n$

(C) $A = P + P(1+i)^n$

(D) $A = Pi^n$

Answer:

Correct Option: (B) $A = P(1 + i)^n$


Explanation:

In Compound Interest, the interest is added to the principal after each compounding period, and future interest is calculated on the increased amount.

The standard formula to find the accumulated amount (A) is:

$A = P(1 + i)^n$

…(i)

Where:

  • P = Principal
  • i = Interest rate per compounding period
  • n = Total number of compounding periods

Hence, the correct formula for compound amount is: $A = P(1 + i)^n$

Question 13. Assertion (A): For a period longer than one year, compound interest generally yields a higher accumulated amount than simple interest at the same annual rate.

Reason (R): Compound interest calculates interest on previously earned interest, while simple interest only calculates interest on the principal.

(A) Both A and R are true and R is the correct explanation of A.

(B) Both A and R are true but R is not the correct explanation of A.

(C) A is true but R is false.

(D) A is false but R is true.

Answer:

Correct Option: (A) Both A and R are true and R is the correct explanation of A.


Explanation:

Assertion (A): True. For a time period longer than one year, compound interest yields a higher accumulated amount than simple interest because it accumulates interest on the previously earned interest.

Reason (R): True. Compound interest is calculated on the principal and also on the accumulated interest, whereas simple interest is calculated only on the original principal amount.

Thus, the reason clearly explains why the assertion holds true.


Therefore, both Assertion and Reason are correct, and the Reason is the correct explanation of the Assertion.

Question 14. Case Study: A person borrows $\textsf{₹}\,20,000$ for 4 years.

If the interest rate is 7% per annum simple interest, what is the total amount to be repaid?

(A) $\textsf{₹}\,5600$

(B) $\textsf{₹}\,25600$

(C) $\textsf{₹}\,1400$

(D) $\textsf{₹}\,21400$

Answer:

Correct Option: (B) $\textsf{₹}\,25600$


Given:

Principal, $P = \textsf{₹}\,20,000$

Rate of Interest, $R = 7\%$ per annum

Time, $T = 4$ years


Using the Simple Interest formula:

$SI = \dfrac{P \times R \times T}{100}$

…(i)

Substituting the values:

$SI = \dfrac{20000 \times 7 \times 4}{100}$

(Substituting in equation (i))

$SI = \dfrac{560000}{100} = \textsf{₹}\,5600$


Total amount to be repaid:

$A = P + SI = 20000 + 5600 = \textsf{₹}\,25600$

…(ii)


Hence, the total amount to be repaid is: $\textsf{₹}\,25600$

Question 15. Case Study: (Same setup as Q14)

If the interest rate is 7% per annum compounded annually, what is the total amount to be repaid after 4 years? (Approximate calculation might be sufficient for options).

(A) $\textsf{₹}\,25600$

(B) $\textsf{₹}\,26215$ (approx)

(C) $\textsf{₹}\,28000$

(D) $\textsf{₹}\,20000$

Answer:

Correct Option: (B) $\textsf{₹}\,26215$ (approx)


Given:

Principal, $P = \textsf{₹}\,20,000$

Rate of Interest, $R = 7\%$ per annum (compounded annually)

Time, $T = 4$ years


Using the Compound Interest formula:

$A = P \left(1 + \dfrac{R}{100} \right)^T$

…(i)

Substituting the values:

$A = 20000 \left(1 + \dfrac{7}{100} \right)^4 = 20000 \times (1.07)^4$

(Substituting in equation (i))

Now calculating $(1.07)^4$ approximately:

$(1.07)^2 = 1.1449$, so

$(1.07)^4 = (1.1449)^2 \approx 1.311$

Therefore,

$A = 20000 \times 1.311 \approx \textsf{₹}\,26215$


Hence, the total amount to be repaid is approximately: $\textsf{₹}\,26215$

Question 16. Simple interest rate and compound interest rate are equivalent if they produce the same _________ amount over the same time period.

(A) principal

(B) interest

(C) accumulated

(D) discounted

Answer:

Correct Option: (C) accumulated


Explanation:

The simple interest rate and the compound interest rate are said to be equivalent when both produce the same accumulated amount (i.e., total amount including interest and principal) over the same time period.

This means, under both types of interest calculation, the final amount at the end of the time period remains equal, although the calculation methods differ.

Therefore, the correct word to complete the sentence is: accumulated.

Question 17. If a simple interest rate of $r_s$ is equivalent to a compound interest rate of $i$ compounded annually over $t$ years, then $1 + r_s t = (1 + i)^t$. For these rates to be equivalent, they should yield the same return for ANY time period $t$.

(A) True

(B) False

(C) True, but only for short periods.

(D) False, they are never truly equivalent for all periods unless the rates are zero.

Answer:

Correct Option: (A) True


Explanation:

We are given the condition for equivalence of simple interest rate $r_s$ and compound interest rate $i$ over $t$ years:

$1 + r_s t = (1 + i)^t$

…(i)

This equation ensures that both types of interest yield the same amount over the same time period.

Therefore, if this equation holds true for all $t$, then the two rates are indeed equivalent for any time period.

Hence, the correct answer is: True.

Question 18. The effective rate of interest is the actual annual rate of interest earned or paid, taking into account the effect of compounding frequency.

(A) True

(B) False

(C) True, but only for simple interest.

(D) False, it only applies to variable interest rates.

Answer:

Correct Option: (A) True


Explanation:

The effective rate of interest (also called effective annual rate or EAR) is the actual annual interest rate earned or paid on an investment or loan after taking into account the effect of compounding within the year.

It is especially relevant when interest is compounded more than once per year (e.g., quarterly or monthly), and it gives a more accurate picture than the nominal rate.

It is calculated using the formula:

$r = \left(1 + \frac{i}{m}\right)^m - 1$

[Where $i$ = nominal rate, $m$ = compounding periods per year]

Hence, the statement is true, and it applies to compound interest scenarios.

Question 19. If the nominal annual interest rate is 8% compounded semi-annually, what is the interest rate per compounding period ($i$)?

(A) 8%

(B) 4%

(C) 16%

(D) 2%

Answer:

Correct Option: (B) 4%


Explanation:

We are given:

  • Nominal annual interest rate = 8%
  • Compounded semi-annually → 2 compounding periods per year

To find the interest rate per compounding period ($i$), we use the formula:

$i = \dfrac{r}{m}$

[Where $r$ = nominal rate, $m$ = compounding periods per year]

$i = \dfrac{8\%}{2} = 4\%$

…(i)

Therefore, the interest rate per compounding period is: 4%

Question 20. The formula for the effective annual rate (EAR) for a nominal rate $j$ compounded $m$ times per year is:

(A) $EAR = (1 + j/m)^m - 1$

(B) $EAR = 1 + j/m$

(C) $EAR = (1 + j)^m - 1$

(D) $EAR = j \times m$

Answer:

Correct Option: (A) $EAR = (1 + j/m)^m - 1$


Explanation:

The Effective Annual Rate (EAR) is the actual annual interest rate taking compounding into account. When a nominal interest rate $j$ is compounded $m$ times per year, the formula for EAR is:

$EAR = \left(1 + \dfrac{j}{m}\right)^m - 1$

…(i)

Here,

  • $j$ is the nominal annual interest rate
  • $m$ is the number of compounding periods per year

This formula correctly accounts for the effect of compounding multiple times a year.

Therefore, option (A) is the correct answer.

Question 21. If the nominal annual interest rate is 10% compounded quarterly, what is the effective annual rate? (Approximate calculation)

(A) 10%

(B) 10.25%

(C) 10.38%

(D) 10.42%

Answer:

Correct Option: (C) 10.38%


Explanation:

We are given:

  • Nominal annual interest rate ($j$) = 10% = 0.10
  • Compounded quarterly → $m = 4$ times per year

To find the Effective Annual Rate (EAR), we use the formula:

$EAR = \left(1 + \dfrac{j}{m}\right)^m - 1$

…(i)

Substituting the values:

$EAR = \left(1 + \dfrac{0.10}{4}\right)^4 - 1$

…(ii)

$= (1 + 0.025)^4 - 1$

…(iii)

Now calculate:

$= (1.025)^4 - 1 = 1.1038 - 1 = 0.1038$

$EAR = 0.1038 = 10.38\%$

Therefore, the effective annual rate is: 10.38%

Question 22. Assertion (A): The effective rate of interest is always equal to the nominal rate of interest.

Reason (R): The effective rate considers the effect of compounding more frequently than annually, which increases the actual return.

(A) Both A and R are true and R is the correct explanation of A.

(B) Both A and R are true but R is not the correct explanation of A.

(C) A is true but R is false.

(D) A is false but R is true.

Answer:

Correct Option: (D) A is false but R is true.


Explanation:

Assertion (A): "The effective rate of interest is always equal to the nominal rate of interest."

This is false. The effective rate of interest is only equal to the nominal rate when interest is compounded annually. When compounding occurs more frequently (like quarterly or monthly), the effective rate becomes higher than the nominal rate.

Reason (R): "The effective rate considers the effect of compounding more frequently than annually, which increases the actual return."

This is true. More frequent compounding leads to a higher effective rate due to interest being calculated on accumulated amounts more often.

Therefore, the assertion is false, but the reason is true. Hence, the correct answer is:

Option (D).

Question 23. Case Study: Bank A offers 7% per annum simple interest. Bank B offers 6.8% per annum compounded semi-annually. Which bank offers a better interest rate over a 1-year period?

Calculate the effective annual rate for Bank B.

(A) $6.8\%$

(B) $(1 + 0.068/2)^2 - 1$

(C) $(1 + 0.034)^2 - 1 \approx 0.068156$

(D) Both (B) and (C).

Answer:

Correct Option: (D) Both (B) and (C).


Explanation:

Bank A: Offers Simple Interest at 7% per annum, so effective interest rate for 1 year = 7%.

Bank B: Offers Compound Interest at 6.8% per annum compounded semi-annually.

To find the Effective Annual Rate (EAR), we use the formula:

$EAR = \left(1 + \dfrac{r}{m}\right)^m - 1$

…(i)

Where:

  • $r = 6.8\% = 0.068$
  • $m = 2$ (since compounded semi-annually)

$EAR = \left(1 + \dfrac{0.068}{2}\right)^2 - 1$

…(ii)

$= (1 + 0.034)^2 - 1$

…(iii)

$= (1.034)^2 - 1 = 1.068156 - 1 = 0.068156 = 6.8156\%$

Conclusion: Since Bank A offers 7% and Bank B offers approximately 6.8156%, Bank A gives a better return over 1 year.

However, options (B) and (C) are both correct expressions of the EAR for Bank B. So the correct choice is:

Option (D).

Question 24. Case Study: (Same setup as Q23)

Which bank offers a better interest rate over a 1-year period?

(A) Bank A

(B) Bank B

(C) Both are equivalent

(D) Cannot be determined without the principal amount.

Answer:

Correct Option: (A) Bank A


Explanation:

From Question 23, we already calculated the effective annual rate (EAR) for Bank B:

$EAR = (1 + 0.034)^2 - 1 = 0.068156$

…(i)

Therefore, Bank B offers an effective rate of 6.8156%.

Bank A offers a simple interest rate of 7% per annum.

Comparison:

  • Bank A: 7.0000%
  • Bank B: 6.8156%

Since 7% > 6.8156%, Bank A offers a better interest rate over a 1-year period.

Hence, the correct answer is: Option (A).

Question 25. Present Value (PV) is the current worth of a future sum of money or stream of cash flows, given a specified rate of return.

(A) True

(B) False

(C) True, but only for simple interest.

(D) False, it is the value in the future.

Answer:

Correct Option: (A) True


Explanation:

The Present Value (PV) is defined as the current worth of a future sum of money or stream of cash flows, given a specified rate of return or discount rate.

It reflects the idea that a certain amount of money today is worth more than the same amount in the future due to its potential earning capacity (time value of money).

The formula for Present Value is:

$PV = \dfrac{FV}{(1 + r)^n}$

…(i)

Where:

  • $PV$ = Present Value
  • $FV$ = Future Value
  • $r$ = Rate of return (per period)
  • $n$ = Number of periods

This concept is applicable to both simple and compound interest scenarios, but it is especially relevant in compound interest and discounted cash flow calculations.

Hence, the statement is: True.

Question 26. If you want to have $\textsf{₹}\,10,000$ in 3 years and the interest rate is 6% per annum compounded annually, what is the present value you need to invest today?

(A) $10000 / (1 + 0.06)^3$

(B) $10000 \times (1 + 0.06)^3$

(C) $10000 / (1 + 0.06 \times 3)$

(D) $10000 - (10000 \times 0.06 \times 3)$

Answer:

Correct Option: (A) $\$10000 / (1 + 0.06)^3$


Explanation:

We are given:

  • Future Value (FV) = $\textsf{₹}\,10,000$
  • Time ($n$) = 3 years
  • Annual Compound Interest Rate ($r$) = 6% = 0.06

We use the Present Value (PV) formula for compound interest:

$PV = \dfrac{FV}{(1 + r)^n}$

…(i)

Substituting the given values:

$PV = \dfrac{10000}{(1 + 0.06)^3}$

[Using formula (i)]

So, to get $\textsf{₹}\,10,000$ in 3 years at 6% compounded annually, you must invest:

$\dfrac{10000}{(1.06)^3} = \dfrac{10000}{1.191016} \approx \textsf{₹}\,8396.86$

Hence, the correct answer is: Option (A).

Question 27. Net Present Value (NPV) is the difference between the present value of cash inflows and the present value of cash outflows over a period of time.

(A) True

(B) False

(C) True, but only for a single cash flow.

(D) False, it is the sum of cash flows.

Answer:

Correct Option: (A) True


Explanation:

Net Present Value (NPV) is a fundamental concept in financial decision-making. It helps determine the profitability of an investment or project.

Definition: NPV is the difference between the present value (PV) of cash inflows and the present value of cash outflows over a period of time.

$NPV = \sum\limits_{t=0}^{n} \dfrac{R_t}{(1 + r)^t} - C_0$

…(i)

Where:

  • $R_t$ = Net cash inflow during the period $t$
  • $r$ = Discount rate
  • $t$ = Time period
  • $C_0$ = Initial investment or cash outflow at time 0

This calculation accounts for the time value of money by discounting future cash flows to their present values and subtracting the initial outlay.

Hence, the statement is: True.

Question 28. If the NPV of a project is positive, it generally means:

(A) The project is not financially viable.

(B) The project is expected to generate returns greater than the required rate of return.

(C) The project is expected to generate returns exactly equal to the required rate of return.

(D) The initial investment is higher than the future cash flows.

Answer:

Correct Option: (B) The project is expected to generate returns greater than the required rate of return.


Explanation:

Net Present Value (NPV) is the difference between the present value of cash inflows and the present value of cash outflows, discounted at the required rate of return.

$NPV = \sum\limits_{t=0}^{n} \dfrac{R_t}{(1 + r)^t} - C_0$

…(i)

If $NPV > 0$, it means:

  • The project’s return is higher than the required rate of return.
  • The investment is expected to generate surplus value over its cost.
  • The project adds value to the firm and is considered financially viable.

Therefore, a positive NPV implies: The project is expected to generate returns greater than the required rate of return.

Question 29. Future Value (FV) is the value of an investment at a specified date in the future, assuming a certain interest rate.

(A) True

(B) False

(C) True, but only for simple interest.

(D) False, it is the value today.

Answer:

Correct Option: (A) True


Explanation:

Future Value (FV) refers to the value of a current asset or investment at a specified date in the future based on an assumed rate of growth or interest.

The formula for future value under compound interest is:

$FV = PV \times (1 + r)^n$

…(i)

Where:

  • $PV$ = Present Value (initial investment)
  • $r$ = Interest rate per period
  • $n$ = Number of periods

This concept applies to both simple and compound interest, though the formulas differ.

Thus, Future Value is indeed the value of an investment at a future date, making the statement True.

Question 30. If you invest $\textsf{₹}\,5000$ today at an interest rate of 7% per annum compounded annually, what will be its future value after 5 years?

(A) $5000 \times (1 + 0.07 \times 5)$

(B) $5000 \times (1 + 0.07)^5$

(C) $5000 / (1 + 0.07)^5$

(D) $5000 + 5000 \times 0.07 \times 5$

Answer:

Correct Option: (B) $5000 \times (1 + 0.07)^5$


Explanation:

Since the interest is compounded annually, the formula for future value (FV) is:

$FV = P \times (1 + r)^n$

…(i)

Where:

  • $P = 5000$ (Principal)
  • $r = 0.07$ (Annual interest rate)
  • $n = 5$ (Years)

Substituting the values in equation (i):

$FV = 5000 \times (1 + 0.07)^5$

…(ii)

This gives the correct future value under compound interest.

Therefore, Option (B) is correct.

Question 31. Assertion (A): Present value and Future value are related through the interest rate and time period.

Reason (R): $\text{FV} = \text{PV} \times (1+i)^n$ (for compound interest).

(A) Both A and R are true and R is the correct explanation of A.

(B) Both A and R are true but R is not the correct explanation of A.

(C) A is true but R is false.

(D) A is false but R is true.

Answer:

Correct Option: (A) Both A and R are true and R is the correct explanation of A.


Explanation:

Assertion (A): Present value (PV) and future value (FV) are indeed connected through both the interest rate and the time period. This is the core concept in the time value of money.

Reason (R): The formula for compound interest is:

$\text{FV} = \text{PV} \times (1 + i)^n$

…(i)

Where:

  • $\text{FV}$ = Future Value
  • $\text{PV}$ = Present Value
  • $i$ = Interest rate per period
  • $n$ = Number of periods

Equation (i) clearly demonstrates how PV and FV are mathematically related through the interest rate and time period.

Hence, both Assertion and Reason are true, and Reason is the correct explanation of Assertion.

Question 32. Case Study: A business is considering a project that requires an initial investment of $\textsf{₹}\,10,000$ and is expected to generate a cash inflow of $\textsf{₹}\,12,000$ in 2 years. The required rate of return is 5% per annum.

What is the present value of the expected cash inflow?

(A) $12000 / (1 + 0.05 \times 2)$

(B) $12000 / (1 + 0.05)^2$

(C) $12000 \times (1 + 0.05)^2$

(D) $12000 - 10000$

Answer:

Correct Option: (B) $12000 / (1 + 0.05)^2$


Explanation:

To find the present value (PV) of a future cash inflow using compound interest, the formula is:

$PV = \dfrac{FV}{(1 + r)^n}$

…(i)

Where:

  • $FV = 12000$ (Future Value)
  • $r = 0.05$ (Annual rate of return)
  • $n = 2$ years

Substituting in equation (i):

$PV = \dfrac{12000}{(1 + 0.05)^2}$

…(ii)

This will give the present value of the expected cash inflow discounted at 5% per annum over 2 years.

Therefore, Option (B) is correct.

Question 33. Case Study: (Same setup as Q32)

What is the Net Present Value (NPV) of the project?

(A) Present Value of Inflow + Initial Investment

(B) Initial Investment - Present Value of Inflow

(C) Present Value of Inflow - Initial Investment

(D) Initial Investment + Future Value of Inflow

Answer:

Correct Option: (C) Present Value of Inflow - Initial Investment


Explanation:

Net Present Value (NPV) is a financial metric used to assess the profitability of an investment or project. The formula for NPV is:

$\text{NPV} = \text{Present Value of Inflow} - \text{Initial Investment}$

…(i)

This formula evaluates whether the project will generate more value than it costs:

  • If NPV > 0 → The project is profitable.
  • If NPV < 0 → The project results in a loss.
  • If NPV = 0 → The project breaks even.

Therefore, the correct expression for calculating NPV is given in Option (C).

Question 34. An annuity is a series of equal payments made at regular intervals over a period of time.

(A) True

(B) False

(C) True, but only if the payments are interest payments.

(D) False, the payments must be unequal.

Answer:

Correct Option: (A) True


Explanation:

An annuity is defined as a series of equal payments made at regular intervals over a specified period of time. These payments can be made monthly, quarterly, annually, etc., and may include interest and/or principal components depending on the context.

Examples of annuities include:

  • Monthly pension payments
  • Home mortgage payments
  • Insurance premiums
  • Fixed deposits with regular payouts

Therefore, the statement is True, making Option (A) the correct answer.

Question 35. In a regular annuity, payments are made at the ________ of each period.

(A) beginning

(B) middle

(C) end

(D) any time

Answer:

Correct Option: (C) end


Explanation:

In a regular annuity (also known as an ordinary annuity), the payments are made at the end of each time period. This is the most common form of annuity used in financial transactions such as loan repayments, mortgage EMIs, and investment payouts.

For example:

  • If rent is paid at the end of every month, it's a regular annuity.
  • If you invest in a bond that pays interest annually at the year-end, it’s also a regular annuity.

Hence, the correct answer is Option (C).

Question 36. What is the future value of a regular annuity of $\textsf{₹}\,1000$ per year for 3 years at an interest rate of 5% per annum?

(A) $1000 \times (1.05)^3$

(B) $1000 + 1000(1.05) + 1000(1.05)^2$

(C) $1000 \times \frac{(1.05)^3 - 1}{0.05}$

(D) $1000 \times 3$

Answer:

Correct Option: (C) $1000 \times \frac{(1.05)^3 - 1}{0.05}$


Explanation:

To find the future value (FV) of a regular annuity, we use the formula:

$FV = A \times \frac{(1 + r)^n - 1}{r}$

[Formula for future value of a regular annuity]

Where:

  • $A = 1000$ (annual payment)
  • $r = 0.05$ (annual interest rate)
  • $n = 3$ (years)

Substituting the values:

$FV = 1000 \times \frac{(1.05)^3 - 1}{0.05}$

… (i)

This is exactly what is shown in Option (C), which is the correct choice.

Note: Other options are incorrect because:

  • Option (A): Calculates compound value of a single payment.
  • Option (B): Represents manually summing future values of each payment, but not in a formulaic or generalizable form.
  • Option (D): Ignores interest and assumes simple total.

Conclusion: The correct formula for computing the future value of a regular annuity is given in Option (C).

Question 37. Which of the following is a simple application of regular annuities?

(A) Loan repayments (EMI calculations).

(B) Saving for a future goal with regular deposits.

(C) Calculating the premium for an insurance policy with regular payments.

(D) All of the above.

Answer:

Correct Option: (D) All of the above


Explanation:

A regular annuity is a series of equal payments made at regular intervals, typically at the end of each period. It has many practical applications in personal finance and business, such as:

  • (A) Loan repayments (EMI calculations): When paying off a loan through EMIs, each payment is structured as part of a regular annuity.
  • (B) Saving for a future goal: Making equal monthly or annual deposits into a savings plan for future use is a form of annuity.
  • (C) Insurance premium payments: Regularly paying premiums for insurance coverage is also a typical annuity application.

Since all three options are valid applications of regular annuities,

Final Answer: Option (D)

Question 38. Assertion (A): The future value of an annuity is the sum of the future values of each individual payment.

Reason (R): Each payment earns interest from the time it is made until the end of the annuity period.

(A) Both A and R are true and R is the correct explanation of A.

(B) Both A and R are true but R is not the correct explanation of A.

(C) A is true but R is false.

(D) A is false but R is true.

Answer:

Correct Option: (A) Both A and R are true and R is the correct explanation of A.


Explanation:

Assertion (A): The future value (FV) of an annuity is indeed calculated by summing up the future values of each individual payment made during the annuity period. Each payment grows over time as it earns interest.

Reason (R): This is also correct. Each payment earns interest from the moment it is made until the end of the annuity term, which results in different future values for each installment depending on how early the payment was made.

Thus, both statements are true, and the reason correctly explains the assertion.

Final Answer: Option (A)

Question 39. Case Study: A person plans to deposit $\textsf{₹}\,500$ at the end of each year for 3 years in a savings account that earns 4% interest compounded annually.

What is the value of the first deposit at the end of 3 years?

(A) $\textsf{₹}\,500$

(B) $\textsf{₹}\,500 \times (1.04)^3$

(C) $\textsf{₹}\,500 \times (1.04)^2$

(D) $\textsf{₹}\,500 \times (1.04)$

Answer:

Correct Option: (C) $\textsf{₹}\,500 \times (1.04)^2$


Explanation:

In a regular annuity, each deposit earns compound interest based on the number of years it remains in the account.

Since the first deposit is made at the end of year 1 and the total duration is 3 years, this amount will remain in the account for 2 more years (from the end of year 1 to the end of year 3).

Therefore, its future value at the end of 3 years is calculated as:

$\textsf{₹}\,500 \times (1.04)^2$

…(i)

Hence, the value of the first deposit at the end of 3 years is Option (C).

Question 40. Case Study: (Same setup as Q39)

What is the total accumulated amount (future value) at the end of 3 years?

(A) $500 + 500(1.04) + 500(1.04)^2$

(B) $500(1.04) + 500(1.04)^2 + 500(1.04)^3$

(C) $500 \times \frac{(1.04)^3 - 1}{0.04}$

(D) Both (A) and (C) are correct formulations.

Answer:

Correct Option: (D) Both (A) and (C) are correct formulations.


Explanation:

The case involves a regular annuity where a person deposits $\textsf{₹}\,500$ at the end of each year for 3 years at an interest rate of 4% compounded annually.

Method 1: Term-wise accumulation

Each deposit earns compound interest for the number of years it remains in the account:

  • The 1st deposit is invested for 2 years: $\textsf{₹}\,500 \times (1.04)^2$
  • The 2nd deposit is invested for 1 year: $\textsf{₹}\,500 \times (1.04)$
  • The 3rd deposit earns no interest: $\textsf{₹}\,500$

So, total future value is:

$\textsf{₹}\,500 + 500(1.04) + 500(1.04)^2$

…(i)


Method 2: Using Future Value formula of Ordinary Annuity

The formula is:

$A = P \times \frac{(1 + r)^n - 1}{r}$

…(ii)

Where:
$P = \textsf{₹}\,500$, $r = 0.04$, $n = 3$

So,

$A = 500 \times \frac{(1.04)^3 - 1}{0.04}$

…(iii)


Conclusion: Both expressions (i) and (iii) represent the future value of the annuity and are correct.

Question 41. Tax is a compulsory financial charge imposed by a government on income, goods, or services, to fund public expenditures.

(A) True

(B) False

(C) True, but it is always voluntary.

(D) False, it is only on income.

Answer:

Correct Option: (A) True


Explanation:

Taxes are mandatory payments levied by the government on individuals and businesses. They are imposed on various economic activities such as:

  • Income (e.g., income tax)
  • Goods and services (e.g., GST, VAT)
  • Property (e.g., property tax)

These taxes are essential to fund public goods and services such as education, healthcare, infrastructure, defense, etc. Hence, they are not voluntary and not limited to income alone.

Therefore, the correct answer is True.

Question 42. Goods and Services Tax (GST) in India is a consumption tax levied on:

(A) Income only.

(B) Goods and services only.

(C) Production of goods only.

(D) Both income and goods/services.

Answer:

Correct Option: (B) Goods and services only


Explanation:

Goods and Services Tax (GST) in India is a consumption-based indirect tax levied on the supply of goods and services. It is charged at every stage of the supply chain — from manufacturer to consumer — but is ultimately borne by the end consumer.

GST is not levied on income; income is taxed separately under the Income Tax Act. Also, GST applies to the supply of goods and services, not just their production.

Therefore, the correct answer is (B) Goods and services only.

Question 43. If the price of an item before GST is $\textsf{₹}\,1000$ and the GST rate is 18%, what is the final price including GST?

(A) $\textsf{₹}\,1018$

(B) $\textsf{₹}\,1180$

(C) $\textsf{₹}\,1000 \times 0.18$

(D) $\textsf{₹}\,1000 - 180$

Answer:

Correct Option: (B) $\textsf{₹}\,1180$


Explanation:

Given:

  • Price before GST = $\textsf{₹}\,1000$
  • GST rate = 18%

Step 1: Calculate GST amount

GST = $18\%$ of $\textsf{₹}\,1000$ = $\frac{18}{100} \times 1000 = \textsf{₹}\,180$


Step 2: Add GST to the original price

Final Price = $\textsf{₹}\,1000 + \textsf{₹}\,180 = \textsf{₹}\,1180$

Therefore, the final price including GST is $\textsf{₹}\,1180$.

Question 44. Income Tax is levied on the income of individuals or entities, usually on an annual basis.

(A) True

(B) False

(C) True, but it is always a fixed percentage of income.

(D) False, it is only on expenditure.

Answer:

Correct Option: (A) True


Explanation:

Income Tax is a direct tax levied by the government on the income earned by individuals, Hindu Undivided Families (HUFs), firms, companies, and other entities.

It is assessed and paid annually based on the financial year's earnings and applicable tax slabs or rates defined under the Income Tax Act, 1961.

Therefore, the statement that "Income Tax is levied on the income of individuals or entities, usually on an annual basis" is true, and option (A) is the correct choice.

Question 45. Assertion (A): Tax is a source of revenue for the government.

Reason (R): Tax collected is used to fund public services like infrastructure, healthcare, and education.

(A) Both A and R are true and R is the correct explanation of A.

(B) Both A and R are true but R is not the correct explanation of A.

(C) A is true but R is false.

(D) A is false but R is true.

Answer:

Correct Option: (A) Both A and R are true and R is the correct explanation of A.


Explanation:

Assertion (A) is true because taxes are a major source of revenue for the government, which helps fund various development and welfare programs.

Reason (R) is also true because the tax collected is used to provide essential public services such as building infrastructure, running healthcare systems, and ensuring access to education.

Since R directly explains why A is true, option (A) is the most appropriate.

Question 46. Case Study: A person has a taxable income of $\textsf{₹}\,7,00,000$ per annum. The income tax slabs are: 0-2.5 Lakhs (0%), 2.5-5 Lakhs (5%), 5-10 Lakhs (20%). (Assume standard deduction rules are applied to arrive at taxable income and ignore rebates for simplicity in calculation focus).

What is the income tax payable on the first $\textsf{₹}\,5,00,000$ of taxable income?

(A) $\textsf{₹}\,12,500$

(B) $\textsf{₹}\,0$

(C) $\textsf{₹}\,25,000$

(D) $\textsf{₹}\,50,000$

Answer:

Correct Option: (A) ₹12,500


Explanation:

The first ₹5,00,000 of taxable income is taxed as per the given slab rates:

  • ₹0 to ₹2,50,000: 0% → ₹0 tax
  • ₹2,50,001 to ₹5,00,000: 5% → 5% of ₹2,50,000 = ₹12,500

Total tax on the first ₹5,00,000 = ₹12,500

Therefore, the correct answer is option (A).

Question 47. Case Study: (Same setup as Q46)

What is the total income tax payable on the taxable income of $\textsf{₹}\,7,00,000$?

(A) $\textsf{₹}\,12,500$

(B) $\textsf{₹}\,12,500 + 20\%$ of $(\textsf{₹}\,7,00,000 - \textsf{₹}\,5,00,000)$

(C) $\textsf{₹}\,12,500 + \textsf{₹}\,40,000 = \textsf{₹}\,52,500$

(D) Both (B) and (C) are correct.

Answer:

Correct Option: (D) Both (B) and (C) are correct


Explanation:

For a taxable income of ₹7,00,000, using the slab system:

  • ₹0 – ₹2,50,000: 0% → ₹0
  • ₹2,50,001 – ₹5,00,000: 5% → ₹12,500
  • ₹5,00,001 – ₹7,00,000: 20% → 20% of ₹2,00,000 = ₹40,000

Total tax = ₹12,500 + ₹40,000 = ₹52,500

Option (B) expresses the breakdown and option (C) gives the final value. Hence, both (B) and (C) are correct.

Question 48. Bills for services like electricity, water, and gas often include charges besides the consumption amount. Which of the following is a type of fixed charge on a bill?

(A) Consumption based charge (per unit).

(B) Tax on consumption.

(C) A monthly meter rent regardless of usage.

(D) Surcharge based on usage exceeding a limit.

Answer:

Correct Option: (C) A monthly meter rent regardless of usage


Explanation:

Fixed charges are those that do not depend on how much of the service you use. A monthly meter rent is a typical example — it is charged irrespective of consumption.

  • (A) and (D) are variable charges based on usage.
  • (B) is also a variable component (tax) applied on consumption.
  • (C) is the only fixed charge listed.

Therefore, the correct answer is option (C).

Question 49. A tariff rate is the rate charged per unit of consumption for a service (e.g., electricity per kWh, water per liter/KL).

(A) True

(B) False

(C) True, but it's always a fixed rate for all consumption levels.

(D) False, it is only for taxes.

Answer:

Correct Option: (A) True


Explanation:

A tariff rate refers to the price charged per unit of consumption for services such as electricity, water, gas, etc. It is usually expressed as:

  • ₹ per kilowatt-hour (kWh) for electricity
  • ₹ per kiloliter (KL) or liter for water

This rate can be fixed or variable (e.g., slab-wise), but the definition itself is correct.

Hence, the correct answer is option (A) True.

Question 50. Surcharge on a bill is typically an additional charge added to the basic bill amount for reasons like peak demand, high consumption, or government policies.

(A) True

(B) False

(C) True, but it always reduces the bill amount.

(D) False, it is a fixed charge.

Answer:

Correct Option: (A) True


Explanation:

A surcharge is an additional charge added to the regular bill amount. It is typically imposed due to:

  • High consumption of resources
  • Usage during peak demand periods
  • Government-imposed levies or policy adjustments

It is meant to discourage excessive usage or to recover extra costs incurred by the provider.

Hence, the correct answer is option (A) True.

Question 51. Case Study: An electricity bill shows the following details: Consumption: 150 units. Tariff Rate: $\textsf{₹}\,8$ per unit. Fixed Charge: $\textsf{₹}\,100$. Surcharge: 10% of the total energy charge.

What is the total energy charge (consumption amount)?

(A) $\textsf{₹}\,150$

(B) $\textsf{₹}\,800$

(C) $\textsf{₹}\,1200$

(D) $\textsf{₹}\,1300$

Answer:

Correct Option: (C) $\textsf{₹}\,1200$


Given:

Consumption = 150 units

Tariff rate = $\textsf{₹}\,8$ per unit


To Find: Total energy charge (only consumption amount)


Solution:

Total energy charge = Units consumed $\times$ Rate per unit

$150 \times 8 = 1200$

…(i)

Hence, the total energy charge is $\textsf{₹}\,1200$.

Question 52. Case Study: (Same setup as Q51)

What is the total bill amount (excluding any applicable taxes like GST on the final amount)?

(A) Total Energy Charge + Fixed Charge

(B) Total Energy Charge + Fixed Charge + Surcharge

(C) $\textsf{₹}\,1200 + \textsf{₹}\,100 + \textsf{₹}\,120$

(D) Both (B) and (C) are correct.

Answer:

Correct Option: (D) Both (B) and (C) are correct


From Question 51:

  • Total Energy Charge = ₹1200
  • Fixed Charge = ₹100
  • Surcharge = ₹120

Total Bill Amount (excluding taxes):

= ₹1200 (Energy) + ₹100 (Fixed) + ₹120 (Surcharge)

= ₹1420


Option (B) correctly includes all components of the bill.

Option (C) gives the same numerical calculation.

Hence, the correct answer is (D) Both (B) and (C) are correct.

Question 53. Which of the following is NOT a concept typically associated with Bills and Tariffs?

(A) Fixed Charge

(B) Discount

(C) Compound Interest

(D) Surcharge

Answer:

Correct Option: (C) Compound Interest


Explanation:

  • Fixed Charge – A standard charge included in many utility bills regardless of usage.
  • Discount – Sometimes offered for early payment or special cases.
  • Surcharge – An additional fee, often applied for late payment or peak usage.
  • Compound Interest – This relates to finance and investment, not directly to billing or tariffs.

Therefore, Compound Interest is not typically a concept associated with bills and tariffs.

Question 54. If a bill includes a "Service Charge", it is typically a fee for the service provided, often calculated as a percentage of the bill or a fixed amount.

(A) True

(B) False

(C) True, but only in restaurants.

(D) False, it is the same as tax.

Answer:

Correct Option: (A) True


Explanation:

A Service Charge is typically a fee added to a bill for services rendered. It may be:

  • A fixed amount (e.g., ₹50 per service)
  • Or calculated as a percentage of the bill (e.g., 10% of the total amount)

It is commonly applied in various industries such as hospitality, travel, and certain retail or personal services—not just in restaurants.

Hence, the statement is True.

Question 55. Match the financial concept with its description:

(i) Simple Interest

(ii) Compound Interest

(iii) Effective Rate

(iv) NPV

(a) Interest on principal plus accumulated interest.

(b) Actual annual interest considering compounding.

(c) Current value of future cash flows minus initial cost.

(d) Interest only on the principal amount.

(A) (i)-(d), (ii)-(a), (iii)-(b), (iv)-(c)

(B) (i)-(a), (ii)-(d), (iii)-(b), (iv)-(c)

(C) (i)-(d), (ii)-(a), (iii)-(c), (iv)-(b)

(D) (i)-(a), (ii)-(d), (iii)-(c), (iv)-(b)

Answer:

Correct Option: (A) (i)-(d), (ii)-(a), (iii)-(b), (iv)-(c)


Matching the concepts:

  • (i) Simple Interest – (d) Interest only on the principal amount.
  • (ii) Compound Interest – (a) Interest on principal plus accumulated interest.
  • (iii) Effective Rate – (b) Actual annual interest considering compounding.
  • (iv) NPV (Net Present Value) – (c) Current value of future cash flows minus initial cost.

This matching accurately defines each financial term.

Question 56. If a sum of money doubles itself at simple interest in 10 years, what is the annual rate of interest?

(A) 5%

(B) 10%

(C) 15%

(D) 20%

Answer:

Correct Option: (B) 10%


Given: A sum doubles in 10 years at simple interest.

Let: Principal = $\textsf{₹}\,P$, Time = 10 years

Then Amount = $\textsf{₹}\,2P$

So, Interest = Amount − Principal = $\textsf{₹}\,2P - \textsf{₹}\,P = \textsf{₹}\,P$

Using Simple Interest formula: $$ I = \frac{P \times R \times T}{100} $$

$P = \frac{P \times R \times 10}{100}$

…(i)

Canceling $P$ from both sides (as $P ≠ 0$):

$1 = \frac{R \times 10}{100}$

Multiplying both sides by 100:

$100 = 10R$

Dividing both sides by 10:

$R = 10\%$


Conclusion: The annual rate of interest is 10%.

Question 57. If a sum of money doubles itself at compound interest, compounded annually, in 10 years, the annual rate of interest is:

(A) Greater than the simple interest rate required to double the money in 10 years.

(B) Less than the simple interest rate required to double the money in 10 years.

(C) Equal to the simple interest rate required to double the money in 10 years.

(D) Cannot be compared without knowing the principal.

Answer:

Correct Option: (A) Greater than the simple interest rate required to double the money in 10 years.


Explanation:

In simple interest (SI), to double the money in 10 years, the required rate is:

Let Principal = ₹P, then Interest = ₹P, Time = 10 years

Using SI formula:

\( I = \frac{P \times R \times T}{100} \Rightarrow P = \frac{P \times R \times 10}{100} \)

Solving: \( R = 10\% \)

In compound interest (CI), the formula is:

\( A = P(1 + r)^t \)

To double: \( 2P = P(1 + r)^{10} \Rightarrow (1 + r)^{10} = 2 \)

Taking 10th root: \( 1 + r = \sqrt[10]{2} \approx 1.0718 \Rightarrow r \approx 0.0718 \) or 7.18%

Conclusion: The compound interest rate required to double a sum in 10 years is approximately 7.18%, which is less than the 10% simple interest rate.

However, the question asks which rate is greater, and compares simple interest rate to compound interest rate. Since CI achieves doubling with a lower rate, it means:

The simple interest rate must be higher to achieve the same result.

Hence, the compound interest rate is less than the simple interest rate.

Correction: The correct option is: (B) Less than the simple interest rate required to double the money in 10 years.

Question 58. If the nominal rate is 6% compounded monthly, the number of compounding periods in a year is:

(A) 1

(B) 6

(C) 12

(D) 365

Answer:

Correct Option: (C) 12


Explanation:

The nominal rate is given as 6% compounded monthly.

This means that interest is calculated and added to the principal every month.

Since there are 12 months in a year, the number of compounding periods per year is:

12


Conclusion: There are 12 compounding periods in a year when the nominal rate is compounded monthly.

Question 59. The present value of a future amount will be lower if:

(A) The interest rate is higher.

(B) The time period is longer.

(C) Both (A) and (B).

(D) The interest rate is lower and the time period is shorter.

Answer:

Correct Option: (C) Both (A) and (B)


Explanation:

The present value (PV) of a future amount is calculated using the formula:

\( PV = \frac{FV}{(1 + r)^n} \)

  • \( FV \) = future value
  • \( r \) = interest rate (discount rate)
  • \( n \) = time period in years

From the formula, it's clear that:

  • If the interest rate increases (\( r \) ↑), the denominator becomes larger, making PV smaller.
  • If the time period increases (\( n \) ↑), the exponent increases, again making PV smaller.

So, both a higher interest rate and a longer time period will reduce the present value of a future amount.


Conclusion: The present value will be lower if both the interest rate is higher and the time period is longer.

Question 60. An annuity due is an annuity where payments are made at the _________ of each period.

(A) beginning

(B) end

(C) middle

(D) random time

Answer:

Correct Option: (A) beginning


Explanation:

An annuity due is a type of annuity in which payments are made at the beginning of each period, rather than at the end.

This is different from an ordinary annuity, where payments are made at the end of each period.


Conclusion: In an annuity due, payments occur at the beginning of each period.

Question 61. Which tax is levied on the value added at each stage of production and distribution?

(A) Income Tax

(B) Wealth Tax

(C) Goods and Services Tax (GST)

(D) Property Tax

Answer:

Correct Option: (C) Goods and Services Tax (GST)


Explanation:

The Goods and Services Tax (GST) is a value-added tax that is levied on the supply of goods and services at each stage of production and distribution.

  • It ensures that tax is applied only on the value added at each stage.
  • Businesses can claim input tax credit on the tax paid on purchases, effectively taxing only the value they add.

Other options:

  • Income Tax – charged on income earned by individuals or entities.
  • Wealth Tax – levied on the net wealth of individuals (now abolished in India).
  • Property Tax – imposed on property ownership, usually by local governments.

Conclusion: GST is the tax levied on the value added at each stage of the supply chain.

Question 62. In the context of an electricity bill, "units consumed" refers to:

(A) The total amount payable.

(B) The fixed charge amount.

(C) The amount of electrical energy used, typically measured in kWh.

(D) The tariff rate per unit.

Answer:

Correct Option: (C) The amount of electrical energy used, typically measured in kWh.


Explanation:

In an electricity bill, "units consumed" refers to the total electrical energy used during the billing period. It is measured in kilowatt-hours (kWh), where:

1 unit = 1 kWh

  • This value is calculated by taking the difference between the current and previous meter readings.
  • The number of units consumed is then multiplied by the tariff rate to determine the energy charge.

Other options explained:

  • (A) Total amount payable includes energy charge + fixed charges + taxes, not just units.
  • (B) Fixed charge is a constant monthly charge, irrespective of usage.
  • (D) Tariff rate is the cost per unit (kWh), not the unit itself.

Conclusion: "Units consumed" correctly refers to the electrical energy used, measured in kWh.

Question 63. If a loan of $\textsf{₹}\,1,00,000$ is to be repaid in 3 equal annual installments at 8% interest compounded annually, this scenario involves the calculation of a(n):

(A) Simple Interest.

(B) Compound Interest.

(C) Annuity Payment.

(D) NPV.

Answer:

Correct Option: (C) Annuity Payment.


Explanation:

The given scenario involves repaying a loan through equal annual installments over a period of 3 years, with interest compounded annually at 8%. This is a classic case of an annuity payment.

  • An annuity is a series of equal payments made at regular intervals over a specified period.
  • Here, the loan is being repaid through 3 equal payments (installments) — this defines a loan amortization annuity.
  • Each installment includes a portion of the principal and compound interest.

Other options:

  • Simple Interest (A): Would apply if interest were not compounded and was calculated only on the principal.
  • Compound Interest (B): While interest is compounded, the focus here is on structured payments, i.e., annuity.
  • NPV (D): Net Present Value is used to evaluate investment profitability, not to calculate loan repayments.

Conclusion: The correct financial concept involved is Annuity Payment.

Question 64. What is the GST amount on an item priced at $\textsf{₹}\,2500$ before tax, if the GST rate is 12%?

(A) $\textsf{₹}\,2500 \times 0.12$

(B) $\textsf{₹}\,300$

(C) $\textsf{₹}\,2500 + 300$

(D) Both (A) and (B).

Answer:

Correct Option: (D) Both (A) and (B).


Explanation:

The GST (Goods and Services Tax) amount on an item priced at ₹2500 with a GST rate of 12% is calculated as follows:

GST = ₹2500 × 12% = ₹2500 × 0.12 = ₹300

So:

  • Option (A): ₹2500 × 0.12 is the correct formula.
  • Option (B): ₹300 is the correct computed value.
  • Option (C): ₹2500 + 300 gives the total price including GST (₹2800), but this is not the GST amount alone.

Conclusion: Both (A) and (B) correctly represent the GST amount — hence, the correct answer is (D).

Question 65. Which concept helps in comparing interest rates with different compounding frequencies?

(A) Simple Interest

(B) Compound Interest

(C) Effective Rate of Interest

(D) Nominal Rate of Interest

Answer:

Correct Option: (C) Effective Rate of Interest


Explanation:

  • The Effective Rate of Interest (ERI) is used to compare interest rates that are compounded at different frequencies (e.g., annually, semi-annually, quarterly).
  • It represents the actual interest earned or paid in a year, considering the effects of compounding.
  • This allows fair comparison between financial products with varying compounding intervals.

Other Options:

  • Simple Interest (A): Does not consider compounding at all.
  • Compound Interest (B): Considers compounding but doesn't facilitate cross-comparison of rates with different frequencies.
  • Nominal Rate of Interest (D): Is the stated rate, not adjusted for compounding.

Conclusion: Effective Rate of Interest is the correct concept used for comparing interest rates with different compounding frequencies.

Question 66. The sum of the present values of all future cash flows, both positive and negative, less the initial investment, is called the _________.

(A) Future Value

(B) Present Value

(C) Net Present Value

(D) Accumulated Amount

Answer:

Correct Option: (C) Net Present Value


Explanation:

  • Net Present Value (NPV) is the sum of the present values of all future cash inflows and outflows associated with an investment, minus the initial investment cost.
  • It is a key financial metric used in capital budgeting to evaluate the profitability of an investment or project.
  • If NPV is positive, the investment is considered profitable; if negative, it is likely to result in a loss.

Other Options:

  • Future Value (A): The value of an investment at a future date, not adjusted to today's value.
  • Present Value (B): The value today of a single future cash flow, not the total net value of all cash flows.
  • Accumulated Amount (D): Usually refers to the future sum of money including interest, not discounted back to present value.

Conclusion: The correct term is Net Present Value (NPV), which accurately describes the difference between the present value of future cash flows and the initial investment.

Question 67. If an investment project has an NPV of zero, it means the project is expected to yield exactly the required rate of return.

(A) True

(B) False

(C) True, but it's still not a good project.

(D) False, it means the project will lose money.

Answer:

Correct Option: (A) True


Explanation:

  • When the Net Present Value (NPV) of a project is zero, it means the present value of future cash inflows is exactly equal to the initial investment (cost).
  • This implies that the project is expected to generate a return equal to the required rate of return (or discount rate).
  • In such a case, the investor neither gains nor loses in terms of present value—it’s a break-even situation.

Why Other Options Are Incorrect:

  • (B) False: Incorrect because a zero NPV does not imply a loss; it implies the investment meets the minimum acceptable return.
  • (C) True, but it's still not a good project: Misleading — while the project may not generate excess returns, it’s not “bad” financially; it simply breaks even.
  • (D) False, it means the project will lose money: Incorrect — a negative NPV indicates a loss, not zero.

Conclusion: A project with NPV = 0 meets the required return, making Option (A) the correct answer.

Question 68. The concept of a sinking fund, where regular payments are made to accumulate a future sum, is an application of:

(A) Simple Interest.

(B) Compound Interest.

(C) Annuity.

(D) Present Value.

Answer:

Correct Option: (C) Annuity


Explanation:

  • A sinking fund is a fund established by making regular, periodic payments to accumulate a specified amount in the future.
  • This concept is a classic example of an annuity, where equal payments are made at regular intervals and invested to earn interest (usually compounded).
  • The goal is to accumulate a future sum, for example, to repay a debt, replace equipment, or fund future liabilities.

Why Other Options Are Incorrect:

  • (A) Simple Interest: Does not account for reinvestment or compounding of payments.
  • (B) Compound Interest: While compound interest is involved in the calculations, the key structure is that of an annuity.
  • (D) Present Value: Sinking fund focuses on accumulating a future value, not discounting back to present value.

Conclusion: A sinking fund is an application of an annuity, making Option (C) the correct answer.

Question 69. Which of the following might be considered a "supply bill"?

(A) Electricity bill

(B) Water bill

(C) Piped gas bill

(D) All of the above

Answer:

Correct Option: (D) All of the above


Explanation:

  • A supply bill typically refers to any bill related to the provision of essential utility services.
  • This includes:
    • Electricity bill – for electrical energy supply.
    • Water bill – for municipal or private water supply.
    • Piped gas bill – for domestic or industrial fuel gas supply.
  • All of these are recurring bills generated for the consumption of basic utilities.

Conclusion: Since electricity, water, and piped gas are all considered supply services, Option (D) – "All of the above" – is correct.

Question 70. If you want to determine how much a future sum of money is worth today, you calculate its _________.

(A) Future Value

(B) Present Value

(C) Accumulated Amount

(D) Net Present Value

Answer:

Correct Option: (B) Present Value


Explanation:

  • Present Value (PV) is the current worth of a future sum of money or stream of cash flows, given a specified rate of return (discount rate).
  • It helps determine how much a future amount is worth in today's terms, considering the time value of money.

Why Other Options Are Incorrect:

  • (A) Future Value: Calculates how much a present amount will grow to in the future, not its value today.
  • (C) Accumulated Amount: Refers to the total future amount after compounding, not the value today.
  • (D) Net Present Value (NPV): Involves evaluating an investment by subtracting the initial investment from the present value of future cash flows. It is more comprehensive than just calculating present value.

Conclusion: To find out what a future sum is worth today, you calculate its Present Value, making Option (B) the correct answer.

Question 71. The difference between the simple interest and compound interest on a sum of $\textsf{₹}\,1000$ at 10% per annum for 2 years is:

(A) $\textsf{₹}\,0$

(B) $\textsf{₹}\,10$

(C) $\textsf{₹}\,20$

(D) $\textsf{₹}\,100$

Answer:

Correct Option: (B) ₹10


Explanation:

We are given:

  • Principal (P) = ₹1000
  • Rate (R) = 10% per annum
  • Time (T) = 2 years

Simple Interest (SI):

SI = (P × R × T) / 100 = (1000 × 10 × 2) / 100 = ₹200

Compound Interest (CI):

CI = P × (1 + R/100)T − P = 1000 × (1.10)2 − 1000 = 1000 × 1.21 − 1000 = ₹210

Difference:

CI − SI = ₹210 − ₹200 = ₹10


Conclusion: The difference between compound and simple interest in this case is ₹10, so the correct answer is Option (B).

Question 72. If a loan has an initial principal of P and is repaid with equal periodic payments (annuity), a part of each payment goes towards interest and the rest towards the _________.

(A) future value

(B) principal repayment

(C) surcharge

(D) tax

Answer:

Correct Option: (B) principal repayment


Explanation:

  • In an annuity-based loan repayment structure (like EMIs), each payment consists of two components:
    • Interest: Charged on the remaining loan balance.
    • Principal Repayment: The part that reduces the outstanding loan amount.
  • As the loan progresses, the interest portion decreases, and the principal repayment portion increases.

Why Other Options Are Incorrect:

  • (A) Future Value: Refers to the value of an investment at a future date, not related to the structure of loan repayments.
  • (C) Surcharge: A tax or fee, not related to loan amortization.
  • (D) Tax: Not a component of periodic loan repayments.

Conclusion: A part of each loan payment in an annuity goes towards principal repayment, making Option (B) the correct answer.

Question 73. The formula for the present value of a regular annuity is used to calculate:

(A) The total accumulated amount of the payments in the future.

(B) The single lump sum amount today that is equivalent to the series of future payments.

(C) The interest earned on each payment.

(D) The total number of payments.

Answer:

Correct Option: (B) The single lump sum amount today that is equivalent to the series of future payments.


Explanation:

  • The present value of a regular annuity represents the value today (present value) of a series of equal payments made at regular intervals in the future, discounted using a specific interest rate.
  • It answers the question: “What is the current worth of a set of future payments?”

Why Other Options Are Incorrect:

  • (A): Refers to the future value of an annuity, not the present value.
  • (C): Interest earned is not what the present value formula calculates directly.
  • (D): The number of payments is a variable in the formula, not the result of it.

Conclusion: The formula for present value helps determine how much a series of future annuity payments is worth in today's terms, making Option (B) the correct answer.

Question 74. If the nominal annual interest rate is 12% compounded monthly, the annual effective rate will be:

(A) Less than 12%.

(B) Exactly 12%.

(C) Greater than 12%.

(D) Cannot be determined without the principal.

Answer:

Correct Option: (C) Greater than 12%


Explanation:

When interest is compounded more frequently than once a year, the Effective Annual Rate (EAR) becomes higher than the Nominal Annual Rate.

The formula to calculate the Effective Annual Rate (EAR) from the Nominal Annual Rate compounded monthly is:

$\text{EAR} = \left(1 + \frac{r}{n}\right)^n - 1$

… (i)

Where:

  • $r = 0.12$ (Nominal annual interest rate = 12%)
  • $n = 12$ (compounded monthly, so 12 times a year)

Substituting the values into equation (i):

$\text{EAR} = \left(1 + \frac{0.12}{12}\right)^{12} - 1$

… (ii)

⇒ $\text{EAR} = (1 + 0.01)^{12} - 1$

⇒ $\text{EAR} = (1.01)^{12} - 1$

⇒ $\text{EAR} \approx 1.1268 - 1$

⇒ $\text{EAR} \approx 0.1268$ or 12.68%

Therefore, the annual effective rate is approximately 12.68%, which is greater than the nominal rate of 12%.

Question 75. The taxable income is the income on which tax is calculated, after accounting for various deductions and exemptions allowed by tax laws.

(A) True

(B) False

(C) True, but there are no deductions allowed in India.

(D) False, tax is calculated on gross income.

Answer:

Correct Option: (A) True


Explanation:

Taxable income refers to the portion of an individual’s or entity’s income that is subject to taxation by the government.

This is not the gross income, but the income that remains after subtracting allowable deductions (like investments under section 80C, interest on home loans, etc.) and exemptions (such as House Rent Allowance, Leave Travel Allowance, etc.).

Hence, tax is calculated on the net taxable income rather than the gross total income.

Therefore, the given statement is true.

Question 76. Which of the following factors typically affects the total amount of an electricity bill?

(A) Number of units consumed.

(B) Applicable tariff rates.

(C) Fixed charges and surcharges.

(D) All of the above.

Answer:

Correct Option: (D) All of the above


Explanation:

The total electricity bill is calculated by considering multiple factors, which include:

  • Number of units consumed: The more electricity (in kWh or units) you use, the higher the bill will be.
  • Applicable tariff rates: These rates vary by consumer category (residential, commercial, industrial) and slab. Higher consumption can move a user into a higher slab rate.
  • Fixed charges and surcharges: Utilities also impose fixed monthly charges and other surcharges (e.g., fuel adjustment cost, electricity duty) regardless of consumption level.

All these components together determine the final amount payable.

Hence, option (D) is the correct answer.

Question 77. If a project's Internal Rate of Return (IRR) is greater than the required rate of return (discount rate), it implies that the project's NPV is:

(A) Positive.

(B) Negative.

(C) Zero.

(D) Undetermined.

Answer:

Correct Option: (A) Positive


Explanation:

The Net Present Value (NPV) is calculated by discounting the future cash flows of a project at a specified required rate of return (also known as the discount rate).

The Internal Rate of Return (IRR) is the rate at which the NPV of all future cash flows becomes zero.

Now, if the IRR > required rate of return, it means that the project earns more than the minimum acceptable return. Hence, discounting the future cash flows at the required rate will result in a positive NPV.

Therefore, it implies:

$\text{NPV} \gt 0$

          ... (i)

Hence, option (A) is the correct answer.

Question 78. In India, Income Tax is a ________ tax, meaning the tax rate increases as the taxable income increases.

(A) proportional

(B) regressive

(C) progressive

(D) fixed

Answer:

Correct Option: (C) progressive


Explanation:

India follows a progressive tax system for income tax, where the tax rate increases with an increase in taxable income.

This means individuals with higher incomes pay taxes at higher rates, and those with lower incomes pay at lower rates, ensuring equity and fairness in the tax burden.

This can be contrasted with:

  • Proportional tax (Option A): A flat rate of tax regardless of income.
  • Regressive tax (Option B): Lower-income individuals pay a higher percentage of their income than higher-income individuals.
  • Fixed tax (Option D): A constant tax amount regardless of income.

Hence, option (C) is the correct answer.

Question 79. If the GST rate is 18% on an item, and the final price is $\textsf{₹}\,1416$, what was the price before GST?

(A) $\textsf{₹}\,1416 - (1416 \times 0.18)$

(B) $\textsf{₹}\,1416 / 1.18$

(C) $\textsf{₹}\,1416 \times 0.18$

(D) $\textsf{₹}\,1416 - 18$

Answer:

Correct Option: (B) $\textsf{₹}\,1416 / 1.18$


Explanation:

The final price of an item after GST is the sum of the original price and the GST amount.

If the GST rate is 18%, then the final price is 118% of the original price, or in other words, $1.18$ times the original price.

Let the price before GST be $x$. Then,

$1.18x = 1416$

          ... (i)

Dividing both sides by $1.18$:

$x = \dfrac{1416}{1.18}$

[Original Price]           ... (ii)

So, the original price before GST is $\textsf{₹}\,\dfrac{1416}{1.18}$.

Hence, option (B) is the correct answer.

Question 80. Which concept is useful for evaluating and comparing different investment opportunities with different cash flow patterns over time?

(A) Simple Interest

(B) Compound Interest

(C) Net Present Value (NPV)

(D) Annuity

Answer:

Correct Option: (C) Net Present Value (NPV)


Explanation:

Net Present Value (NPV) is a financial metric used to evaluate and compare different investment opportunities based on their expected future cash flows, discounted to the present value.

The formula for NPV is:

$NPV = \sum\limits_{t=0}^{n} \dfrac{C_t}{(1 + r)^t}$

[Where $C_t$ = cash flow at time $t$, $r$ = discount rate]           ... (i)

Key Features:

  • Accounts for the time value of money.
  • Useful for comparing projects with varying cash flow timings and amounts.
  • A positive NPV indicates a profitable investment.

Other options are not suitable for this purpose:

  • Simple Interest: Only considers interest on the principal, not useful for complex cash flows.
  • Compound Interest: Deals with interest on accumulated interest, not suitable for evaluating investment profitability.
  • Annuity: Refers to a fixed series of payments, not a tool for comparison of investment opportunities.

Therefore, the correct answer is (C) Net Present Value (NPV).

Question 81. The formula for the present value of an amount received in the future is derived from the formula for _________.

(A) Simple Interest

(B) Compound Interest (specifically, the future value formula)

(C) Annuity

(D) NPV

Answer:

Correct Option: (B) Compound Interest (specifically, the future value formula)


Explanation:

The concept of present value is based on the time value of money, which is a core idea in compound interest.

The future value (FV) formula under compound interest is:

$FV = PV \times (1 + r)^n$

[Future value formula]           ... (i)

By rearranging this equation to solve for Present Value (PV), we get:

$PV = \dfrac{FV}{(1 + r)^n}$

[Present value formula]           ... (ii)

Where,

  • $FV$ = Future Value
  • $PV$ = Present Value
  • $r$ = Rate of interest per period
  • $n$ = Number of periods

This derivation shows that present value is directly obtained from the compound interest formula.

Hence, the correct answer is (B).

Question 82. If the GST on a service is $\textsf{₹}\,540$ at a rate of 18%, what was the cost of the service before GST?

(A) $\textsf{₹}\,540 / 0.18$

(B) $\textsf{₹}\,540 \times 0.18$

(C) $\textsf{₹}\,540 / 1.18$

(D) $\textsf{₹}\,540 + 18$

Answer:

Correct Option: (A) $\textsf{₹}\,540 / 0.18$


Explanation:

We are given the GST amount and the GST rate. We need to find the original cost (i.e., the value before GST was added).

Let the original cost of the service be $x$.

GST is 18% of the original cost, i.e.,

$18\% \text{ of } x = 540$

[Given]           ... (i)

We convert 18% to decimal:

$\dfrac{18}{100} \cdot x = 540$

[Converting percentage to fraction]           ... (ii)

Now solving for $x$:

$x = \dfrac{540}{0.18}$

[Dividing both sides by 0.18]           ... (iii)

Therefore, the original cost before GST was:

$\textsf{₹}\,540 \div 0.18 = \textsf{₹}\,3000$

Hence, the correct answer is (A).



Short Answer Type Questions

Question 1. Define Simple Interest (SI). Write the formula for calculating SI.

Answer:

Definition:

Simple Interest (SI) is the interest calculated on the principal amount for a specified period at a given rate of interest. It does not take into account any interest on previously earned interest.


Formula:

The formula to calculate Simple Interest is:

$SI = \dfrac{P \times R \times T}{100}$

[Formula for Simple Interest]           ... (i)

Where:

  • $SI$ = Simple Interest
  • $P$ = Principal amount
  • $R$ = Rate of interest per annum
  • $T$ = Time (in years)

Note: The units of $R$ and $T$ must be consistent, typically rate per annum and time in years.

Question 2. Define Compound Interest (CI). Explain the difference between SI and CI.

Answer:

Definition:

Compound Interest (CI) is the interest calculated on the initial principal as well as on the accumulated interest from previous periods. This means that interest is added to the principal at the end of each compounding period, and future interest is calculated on the new total.


Formula for Compound Interest (when compounded annually):

If the principal is $P$, the annual rate of interest is $R\%$, and the time is $T$ years, then:

$A = P \left(1 + \dfrac{R}{100} \right)^T$

[Amount after $T$ years]           ... (i)

$CI = A - P$

[Compound Interest]           ... (ii)


Difference between SI and CI:

Simple Interest (SI) Compound Interest (CI)
Interest is calculated only on the original principal. Interest is calculated on the principal and the accumulated interest.
Same interest is earned every year. Interest increases every year as it is added to the principal.
Formula: $SI = \dfrac{P \times R \times T}{100}$ Formula: $CI = P \left(1 + \dfrac{R}{100} \right)^T - P$
Growth is linear. Growth is exponential.

Question 3. A principal amount of $\textsf{₹} 15,000$ is invested at $8\%$ per annum simple interest for 3 years. Calculate the simple interest earned.

Answer:

Given:

Principal, $P = \textsf{₹} 15,000$

Rate of interest, $R = 8\%$ per annum

Time, $T = 3$ years


To Find:

Simple Interest (SI)


Formula:

$SI = \dfrac{P \times R \times T}{100}$

[Formula for Simple Interest]           ... (i)


Substituting the values:

$SI = \dfrac{15000 \times 8 \times 3}{100}$

[Substituting $P$, $R$, and $T$]           ... (ii)

$SI = \dfrac{360000}{100}$

$SI = \textsf{₹} 3,600$


Final Answer:

The simple interest earned is $\textsf{₹} 3,600$.

Question 4. Calculate the accumulated amount if $\textsf{₹} 10,000$ is invested at $5\%$ per annum simple interest for 4 years.

Answer:

Given:

Principal, $P = \textsf{₹} 10,000$

Rate of Interest, $R = 5\%$ per annum

Time, $T = 4$ years


To Find:

Accumulated Amount (A)


Step 1: Calculate Simple Interest (SI)

$SI = \dfrac{P \times R \times T}{100}$

[Formula for Simple Interest]           ... (i)

$SI = \dfrac{10000 \times 5 \times 4}{100}$

[Substituting values]           ... (ii)

$SI = \dfrac{200000}{100} = \textsf{₹} 2,000$


Step 2: Calculate Accumulated Amount

$A = P + SI$

[Total amount after interest]           ... (iii)

$A = 10000 + 2000 = \textsf{₹} 12,000$


Final Answer:

The accumulated amount is $\textsf{₹} 12,000$.

Question 5. Find the compound interest on $\textsf{₹} 20,000$ for 2 years at $10\%$ per annum compounded annually.

Answer:

Given:

Principal, $P = \textsf{₹} 20,000$

Rate of Interest, $R = 10\%$ per annum

Time, $T = 2$ years


To Find:

Compound Interest (CI)


Step 1: Calculate Amount (A)

$A = P \left(1 + \dfrac{R}{100} \right)^T$

[Compound Interest Formula]           ... (i)

$A = 20000 \left(1 + \dfrac{10}{100} \right)^2$

[Substituting values]           ... (ii)

$A = 20000 \left(1.1\right)^2$

$A = 20000 \times 1.21 = \textsf{₹} 24,200$


Step 2: Calculate Compound Interest (CI)

$CI = A - P$

[Compound Interest is Amount minus Principal]           ... (iii)

$CI = 24200 - 20000 = \textsf{₹} 4,200$


Final Answer:

The compound interest is $\textsf{₹} 4,200$.

Question 6. What is the accumulated amount if $\textsf{₹} 5,000$ is invested for 3 years at $6\%$ per annum compounded annually?

Answer:

Given:

Principal, $P = \textsf{₹} 5,000$

Rate of Interest, $R = 6\%$ per annum

Time, $T = 3$ years


To Find:

Accumulated Amount (A)


Step 1: Use the compound interest formula to find the amount (A)

$A = P \left(1 + \dfrac{R}{100} \right)^T$

[Compound Interest Formula]           ... (i)

$A = 5000 \left(1 + \dfrac{6}{100} \right)^3$

[Substituting values]           ... (ii)

$A = 5000 \left(1.06\right)^3$

$A = 5000 \times 1.191016 = \textsf{₹} 5,955.08$


Final Answer:

The accumulated amount is $\textsf{₹} 5,955.08$.

Question 7. Define 'Effective Rate of Interest'. Why is it useful?

Answer:

Definition:

Effective Rate of Interest (also called Effective Annual Rate) is the actual interest earned or paid on an investment or loan in one year, taking into account the effect of compounding over the year.


Formula:

$\text{Effective Rate} = \left(1 + \dfrac{r}{n}\right)^n - 1$

[Where $r$ is nominal rate, $n$ is compounding frequency]           ... (i)


Usefulness:

The effective rate is useful because:

  • It helps in comparing different investment or loan options that have different compounding frequencies.
  • It reflects the true cost of borrowing or the actual return on investment.
  • It assists in making better financial decisions by providing a standardised interest rate.

Example:

If the nominal interest rate is $12\%$ compounded monthly, then:

$\text{Effective Rate} = \left(1 + \dfrac{0.12}{12} \right)^{12} - 1$

[Using formula]           ... (ii)

$= \left(1 + 0.01\right)^{12} - 1 = (1.01)^{12} - 1$

$= 1.126825 - 1 = 0.126825 = 12.6825\%$

So, the effective rate is 12.68%, which is greater than the nominal rate.

Question 8. Find the effective annual rate of interest equivalent to a nominal rate of $8\%$ per annum compounded quarterly.

Answer:

Given:

Nominal Annual Interest Rate, $r = 8\% = 0.08$

Compounded Quarterly $\Rightarrow$ Number of Compounding Periods, $n = 4$


To Find:

Effective Annual Rate (EAR)


Step 1: Use the formula for Effective Rate

$\text{Effective Rate} = \left(1 + \dfrac{r}{n} \right)^n - 1$

[EAR formula]           ... (i)

$= \left(1 + \dfrac{0.08}{4} \right)^4 - 1$

[Substitute values]           ... (ii)

$= \left(1 + 0.02\right)^4 - 1$

$= (1.02)^4 - 1$

$= 1.08243216 - 1 = 0.08243216$


Final Answer:

The effective annual rate is 8.2432%

Question 9. What is the nominal rate of interest if the effective rate is $10.25\%$ per annum and interest is compounded half-yearly?

Answer:

Given:

Effective Annual Rate (EAR) $= 10.25\% = 0.1025$

Compounded Half-Yearly $\Rightarrow n = 2$


To Find:

Nominal Annual Interest Rate $r$


Step 1: Use the formula for Effective Rate

$\text{Effective Rate} = \left(1 + \dfrac{r}{n} \right)^n - 1$

[EAR formula]           ... (i)


Step 2: Substitute the values

$0.1025 = \left(1 + \dfrac{r}{2} \right)^2 - 1$

[Substitute EAR and $n$]           ... (ii)

$\Rightarrow \left(1 + \dfrac{r}{2} \right)^2 = 1.1025$

Take square root on both sides:

$1 + \dfrac{r}{2} = \sqrt{1.1025} = 1.05$

$\Rightarrow \dfrac{r}{2} = 1.05 - 1 = 0.05$

$\Rightarrow r = 2 \times 0.05 = 0.10 = 10\%$


Final Answer:

The nominal rate of interest is 10% per annum.

Question 10. Define 'Present Value' (PV). Write the formula for PV of a single amount received in the future under compound interest.

Answer:

Definition:

Present Value (PV) is the current worth of a sum of money that is to be received in the future, discounted at a particular rate of compound interest.

It answers the question: “How much is a future amount worth today?”


Formula for Present Value (PV):

$PV = \dfrac{A}{(1 + r)^n}$

[Formula for Present Value]           ... (i)


Where:

$PV$ = Present Value

$A$ = Future Amount

$r$ = Rate of compound interest per period

$n$ = Number of periods


Conclusion:

The higher the interest rate or the longer the time period, the lower the present value of a future amount.

Question 11. Find the Present Value of $\textsf{₹} 12,100$ due in 2 years at $10\%$ per annum compound interest.

Answer:

Given:

Future Value ($A$) $= \textsf{₹} 12,100$

Rate of Compound Interest ($r$) $= 10\% = 0.10$

Time ($n$) $= 2$ years


To Find:

Present Value ($PV$)


Using the Present Value Formula:

$PV = \dfrac{A}{(1 + r)^n}$

[Formula for Present Value]           ... (i)


Substitute the values:

$PV = \dfrac{12,100}{(1 + 0.10)^2}$

$PV = \dfrac{12,100}{(1.10)^2}$

$PV = \dfrac{12,100}{1.21}$

$PV = \textsf{₹} 10,000$


Final Answer:

The Present Value is $\textsf{₹} 10,000$.

Question 12. Define 'Future Value' (FV). How is it related to Present Value?

Answer:

Definition:

Future Value (FV) is the amount of money that an investment made today will grow to after earning interest for a specified number of periods at a given compound interest rate.

It answers the question: "What will be the value of today’s money in the future?"


Formula for Future Value (FV):

$FV = PV \times (1 + r)^n$

[Formula for Future Value]           ... (i)


Where:

$FV$ = Future Value

$PV$ = Present Value

$r$ = Rate of compound interest per period

$n$ = Number of periods


Relation between Present Value and Future Value:

The Future Value is derived by compounding the Present Value over a number of time periods using the compound interest formula.

Alternatively, the Present Value is the discounted value of the Future Value:

$PV = \dfrac{FV}{(1 + r)^n}$

[Rearranged form of (i)]           ... (ii)


Conclusion:

Future Value and Present Value are inversely related: as time or interest rate increases, the Present Value decreases for the same Future Value, and vice versa.

Question 13. An amount of $\textsf{₹} 8,000$ is invested today. What is its Future Value after 3 years if the interest rate is $7\%$ per annum compounded annually?

Answer:

Given:

Present Value ($PV$) $= \textsf{₹} 8,000$

Rate of interest ($r$) $= 7\% = 0.07$

Time ($n$) $= 3$ years


To Find:

Future Value ($FV$)


Using the Formula:

$FV = PV \times (1 + r)^n$

[Formula for Future Value]           ... (i)


Substituting the values:

$FV = 8,000 \times (1 + 0.07)^3$

$FV = 8,000 \times (1.07)^3$

$FV = 8,000 \times 1.225043$

$FV = \textsf{₹} 9,800.34$


Final Answer:

The Future Value is $\textsf{₹} 9,800.34$.

Question 14. Define 'Annuity'. What is a 'Regular Annuity'?

Answer:

Definition of Annuity:

An annuity is a series of equal payments or receipts made at regular intervals over a specified period of time. These payments can be made weekly, monthly, quarterly, yearly, etc.


Definition of Regular Annuity:

A Regular Annuity (also known as an ordinary annuity) is a type of annuity in which the payments are made at the end of each period.

Examples include monthly deposits into a savings account, monthly rent payments, or insurance premiums.


Key Characteristics of a Regular Annuity:

  • Equal payments
  • Fixed time intervals
  • Payments occur at the end of each period

Question 15. A person deposits $\textsf{₹} 1,000$ at the end of each year for 3 years in an account paying $5\%$ per annum compound interest. Find the accumulated amount at the end of 3 years.

Answer:

Given:

Annual deposit = $\textsf{₹} 1,000$

Rate of interest ($r$) = $5\% = 0.05$

Number of years ($n$) = 3


To Find:

Accumulated amount at the end of 3 years


Using the formula for Future Value of an Ordinary Annuity:

$FV = P \times \frac{(1 + r)^n - 1}{r}$

[Future Value of ordinary annuity]           ... (i)


Substituting the values:

$FV = 1,000 \times \frac{(1 + 0.05)^3 - 1}{0.05}$

$FV = 1,000 \times \frac{1.157625 - 1}{0.05}$

$FV = 1,000 \times \frac{0.157625}{0.05}$

$FV = 1,000 \times 3.1525$

$FV = \textsf{₹} 3,152.50$


Final Answer:

The accumulated amount at the end of 3 years is $\textsf{₹} 3,152.50$.

Question 16. Define 'Tax'. What are the two main types of taxes?

Answer:

Definition of Tax:

Tax is a mandatory financial charge or levy imposed by a government on individuals or businesses to fund public expenditures and government activities. It is collected to support various services like education, healthcare, infrastructure, defense, etc.


The Two Main Types of Taxes:

1. Direct Tax:

A direct tax is a type of tax that is paid directly by an individual or organization to the government. It is imposed on income or wealth.

Examples: Income Tax, Wealth Tax, Corporate Tax.

2. Indirect Tax:

An indirect tax is a tax that is levied on goods and services rather than on income or profits. It is collected by an intermediary (like a retailer) from the person who bears the ultimate economic burden of the tax (the consumer).

Examples: Goods and Services Tax (GST), Customs Duty, Excise Duty.

Question 17. A product is sold for $\textsf{₹} 500$ excluding GST. If the GST rate is $18\%$, calculate the GST amount.

Answer:

Given:

Selling Price (excluding GST) = $\textsf{₹} 500$

GST rate = $18\%$


To Find:

GST amount


Solution:

GST is calculated as:

$\text{GST} = \text{Price} \times \frac{\text{Rate}}{100}$

[Formula for GST]           ... (i)

Substituting the values:

$\text{GST} = 500 \times \frac{18}{100}$

$\text{GST} = 500 \times 0.18$

$\text{GST} = \textsf{₹} 90$


Final Answer:

The GST amount is $\textsf{₹} 90$.

Question 18. If the price of an item including $12\%$ GST is $\textsf{₹} 560$, find the original price before GST.

Answer:

Given:

Price including GST = $\textsf{₹} 560$

GST Rate = $12\%$


To Find:

Original price before GST


Solution:

Let the original price before GST be $\textsf{₹} x$.

Then, price including GST = $x + 12\%$ of $x = x\left(1 + \frac{12}{100}\right) = x \times 1.12$

$x \times 1.12 = 560$

[Given]           ... (i)

Dividing both sides by $1.12$:

$x = \frac{560}{1.12}$

$x = \textsf{₹} 500$


Final Answer:

The original price before GST is $\textsf{₹} 500$.

Question 19. Briefly explain the concept of 'Income Tax' in India.

Answer:

Meaning of Income Tax:

Income Tax is a direct tax imposed by the Government of India on the income earned by individuals, Hindu Undivided Families (HUFs), companies, firms, LLPs, and other entities during a financial year.


Key Features:

  • It is governed by the Income Tax Act, 1961.
  • The tax is levied on total income, which includes income from salaries, house property, business or profession, capital gains, and other sources.
  • The tax rates vary based on the income slabs, residential status, and the age of the individual.
  • There are provisions for deductions (e.g., under Section 80C, 80D, etc.) to reduce taxable income.
  • Tax is collected by means such as Tax Deducted at Source (TDS), advance tax, and self-assessment tax.

Purpose:

The revenue collected through income tax is used by the government to fund infrastructure, education, health care, defense, and various social welfare schemes.


Final Note:

Every eligible taxpayer must file an Income Tax Return (ITR) annually to declare their income and taxes paid. Non-compliance can lead to penalties and legal consequences.

Question 20. An individual has a taxable income of $\textsf{₹} 6,00,000$. If the income tax rate is $10\%$ for income between $\textsf{₹} 5,00,001$ and $\textsf{₹} 10,00,000$, calculate the tax liability for this portion of income.

Answer:

Given:

Taxable income = $\textsf{₹} 6,00,000$

Tax rate for income between $\textsf{₹} 5,00,001$ and $\textsf{₹} 10,00,000$ = $10\%$


To Find:

Tax liability for the portion of income falling between $\textsf{₹} 5,00,001$ and $\textsf{₹} 10,00,000$


Solution:

Income falling under $10\%$ slab = $\textsf{₹} 6,00,000 - \textsf{₹} 5,00,000 = \textsf{₹} 1,00,000$

Tax @ $10\%$ on $\textsf{₹} 1,00,000$ = $\frac{10}{100} \times \textsf{₹} 1,00,000 = \textsf{₹} 10,000$


Final Answer:

The tax liability is $\textsf{₹} 10,000$.

Question 21. What is a 'Tariff Rate' in the context of utility bills?

Answer:

Definition:

A Tariff Rate refers to the rate or price charged by a utility service provider (such as electricity, water, or gas companies) for the consumption of services by a consumer.


Explanation:

Utility providers charge users based on how much of the service they consume. The tariff rate is generally expressed in terms of units:

  • Electricity – per kilowatt-hour (kWh)
  • Water – per kilolitre (kL) or litre
  • Gas – per cubic meter (m³) or per unit

Types of Tariff Rates:

  • Flat Rate Tariff: A single rate is charged for all units consumed.
  • Slab Rate Tariff: Different rates are applied based on the number of units consumed (e.g., first 100 units at one rate, next 200 units at a higher rate).
  • Time-of-Day Tariff: Higher rates during peak hours and lower during off-peak hours.

Importance:

Tariff rates help determine how much a consumer pays for utility services and encourage efficient usage by penalizing excessive consumption through higher rates in progressive slabs.

Question 22. Explain the difference between a 'Fixed Charge' and a 'Usage Charge' in an electricity bill.

Answer:

Fixed Charge:

This is a flat fee charged by the electricity provider regardless of the amount of electricity consumed. It is typically charged to cover costs related to meter maintenance, infrastructure, administrative expenses, and availability of power supply.

Key Features:

  • Does not vary with consumption
  • Charged even if no electricity is used
  • Ensures minimum revenue for the utility provider

Usage Charge (also called Energy Charge):

This is the variable component of the electricity bill based on the actual amount of electricity consumed by the user. It is usually charged per kilowatt-hour (kWh).

Key Features:

  • Varies with the number of units consumed
  • Encourages efficient use of electricity
  • Usually follows a slab or tier system (e.g., first 100 units at one rate, next 100 at a higher rate, etc.)

Conclusion:

The Fixed Charge is a constant fee for service availability and infrastructure, while the Usage Charge is based on actual consumption and encourages responsible usage of electricity.

Question 23. An electricity bill has a fixed charge of $\textsf{₹} 100$ and a usage charge of $\textsf{₹} 5$ per unit. If a consumer uses 150 units, calculate the total bill amount (excluding taxes/surcharges).

Answer:

Given:

Fixed charge = $\textsf{₹} 100$

Usage charge per unit = $\textsf{₹} 5$

Units consumed = 150


To Find:

Total bill amount (excluding taxes/surcharges)


Solution:

Step 1: Calculate usage charge.

Usage charge = 150 × $\textsf{₹} 5 = \textsf{₹} 750$

Step 2: Add fixed charge.

Total bill = Fixed charge + Usage charge

$\textsf{₹} 100 + \textsf{₹} 750 = \textsf{₹} 850$

…(i)


Final Answer:

Total bill amount = $\textsf{₹} 850$

Question 24. What is a 'Surcharge' or 'Service Charge' on a bill?

Answer:

Surcharge:

A surcharge is an additional charge added to the base amount of a bill. It is typically imposed by the government or the utility provider for various purposes such as fuel cost adjustment, late payment penalties, or funding infrastructure development.

Examples of Surcharges:

  • Fuel Surcharge
  • Late Payment Surcharge
  • Electricity Duty or Tax

Service Charge:

A service charge is a fee levied to cover the cost of providing a particular service. It may be fixed or based on usage. This charge is usually included in bills like restaurant bills, electricity bills, internet bills, etc.

Purpose:

  • Maintenance of infrastructure
  • Operational and administrative costs
  • To ensure quality of service

Conclusion:

Surcharge is an extra amount added due to specific reasons (like fuel cost), whereas Service Charge is a fee for the actual service provided. Both are mandatory components of a final bill amount.

Question 25. A bill amount is $\textsf{₹} 800$. A service charge of $10\%$ is added. Calculate the total amount payable.

Answer:

Given:

Bill amount = $\textsf{₹} 800$

Service charge = $10\%$ of bill amount


To Find:

Total amount payable including service charge


Solution:

Step 1: Calculate $10\%$ of $\textsf{₹} 800$

$\frac{10}{100} \times 800 = \textsf{₹} 80$

…(i)

Step 2: Add the service charge to the original bill amount

$\textsf{₹} 800 + \textsf{₹} 80 = \textsf{₹} 880$

…(ii)


Final Answer:

Total amount payable = $\textsf{₹} 880$

Question 26. Find the simple interest earned on $\textsf{₹} 25,000$ at $6\%$ per annum from 15th March 2023 to 15th August 2023.

Answer:

Given:

Principal (P) = $\textsf{₹} 25,000$

Rate of interest (R) = $6\%$ per annum

Time period = From 15th March 2023 to 15th August 2023


To Find:

Simple Interest (SI)


Solution:

Step 1: Calculate the time in years

From 15th March to 15th August = 5 months = $\frac{5}{12}$ years

Step 2: Use the formula of Simple Interest:

$SI = \frac{P \times R \times T}{100}$

…(i)

Step 3: Substitute the values into the formula:

$SI = \frac{25000 \times 6 \times \frac{5}{12}}{100}$

…(ii)

Simplifying:

$SI = \frac{25000 \times 6 \times 5}{100 \times 12} = \frac{750000}{1200} = \textsf{₹} 625$


Final Answer:

Simple Interest earned = $\textsf{₹} 625$

Question 27. If an investment doubles itself in 10 years at simple interest, find the rate of interest per annum.

Answer:

Given:

Time (T) = 10 years

The investment doubles, i.e., the amount becomes twice the principal.


To Find:

Rate of interest per annum (R)


Solution:

Let the Principal (P) = $\textsf{₹} 100$ (for ease of calculation)

Since it doubles, the Amount (A) = $\textsf{₹} 200$

Therefore, Simple Interest (SI) = A - P = $\textsf{₹} 200 - \textsf{₹} 100 = \textsf{₹} 100$

Using the formula for Simple Interest:

$SI = \frac{P \times R \times T}{100}$

…(i)

Substitute the known values:

$100 = \frac{100 \times R \times 10}{100}$

…(ii)

Simplifying:

$100 = R \times 10$

⇒ $R = \frac{100}{10} = 10\%$


Final Answer:

Rate of interest per annum = $10\%$

Question 28. An amount of $\textsf{₹} 10,000$ becomes $\textsf{₹} 12,100$ in 2 years at compound interest. Find the rate of interest per annum.

Answer:

Given:

Principal (P) = $\textsf{₹} 10,000$

Amount (A) = $\textsf{₹} 12,100$

Time (T) = 2 years


To Find:

Rate of interest per annum (R)


Solution:

Using the compound interest formula:

$A = P\left(1 + \frac{R}{100}\right)^T$

…(i)

Substitute the known values:

$12,100 = 10,000\left(1 + \frac{R}{100}\right)^2$

…(ii)

Divide both sides by 10,000:

$\left(1 + \frac{R}{100}\right)^2 = \frac{12100}{10000} = 1.21$

Take square root on both sides:

$1 + \frac{R}{100} = \sqrt{1.21} = 1.1$

Now, subtract 1 from both sides:

$\frac{R}{100} = 0.1$

Multiply both sides by 100:

$R = 10\%$


Final Answer:

Rate of interest per annum = $10\%$

Question 29. What is the effective annual rate of interest equivalent to a nominal rate of $12\%$ per annum compounded monthly?

Answer:

Given:

Nominal annual interest rate = $12\%$

Compounding frequency = Monthly = $12$ times a year


To Find:

Effective Annual Rate (EAR)


Solution:

We use the formula for Effective Annual Rate (EAR):

$EAR = \left(1 + \frac{r}{n}\right)^n - 1$

…(i)

Where:

$r = 0.12$ (nominal annual rate in decimal)

$n = 12$ (number of compounding periods per year)

Substitute the values:

$EAR = \left(1 + \frac{0.12}{12}\right)^{12} - 1$

$EAR = \left(1 + 0.01\right)^{12} - 1$

$EAR = (1.01)^{12} - 1$

$EAR \approx 1.126825 - 1$

$EAR \approx 0.126825$

Convert to percentage:

$EAR \approx 12.6825\%$


Final Answer:

Effective Annual Rate = $12.68\%$ (approximately)

Question 30. Find the Future Value of $\textsf{₹} 5,000$ invested for 4 years at $6\%$ per annum simple interest.

Answer:

Given:

Principal (P) = $\textsf{₹} 5,000$

Rate of Interest (R) = $6\%$ per annum

Time (T) = 4 years


To Find:

Future Value (Amount)


Solution:

First, we calculate Simple Interest (SI) using the formula:

$SI = \frac{P \times R \times T}{100}$

…(i)

Substitute the values:

$SI = \frac{5000 \times 6 \times 4}{100}$

$SI = \frac{120000}{100} = \textsf{₹} 1,200$

Now, Future Value = Principal + Simple Interest

$A = P + SI = 5000 + 1200$

…(ii)

$A = \textsf{₹} 6,200$


Final Answer:

Future Value = $\textsf{₹} 6,200$

Question 31. Calculate the Compound Interest on $\textsf{₹} 10,000$ for 1.5 years at $10\%$ per annum compounded half-yearly.

Answer:

Given:

Principal (P) = $\textsf{₹} 10,000$

Rate of Interest (R) = $10\%$ per annum

Time = $1.5$ years

Compounded half-yearly $\Rightarrow$ $n = 2$ times per year


To Find:

Compound Interest (CI)


Solution:

Convert time in terms of number of half-years:

$n \times t = 2 \times 1.5 = 3$

…(i)

Rate per half-year = $\frac{10}{2} = 5\%$

Use Compound Interest formula for Amount (A):

$A = P \left(1 + \frac{r}{100}\right)^n$

…(ii)

Substitute the values:

$A = 10000 \left(1 + \frac{5}{100}\right)^3$

$A = 10000 \times (1.05)^3$

$A = 10000 \times 1.157625$

$A = \textsf{₹} 11,576.25$

Now, Compound Interest (CI) = Amount - Principal

$CI = A - P = 11,576.25 - 10,000$

…(iii)

$CI = \textsf{₹} 1,576.25$


Final Answer:

Compound Interest = $\textsf{₹} 1,576.25$

Question 32. Find the Present Value of $\textsf{₹} 10,000$ due in 3 years at $8\%$ per annum simple interest.

Answer:

Given:

Future Value (A) = $\textsf{₹} 10,000$

Rate of Interest (R) = $8\%$ per annum

Time (T) = 3 years


To Find:

Present Value (P)


Solution:

We use the formula to find Present Value under simple interest:

$P = \frac{A}{1 + \frac{R \times T}{100}}$

…(i)

Substitute the values:

$P = \frac{10000}{1 + \frac{8 \times 3}{100}} = \frac{10000}{1 + \frac{24}{100}}$

$P = \frac{10000}{1.24}$

$P = \textsf{₹} 8,064.52$


Final Answer:

Present Value = $\textsf{₹} 8,064.52$

Question 33. Define 'Net Present Value' (NPV). What does a positive NPV indicate?

Answer:

Definition of Net Present Value (NPV):

The Net Present Value (NPV) is the difference between the present value of cash inflows and the present value of cash outflows over a period of time.

It is a method used in capital budgeting to evaluate the profitability of an investment or project.


Formula:

$NPV = \sum\limits_{t=1}^{n} \frac{C_t}{(1 + r)^t} - C_0$

…(i)

Where:

  • $C_t$ = Cash inflow at time $t$
  • $C_0$ = Initial investment (cash outflow at time $t = 0$)
  • $r$ = Discount rate
  • $t$ = Time period
  • $n$ = Number of periods

Interpretation of Positive NPV:

A positive NPV means that the present value of cash inflows is greater than the present value of cash outflows. This implies:

  • The investment is expected to generate more value than its cost.
  • The project or investment is profitable.
  • It should be accepted as it adds value to the firm.

Conclusion:

NPV helps in making informed financial decisions. A positive NPV indicates a financially viable project.

Question 34. A project requires an initial investment of $\textsf{₹} 10,000$ and is expected to generate a cash flow of $\textsf{₹} 12,000$ at the end of one year. Calculate the NPV if the discount rate is $10\%$ per annum.

Answer:

Given:

  • Initial Investment, $C_0 = \textsf{₹} 10,000$
  • Cash inflow after one year, $C_1 = \textsf{₹} 12,000$
  • Discount rate, $r = 10\% = 0.10$
  • Time period, $t = 1$ year

To Find:

Net Present Value (NPV)


Formula:

$NPV = \sum\limits_{t=1}^{n} \frac{C_t}{(1 + r)^t} - C_0$

…(i)


Substitute the values:

$NPV = \frac{12,000}{(1 + 0.10)^1} - 10,000$

…(ii)

$NPV = \frac{12,000}{1.10} - 10,000$

…(iii)

$NPV = 10,909.09 - 10,000$

…(iv)

$NPV = \textsf{₹} 909.09$

…(v)


Conclusion:

Since NPV is positive ($\textsf{₹} 909.09$), the project is financially viable and should be accepted.

Question 35. A person deposits $\textsf{₹} 500$ at the beginning of each year for 2 years in an account paying $6\%$ per annum compound interest. Find the accumulated amount at the end of 2 years. (This is an annuity due for 2 periods).

Answer:

Given:

  • Annual deposit = $\textsf{₹} 500$
  • Rate of interest, $r = 6\% = 0.06$ per annum
  • Number of periods, $n = 2$ years
  • Type of annuity: Annuity Due (payment at the beginning of each period)

To Find:

The accumulated amount at the end of 2 years.


Formula for Annuity Due (Future Value):

$FV = P \times \frac{(1 + r)^n - 1}{r} \times (1 + r)$

…(i)


Substitute the values:

$FV = 500 \times \frac{(1 + 0.06)^2 - 1}{0.06} \times (1 + 0.06)$

…(ii)

$FV = 500 \times \frac{(1.1236 - 1)}{0.06} \times 1.06$

…(iii)

$FV = 500 \times \frac{0.1236}{0.06} \times 1.06$

…(iv)

$FV = 500 \times 2.06 \times 1.06$

…(v)

$FV = 500 \times 2.1836$

…(vi)

$FV = \textsf{₹} 1,091.80$

…(vii)


Conclusion:

The accumulated amount at the end of 2 years is $\textsf{₹} 1,091.80$.

Question 36. What is Input Tax Credit (ITC) under GST? Explain in brief.

Answer:

Definition:

Input Tax Credit (ITC) under GST refers to the credit a registered taxpayer can claim for the tax paid on purchases (inputs) which are used in the course of business for making taxable supplies.


Explanation:

When a business purchases goods or services, it pays GST on those purchases. If these goods or services are used to make further taxable supplies, then the business can reduce its tax liability by claiming the credit of the tax paid on inputs. This mechanism ensures that the tax is collected only on the value addition at each stage of the supply chain.


Example:

Suppose a manufacturer pays $\textsf{₹} 10,000$ as GST on raw materials (input) and collects $\textsf{₹} 15,000$ as GST on the finished product (output). The manufacturer can claim ITC of $\textsf{₹} 10,000$ and needs to pay only $\textsf{₹} 5,000$ to the government.


Conditions for claiming ITC:

  • The buyer must possess a valid tax invoice.
  • The goods/services must have been received.
  • Supplier must have paid the tax to the government.
  • The buyer must have filed the GST returns.

Conclusion:

ITC is a fundamental feature of the GST regime, promoting transparency and removing the cascading effect of taxes.

Question 37. A product's price is $\textsf{₹} 800$. If it falls under the $28\%$ GST slab, what is the final price including GST?

Answer:

Given:

  • Price of the product (excluding GST) = $\textsf{₹} 800$
  • GST rate = $28\%$

To Find:

The final price of the product including GST.


Formula:

Final Price = Base Price + GST

…(i)

Calculate GST:

$GST = 28\%$ of $\textsf{₹} 800 = \frac{28}{100} \times 800 = \textsf{₹} 224$

…(ii)


Final Calculation:

Final Price = $\textsf{₹} 800 + \textsf{₹} 224$

…(iii)

Final Price = $\textsf{₹} 1,024$

…(iv)


Conclusion:

The final price of the product including $28\%$ GST is $\textsf{₹} 1,024$.

Question 38. Explain the concept of tax slabs in Income Tax calculation.

Answer:

Definition:

Tax slabs in income tax refer to the categorization of income ranges with different tax rates applied to each range. This system ensures a progressive tax structure, where individuals with higher income pay a higher percentage of tax.


Explanation:

The Income Tax Department of India has defined specific slabs for different categories of taxpayers such as individuals (below 60 years), senior citizens (60–80 years), and super senior citizens (above 80 years). Each slab specifies the income range and the corresponding tax rate.


Example (Old Regime – for individuals below 60 years as per previous structure):

Income Range Tax Rate
Up to $\textsf{₹} 2.5$ lakhNil
$\textsf{₹} 2.5$ lakh – $\textsf{₹} 5$ lakh5%
$\textsf{₹} 5$ lakh – $\textsf{₹} 10$ lakh20%
Above $\textsf{₹} 10$ lakh30%

Note: The New Tax Regime introduced optional slabs with lower rates and no exemptions/deductions.


Conclusion:

Tax slabs are a mechanism to promote equity in taxation by taxing different income levels at proportionate rates. Taxpayers must choose the regime and compute taxes accordingly.

Question 39. A consumer used 250 units of water. The tariff rate is $\textsf{₹} 15$ per unit. Calculate the water consumption charge.

Answer:

Given:

  • Water consumption = 250 units
  • Tariff rate = $\textsf{₹} 15$ per unit

To Find:

Total water consumption charge


Formula:

Total Charge = Number of units × Rate per unit

…(i)


Calculation:

Total Charge = $250 \times \textsf{₹} 15 = \textsf{₹} 3,750$

…(ii)


Conclusion:

The total water consumption charge is $\textsf{₹} 3,750$.

Question 40. An invoice amount is $\textsf{₹} 1,500$. A service charge of $5\%$ and a surcharge of $2\%$ are added. Calculate the total amount including these charges.

Answer:

Given:

  • Invoice Amount = $\textsf{₹} 1,500$
  • Service Charge = $5\%$
  • Surcharge = $2\%$

To Find:

Total amount including service charge and surcharge


Step 1: Calculate Service Charge

$5\%$ of $\textsf{₹} 1,500 = \frac{5}{100} \times 1500 = \textsf{₹} 75$

…(i)


Step 2: Calculate Surcharge

$2\%$ of $\textsf{₹} 1,500 = \frac{2}{100} \times 1500 = \textsf{₹} 30$

…(ii)


Step 3: Calculate Total Amount

Total = $\textsf{₹} 1,500 + \textsf{₹} 75 + \textsf{₹} 30 = \textsf{₹} 1,605$

…(iii)


Conclusion:

The total amount including service charge and surcharge is $\textsf{₹} 1,605$.

Question 41. Find the equivalent simple interest rate for $10\%$ per annum compound interest for 3 years.

Answer:

Given:

  • Compound Interest Rate = $10\%$ per annum
  • Time = 3 years

To Find:

Equivalent Simple Interest Rate for 3 years


Assume:

Let the Principal be $\textsf{₹} 100$ (for ease of calculation)


Step 1: Calculate Compound Amount after 3 years

Using the formula:

$A = P(1 + \frac{r}{100})^n$

…(i)

Substituting the values:

$A = 100(1 + \frac{10}{100})^3 = 100(1.1)^3 = 100 \times 1.331 = \textsf{₹} 133.10$

…(ii)


Step 2: Calculate Compound Interest

CI = $133.10 - 100 = \textsf{₹} 33.10$

…(iii)


Step 3: Find Equivalent Simple Interest Rate

Use the formula for Simple Interest:

$SI = \frac{P \times R \times T}{100}$

…(iv)

Substitute known values:

$33.10 = \frac{100 \times R \times 3}{100}$

…(v)

Simplifying:

$33.10 = 3R \Rightarrow R = \frac{33.10}{3} = 11.03\%$

…(vi)


Conclusion:

The equivalent simple interest rate for $10\%$ compound interest over 3 years is approximately $11.03\%$ per annum.

Question 42. If an amount triples itself in 8 years at compound interest, what is the approximate rate of interest?

Answer:

Given:

  • Time = 8 years
  • Amount becomes triple of the principal

To Find:

Rate of compound interest (approximate)


Assume:

Let the Principal $P = \textsf{₹} 100$ (for ease of calculation)

Then, Amount $A = 3P = \textsf{₹} 300$


Step 1: Use Compound Interest Formula

$A = P(1 + \frac{r}{100})^t$

…(i)

Substitute values:

$300 = 100(1 + \frac{r}{100})^8$

…(ii)

Divide both sides by 100:

$3 = (1 + \frac{r}{100})^8$

…(iii)


Step 2: Take 8th root on both sides

To isolate $r$, take the 8th root of 3:

$1 + \frac{r}{100} = 3^{1/8}$

…(iv)

Now calculate $3^{1/8}$:

$3^{1/8} \approx 1.147$


Step 3: Solve for r

$1 + \frac{r}{100} = 1.147 \Rightarrow \frac{r}{100} = 0.147$

…(v)

Multiply both sides by 100:

$r = 14.7\%$

…(vi)


Conclusion:

The approximate compound interest rate is $14.7\%$ per annum.

Question 43. What is the Present Value of an annuity of $\textsf{₹} 2,000$ per year for 3 years at $8\%$ per annum discount rate, paid at the end of each year?

Answer:

Given:

  • Annuity Payment (R) = $\textsf{₹} 2,000$ per year
  • Time period (n) = 3 years
  • Discount Rate (r) = $8\%$ per annum
  • Payments are made at the end of each year

To Find:

Present Value (PV) of the annuity


Formula:

Present Value of Ordinary Annuity (payments at end of each period):

$PV = R \times \frac{1 - (1 + r)^{-n}}{r}$

…(i)


Step 1: Substitute the values:

$R = 2,000$, $r = 0.08$, $n = 3$

$PV = 2000 \times \frac{1 - (1 + 0.08)^{-3}}{0.08}$

…(ii)

Simplify inside the brackets:

$PV = 2000 \times \frac{1 - (1.08)^{-3}}{0.08}$

Now calculate $(1.08)^{-3} \approx 0.7938$


Step 2: Calculate the fraction

$\frac{1 - 0.7938}{0.08} = \frac{0.2062}{0.08} \approx 2.5775$


Step 3: Multiply with annuity payment

$PV = 2000 \times 2.5775 = \textsf{₹} 5,155$

…(iii)


Conclusion:

The Present Value of the annuity is approximately $\textsf{₹} 5,155$.

Question 44. A shopkeeper sells an item for $\textsf{₹} 1,000$ after charging $18\%$ GST. How much is the GST amount?

Answer:

Given:

  • Selling Price including GST = $\textsf{₹} 1,000$
  • GST Rate = $18\%$

To Find:

The amount of GST included in the selling price


Step 1: Let the price before GST be $\textsf{₹} x$

Then the final price after adding $18\%$ GST becomes:

$x + 18\% \text{ of } x = 1000$

…(i)

That is, $x(1 + \frac{18}{100}) = 1000$

$x \times 1.18 = 1000$


Step 2: Solve for $x$

$x = \frac{1000}{1.18} \approx 847.46$


Step 3: Calculate GST

GST = Selling Price - Price before GST

$1000 - 847.46 = \textsf{₹} 152.54$

…(ii)


Conclusion:

The GST amount charged is approximately $\textsf{₹} 152.54$.

Question 45. An electricity meter reading in June was 1250 units and in July was 1420 units. How many units were consumed in July?

Answer:

Given:

  • Electricity meter reading in June = 1250 units
  • Electricity meter reading in July = 1420 units

To Find:

The number of units consumed in July


Solution:

Units consumed = July reading - June reading

$= 1420 - 1250$

…(i)

$= 170$ units


Conclusion:

The number of units consumed in July is 170 units.

Question 46. Explain the difference between nominal and effective interest rates with an example.

Answer:

Explanation:

The concepts of nominal interest rate and effective interest rate are crucial in finance as they help in understanding the true cost of borrowing or the true return on an investment, especially when interest is compounded more frequently than once a year.


1. Nominal Interest Rate:

The nominal interest rate (also known as the stated rate or annual percentage rate, APR) is the advertised or quoted interest rate on a loan or investment for a given period, typically an annual period, without taking into account the effect of compounding.

It represents the simple interest rate over the period, even if interest is calculated and added to the principal more than once within that period. For example, if a loan has a nominal interest rate of 12% per annum, and interest is compounded monthly, it means the annual rate is 12%, but 1% ($12\% / 12$ months) is applied each month.


2. Effective Interest Rate:

The effective interest rate (also known as the annual effective rate or annual equivalent rate, AER) is the true annual rate of interest paid on a loan or earned on an investment, considering the effect of compounding over the given period. It takes into account how frequently the interest is compounded within a year.

When interest is compounded more frequently (e.g., semi-annually, quarterly, monthly, or daily), the actual interest earned or paid will be higher than the nominal rate because the interest itself starts earning interest. The effective rate reflects this impact of compounding.

The formula to calculate the effective interest rate is:

$\text{Effective Rate} = \left(1 + \frac{i}{n}\right)^n - 1$

... (i)

Where:

  • $i$ = nominal annual interest rate (as a decimal)
  • $n$ = number of compounding periods per year

3. Key Differences:

Feature Nominal Interest Rate Effective Interest Rate
Definition Stated or advertised annual rate without considering compounding. True annual rate considering the effect of compounding.
Compounding Does not account for compounding frequency. Accounts for the compounding frequency.
Application Used for calculating interest per compounding period. Used for comparing different investment/loan options with varying compounding frequencies.
Value Usually lower than the effective rate when compounding is more than once a year. Usually higher than the nominal rate when compounding is more than once a year. It is equal to the nominal rate if compounded annually.

4. Example:

Suppose you have two investment options for $\textsf{₹} 10,000$:

Option A: Nominal interest rate of $5\%$ per annum, compounded annually.

Option B: Nominal interest rate of $5\%$ per annum, compounded quarterly.

Let's calculate the effective interest rate for each option:

For Option A:

$i = 0.05$ (5%)

$n = 1$ (compounded annually)

Using the formula:

$\text{Effective Rate}_A = \left(1 + \frac{0.05}{1}\right)^1 - 1$

$\text{Effective Rate}_A = (1 + 0.05) - 1$

$\text{Effective Rate}_A = 1.05 - 1$

$\text{Effective Rate}_A = 0.05 = 5\%$

In this case, the nominal rate and effective rate are the same because the interest is compounded only once a year.

For Option B:

$i = 0.05$ (5%)

$n = 4$ (compounded quarterly, as there are 4 quarters in a year)

Using the formula:

$\text{Effective Rate}_B = \left(1 + \frac{0.05}{4}\right)^4 - 1$

$\text{Effective Rate}_B = (1 + 0.0125)^4 - 1$

$\text{Effective Rate}_B = (1.0125)^4 - 1$

$\text{Effective Rate}_B \approx 1.050945 - 1$

$\text{Effective Rate}_B \approx 0.050945 = 5.0945\%$

Conclusion:

Even though both options have the same nominal interest rate of $5\%$, the effective interest rate for Option B ($5.0945\%$) is higher than that for Option A ($5\%$). This is because the interest in Option B is compounded more frequently (quarterly), allowing the interest to earn interest sooner.

Therefore, to truly compare the profitability of these two investments, one should look at the effective interest rate, which clearly shows that Option B would yield a slightly higher return over a year despite having the same nominal rate.

Question 47. Find the compound interest on $\textsf{₹} 50,000$ for 9 months at $12\%$ per annum compounded quarterly.

Answer:

Solution:

Given:

  • Principal amount (P) = $\textsf{₹} 50,000$
  • Time (T) = 9 months
  • Annual Interest Rate (R) = $12\%$ per annum
  • Compounding Frequency = Quarterly

To Find:

Compound Interest (CI)


Since the interest is compounded quarterly, we need to adjust the annual rate and the time period to match the compounding frequency.

1. Calculate the interest rate per quarter ($i$):

Annual Interest Rate = $12\%$

Number of quarters in a year = 4

$\text{Rate per quarter} (i) = \frac{\text{Annual Rate}}{\text{Number of compounding periods per year}}$

... (i)

$i = \frac{12\%}{4} = 3\%$ per quarter

In decimal form, $i = \frac{3}{100} = 0.03$

2. Calculate the number of compounding periods ($n$):

Total time = 9 months

Since there are 3 months in a quarter, the number of compounding periods is:

$\text{Number of periods} (n) = \frac{\text{Total Time in months}}{\text{Months per quarter}}$

... (ii)

$n = \frac{9 \text{ months}}{3 \text{ months/quarter}} = 3$ quarters

3. Calculate the Amount (A) using the compound interest formula:

$\text{Amount} (A) = P(1 + i)^n$

... (iii)

Substituting the values:

$A = 50000(1 + 0.03)^3$

$A = 50000(1.03)^3$

First, calculate $(1.03)^3$:

$(1.03)^2 = 1.03 \times 1.03 = 1.0609$

$(1.03)^3 = 1.0609 \times 1.03$

$\begin{array}{cc}& & 1 & . & 0 & 6 & 0 & 9 \\ \times & & 1 & . & 0 & 3 \\ \hline && 0 & 3 & 1 & 8 & 2 & 7 \\ 0 & 0 & 0 & 0 & 0 & 0 & \times \\ 1 & 0 & 6 & 0 & 9 & \times & \times \\ \hline 1 & . & 0 & 9 & 2 & 7 & 2 & 7 \\ \hline \end{array}$

So, $(1.03)^3 = 1.092727$

Now, calculate $A$:

$A = 50000 \times 1.092727$

$A = 54636.35$

4. Calculate the Compound Interest (CI):

$\text{Compound Interest} (CI) = A - P$

... (iv)

$CI = \textsf{₹} 54636.35 - \textsf{₹} 50000$

$CI = \textsf{₹} 4636.35$

The compound interest on $\textsf{₹} 50,000$ for 9 months at $12\%$ per annum compounded quarterly is $\textsf{₹} 4636.35$.

Question 48. If the effective annual rate is $5.0625\%$, what is the nominal rate compounded annually?

Answer:

Solution:

Given:

  • Effective Annual Rate = $5.0625\%$
  • Compounding Frequency = Annually

To Find:

Nominal Interest Rate


The formula for the effective interest rate is given by:

$\text{Effective Rate} = \left(1 + \frac{i}{n}\right)^n - 1$

... (i)

Where:

  • $i$ = nominal annual interest rate (as a decimal)
  • $n$ = number of compounding periods per year

In this problem:

  • Effective Rate = $5.0625\% = 0.050625$ (as a decimal)
  • Since the interest is compounded annually, the number of compounding periods per year ($n$) is 1.

Substitute these values into the formula (i):

$0.050625 = \left(1 + \frac{i}{1}\right)^1 - 1$

$0.050625 = (1 + i) - 1$

$0.050625 = 1 + i - 1$

$0.050625 = i$

To express this as a percentage, multiply by 100:

$i = 0.050625 \times 100\% = 5.0625\%$

Conclusion:

When interest is compounded annually, the nominal interest rate is equal to the effective annual rate.

Therefore, the nominal rate compounded annually is $5.0625\%$.

Question 49. What is the Present Value of $\textsf{₹} 5,000$ due in 1 year at $5\%$ per annum compounded half-yearly?

Answer:

Solution:

Given:

  • Future Value (FV) = $\textsf{₹} 5,000$
  • Time (T) = 1 year
  • Annual Interest Rate (R) = $5\%$ per annum
  • Compounding Frequency = Half-yearly

To Find:

Present Value (PV)


The formula for the Present Value when compounded is derived from the compound interest formula:

... (i)

Rearranging to find PV:

... (ii)

Where:

  • FV = Future Value
  • PV = Present Value
  • $i$ = interest rate per compounding period
  • $n$ = total number of compounding periods

1. Calculate the interest rate per compounding period ($i$):

The annual interest rate is $5\%$. Since it is compounded half-yearly, there are 2 compounding periods in a year.

$i = \frac{\text{Annual Rate}}{\text{Number of compounding periods per year}} = \frac{5\%}{2} = 2.5\%$ per half-year

As a decimal, $i = 0.025$

2. Calculate the total number of compounding periods ($n$):

The time period is 1 year. Since it's compounded half-yearly, there are 2 periods per year.

$n = \text{Time in years} \times \text{Number of compounding periods per year} = 1 \text{ year} \times 2 = 2$ periods

3. Substitute the values into the Present Value formula (ii):

$PV = \frac{5000}{(1 + 0.025)^2}$

$PV = \frac{5000}{(1.025)^2}$

Calculate $(1.025)^2$:

$\begin{array}{cc}& & 1 & . & 0 & 2 & 5 \\ \times & & 1 & . & 0 & 2 & 5 \\ \hline && & & & 5 & 1 & 2 & 5 \\ && & 2 & 0 & 5 & 0 & \times \\ && 0 & 0 & 0 & 0 & \times & \times \\ & 1 & 0 & 2 & 5 & \times & \times & \times \\ \hline 1 & . & 0 & 5 & 0 & 6 & 2 & 5 \\ \hline \end{array}$

So, $(1.025)^2 = 1.050625$

Now, calculate PV:

$PV = \frac{5000}{1.050625}$

$PV \approx 4758.8095238$

Rounding to two decimal places (for currency):

$PV \approx \textsf{₹} 4758.81$

The Present Value of $\textsf{₹} 5,000$ due in 1 year at $5\%$ per annum compounded half-yearly is approximately $\textsf{₹} 4758.81$.

Question 50. An investment of $\textsf{₹} 20,000$ is made today. What is its value after 2 years if the rate is $7\%$ p.a. simple interest?

Answer:

Problem Statement:

Calculate the value of an investment after 2 years at $7\%$ p.a. simple interest, given an initial investment of $\textsf{₹} 20,000$.


Given:

Principal (P) $= \textsf{₹} 20,000$

Rate (R) $= 7\%$ p.a.

Time (T) $= 2$ years


To Find:

The Value of investment after 2 years, which is the Amount (A).


Formulae Used:

Simple Interest (S.I.) is given by the formula:

$\text{S.I.} = \frac{P \times R \times T}{100}$

... (1)

Amount (A) is the sum of Principal and Simple Interest:

$\text{A} = P + \text{S.I.}$

... (2)


Solution:

First, calculate the Simple Interest (S.I.) using equation (1):

$\text{S.I.} = \frac{20000 \times 7 \times 2}{100}$

$\text{S.I.} = \frac{200 \times \cancel{100} \times 7 \times 2}{\cancel{100}}$

(Cancelling 100 from numerator and denominator)

$\text{S.I.} = 200 \times 7 \times 2$

$\text{S.I.} = 1400 \times 2$

$\text{S.I.} = \textsf{₹} 2800$

... (3)

Now, calculate the Amount (A) using equation (2) and the calculated Simple Interest:

$\text{A} = P + \text{S.I.}$

$\text{A} = \textsf{₹} 20000 + \textsf{₹} 2800$

$\text{A} = \textsf{₹} 22800$

(Adding Principal and Simple Interest)

Therefore, the value of the investment after 2 years is $\textsf{₹} 22,800$.

Question 51. Define an annuity due. How is its future value different from a regular annuity?

Answer:

Problem Statement:

Define an annuity due and explain how its future value differs from a regular annuity.


To Define and Explain:

Definition of Annuity Due.

Comparison of its Future Value with a Regular Annuity.


Definition:

An annuity is a series of equal payments or receipts made at regular intervals. Annuities are generally classified based on the timing of these payments.

An Annuity Due is a type of annuity where payments are made at the beginning of each period. Examples include rent payments (paid at the start of the month) or insurance premiums. Because payments are made at the beginning of the period, each payment has an additional period to earn interest compared to a regular annuity.


Difference in Future Value from a Regular Annuity:

To understand the difference, let's first briefly define a regular annuity:

A Regular Annuity (or Ordinary Annuity) is a series of equal payments made at the end of each period. Most loan payments, like car loans or mortgages, are examples of regular annuities, where payments are made after a period of using the asset.

The key difference in their future values stems directly from the timing of the payments:

1. Timing of Payments:

* In an Annuity Due, payments occur at the start of each period.

* In a Regular Annuity, payments occur at the end of each period.

2. Interest Earned:

* Since payments in an annuity due are made at the beginning of the period, each payment has one extra period to earn interest compared to a corresponding payment in a regular annuity.

* For example, if the first payment of a regular annuity is made at the end of year 1, the first payment of an annuity due is made at the beginning of year 1 (which is effectively today). This "today's" payment earns interest for the entire first period.

3. Future Value Comparison:

* Consequently, the future value of an annuity due will always be greater than the future value of a regular annuity, assuming the same payment amount, interest rate, and number of periods.

* This is because every payment in an annuity due earns interest for an additional period. The future value (FV) of an annuity due can be calculated by multiplying the future value of a regular annuity by $ (1 + r) $, where $r$ is the interest rate per period.

Let's denote:

$Pmt$ = Payment amount per period

$r$ = Interest rate per period

$n$ = Number of periods

The formula for the Future Value of a Regular Annuity ($FV_{Reg}$) is:

$FV_{Reg} = Pmt \times \left[ \frac{(1+r)^n - 1}{r} \right]$

... (i)

The formula for the Future Value of an Annuity Due ($FV_{Due}$) is:

$FV_{Due} = Pmt \times \left[ \frac{(1+r)^n - 1}{r} \right] \times (1+r)$

... (ii)

From the formulas, it is clear that:

$\text{FV}_{Due} = \text{FV}_{Reg} \times (1+r)$

(Relationship between the two future values)

This confirms that an annuity due has a higher future value because its payments earn interest for one more period.

Question 52. An individual's taxable income falls into a slab taxed at $20\%$. If the tax amount for this slab is $\textsf{₹} 50,000$, what is the income within this slab?

Answer:

Problem Statement:

Given a tax amount of $\textsf{₹} 50,000$ for an income slab taxed at $20\%$, find the income within that specific slab.


Given:

Tax Rate for the slab $= 20\%$

Tax Amount for this slab $= \textsf{₹} 50,000$


To Find:

The Income within this slab.


Formulae Used:

The relationship between Tax Amount, Tax Rate, and Income is given by:

$\text{Tax Amount} = \text{Income} \times \frac{\text{Tax Rate}}{100}$

... (1)

Rearranging the formula to find Income:

$\text{Income} = \frac{\text{Tax Amount}}{\text{Tax Rate}/100}$

... (2)

Or equivalently:

$\text{Income} = \text{Tax Amount} \times \frac{100}{\text{Tax Rate}}$

... (3)


Solution:

Let the income within this slab be $I$.

Using the formula from equation (2) or (3):

$I = \frac{\textsf{₹} 50,000}{20/100}$

$I = \textsf{₹} 50,000 \times \frac{100}{20}$

Simplify the fraction $\frac{100}{20}$:

$\frac{\cancel{100}^{5}}{\cancel{20}_{1}} = 5$

Now, substitute this value back into the equation for $I$:

$I = \textsf{₹} 50,000 \times 5$

$I = \textsf{₹} 250,000$

Thus, the income within this slab is $\textsf{₹} 250,000$.

Question 53. A water bill has a fixed charge of $\textsf{₹} 50$. The consumption charge is $\textsf{₹} 10$ per kilolitre. If 10 kilolitres were consumed, calculate the total bill (excluding other charges).

Answer:

Problem Statement:

Calculate the total water bill given a fixed charge, a per kilolitre consumption charge, and the total consumption in kilolitres.


Given:

Fixed Charge $= \textsf{₹} 50$

Consumption Charge per kilolitre $= \textsf{₹} 10$

Total Consumption $= 10$ kilolitres


To Find:

The Total Bill (excluding other charges).


Solution:

The total water bill consists of two components: the fixed charge and the consumption charge.

Step 1: Calculate the consumption charge.

The consumption charge is calculated by multiplying the consumption charge per kilolitre by the total kilolitres consumed.

$\text{Consumption Charge} = \text{Consumption} \times \text{Charge per kilolitre}$

... (i)

$\text{Consumption Charge} = 10 \text{ kilolitres} \times \textsf{₹} 10/\text{kilolitre}$

$\text{Consumption Charge} = \textsf{₹} 100$

... (ii)

Step 2: Calculate the total bill.

The total bill is the sum of the fixed charge and the consumption charge.

$\text{Total Bill} = \text{Fixed Charge} + \text{Consumption Charge}$

... (iii)

$\text{Total Bill} = \textsf{₹} 50 + \textsf{₹} 100$

$\text{Total Bill} = \textsf{₹} 150$

Therefore, the total water bill (excluding other charges) is $\textsf{₹} 150$.

Question 54. What is the concept of 'Present Value of an Annuity'? Why is it calculated?

Answer:

Problem Statement:

Define the concept of 'Present Value of an Annuity' and explain why it is calculated.


To Define and Explain:

Definition of Present Value of an Annuity.

Reasons for calculating the Present Value of an Annuity.


Concept of 'Present Value of an Annuity':

An annuity is a series of equal payments or receipts made at fixed intervals over a specified period. Examples include regular pension payments, loan repayments (like mortgages or car loans), or insurance policy payouts.

The Present Value of an Annuity (PVA) is the current worth of a series of future payments or receipts, discounted back to the present time at a specific interest (or discount) rate. In simpler terms, it answers the question: "How much money would I need to invest today, at a given interest rate, to be able to receive a series of equal payments in the future?"

This concept is based on the fundamental principle of the time value of money, which states that a rupee today is worth more than a rupee in the future due to its potential earning capacity (i.e., interest). Therefore, future cash flows must be discounted to their equivalent value today.

The formula for the Present Value of a Regular Annuity ($PVA$) is generally given by:

$PVA = Pmt \times \left[ \frac{1 - (1+r)^{-n}}{r} \right]$

... (i)

Where:

  • $Pmt$ = Payment amount per period
  • $r$ = Interest rate (or discount rate) per period
  • $n$ = Total number of periods

Why is it Calculated?

The Present Value of an Annuity is calculated for various financial and investment decisions. Its primary purpose is to allow for a fair comparison of future cash flows in today's terms. Here are some key reasons why it is calculated:

1. Loan Amortization and Valuation:

* It is used to determine the principal amount of a loan (like a mortgage or car loan) given fixed periodic payments, the interest rate, and the loan term. The loan amount is essentially the present value of all future loan payments.

* It helps in calculating how much a borrower can afford to borrow based on their ability to make regular payments.

2. Investment and Retirement Planning:

* Individuals or institutions use it to figure out how much money they need to save or invest today to provide a stream of regular income in retirement (e.g., pension plans) or for future expenses.

* It helps in evaluating investment opportunities that promise fixed periodic returns.

3. Valuation of Bonds and Other Financial Instruments:

* The price of a bond is often calculated as the present value of its future interest payments (coupon payments, which form an annuity) plus the present value of its face value (a single lump sum) at maturity.

* Similar valuation can be applied to other financial instruments that provide periodic cash flows.

4. Legal Settlements and Insurance Payouts:

* In legal cases, a lump-sum settlement might be determined by calculating the present value of a stream of future payments that would otherwise be made to the plaintiff (e.g., for lost income or medical expenses).

* Insurance companies use it to determine the lump-sum equivalent of future periodic annuity payments from life insurance or structured settlements.

5. Capital Budgeting Decisions:

* Businesses use present value concepts to evaluate potential projects or investments. When a project is expected to generate a series of constant cash inflows, the present value of these inflows helps determine the project's profitability and feasibility.

In essence, calculating the Present Value of an Annuity enables financial professionals, businesses, and individuals to make informed decisions by bringing all future cash flows back to a common point in time (the present) for accurate comparison and valuation.

Question 55. A bill has a usage charge and a fixed charge. The usage charge is $\textsf{₹} 300$ and the fixed charge is $\textsf{₹} 120$. A government surcharge of $5\%$ on the total of usage and fixed charges is applied. Calculate the total amount with the surcharge.

Answer:

Problem Statement:

Calculate the total bill including a government surcharge, given usage charge, fixed charge, and the surcharge percentage on their total.


Given:

Usage Charge $= \textsf{₹} 300$

Fixed Charge $= \textsf{₹} 120$

Government Surcharge $= 5\%$ on the total of usage and fixed charges


To Find:

The Total Amount with the surcharge.


Solution:

Step 1: Calculate the total of usage and fixed charges.

Let $Base \text{ Charge}$ be the sum of usage and fixed charges.

$\text{Base Charge} = \text{Usage Charge} + \text{Fixed Charge}$

... (i)

$\text{Base Charge} = \textsf{₹} 300 + \textsf{₹} 120$

$\text{Base Charge} = \textsf{₹} 420$

... (ii)

Step 2: Calculate the government surcharge amount.

The surcharge is $5\%$ of the Base Charge.

$\text{Surcharge Amount} = \text{Base Charge} \times \frac{\text{Surcharge Rate}}{100}$

... (iii)

$\text{Surcharge Amount} = \textsf{₹} 420 \times \frac{5}{100}$

$\text{Surcharge Amount} = \textsf{₹} 420 \times 0.05$

$\text{Surcharge Amount} = \textsf{₹} 21.00$

... (iv)

Step 3: Calculate the total amount with the surcharge.

The total amount is the sum of the Base Charge and the Surcharge Amount.

$\text{Total Amount} = \text{Base Charge} + \text{Surcharge Amount}$

... (v)

$\text{Total Amount} = \textsf{₹} 420 + \textsf{₹} 21$

$\text{Total Amount} = \textsf{₹} 441$

Therefore, the total amount with the surcharge is $\textsf{₹} 441$.



Long Answer Type Questions

Question 1. Calculate the compound interest on $\textsf{₹} 50,000$ for 3 years at $10\%$ per annum. Compare this with the simple interest for the same period and rate. What is the difference in interest earned?

Answer:

Problem Statement:

Calculate the compound interest on $\textsf{₹} 50,000$ for 3 years at $10\%$ per annum. Compare this with the simple interest for the same period and rate, and determine the difference in interest earned.


Given:

Principal (P) $= \textsf{₹} 50,000$

Time (T or n) $= 3$ years

Rate (R or i) $= 10\%$ per annum


To Find:

1. Compound Interest (C.I.)

2. Simple Interest (S.I.)

3. Difference in Interest Earned ($C.I. - S.I.$)


Formulae Used:

1. Simple Interest (S.I.):

$\text{S.I.} = \frac{P \times R \times T}{100}$

... (1)

2. Amount (A) with Compound Interest:

$\text{A} = P \left(1 + \frac{R}{100}\right)^n$

... (2)

3. Compound Interest (C.I.):

$\text{C.I.} = A - P$

... (3)


Solution:

Part 1: Calculate Simple Interest (S.I.)

Using formula (1):

$\text{S.I.} = \frac{50000 \times 10 \times 3}{100}$

$\text{S.I.} = 500 \times 10 \times 3$

$\text{S.I.} = 5000 \times 3$

$\text{S.I.} = \textsf{₹} 15,000$

... (4)

Part 2: Calculate Compound Interest (C.I.)

First, calculate the Amount (A) using formula (2):

$\text{A} = 50000 \left(1 + \frac{10}{100}\right)^3$

$\text{A} = 50000 \left(1 + 0.1\right)^3$

$\text{A} = 50000 (1.1)^3$

We need to calculate $(1.1)^3$:

$1.1 \times 1.1 = 1.21$

$1.21 \times 1.1 = 1.331$

So, $(1.1)^3 = 1.331$

Now, substitute this value back into the amount calculation:

$\text{A} = 50000 \times 1.331$

$\text{A} = \textsf{₹} 66,550$

Next, calculate the Compound Interest (C.I.) using formula (3):

$\text{C.I.} = \text{A} - P$

$\text{C.I.} = \textsf{₹} 66,550 - \textsf{₹} 50,000$

$\text{C.I.} = \textsf{₹} 16,550$

... (5)

Part 3: Calculate the Difference in Interest Earned

Difference $= \text{C.I.} - \text{S.I.}$

Difference $= \textsf{₹} 16,550 - \textsf{₹} 15,000$

$\text{Difference} = \textsf{₹} 1,550$

(Subtracting Simple Interest from Compound Interest)


Conclusion:

The Simple Interest earned is $\textsf{₹} 15,000$.

The Compound Interest earned is $\textsf{₹} 16,550$.

The difference in interest earned (Compound Interest minus Simple Interest) is $\textsf{₹} 1,550$.

Question 2. Find the accumulated amount of $\textsf{₹} 1,00,000$ invested for 5 years at $8\%$ per annum compounded quarterly.

Answer:

Problem Statement:

Calculate the accumulated amount of $\textsf{₹} 1,00,000$ invested for 5 years at $8\%$ per annum compounded quarterly.


Given:

Principal (P) $= \textsf{₹} 1,00,000$

Time (t) $= 5$ years

Annual Rate (R) $= 8\%$ per annum $= 0.08$

Compounding Frequency (m) $= \text{quarterly} = 4$ times a year


To Find:

The Accumulated Amount (A).


Formula Used:

The formula for the accumulated amount when interest is compounded $m$ times a year is:

$\text{A} = P \left(1 + \frac{R}{m}\right)^{mt}$

... (1)

Where:

  • $A$ = Accumulated Amount / Future Value
  • $P$ = Principal Amount
  • $R$ = Annual Interest Rate (as a decimal)
  • $m$ = Number of times interest is compounded per year
  • $t$ = Time in years

Solution:

First, determine the interest rate per compounding period ($\frac{R}{m}$) and the total number of compounding periods ($mt$).

Rate per period ($\frac{R}{m}$):

$\frac{R}{m} = \frac{0.08}{4} = 0.02$

Total number of periods ($mt$):

$mt = 4 \times 5 = 20$ periods

Now, substitute these values into formula (1):

$\text{A} = 1,00,000 \left(1 + 0.02\right)^{20}$

$\text{A} = 1,00,000 (1.02)^{20}$

To calculate $(1.02)^{20}$, we typically use a financial calculator or a logarithm table. For this calculation:

$(1.02)^{20} \approx 1.485947396$

Now, multiply this by the principal:

$\text{A} = 1,00,000 \times 1.485947396$

$\text{A} = \textsf{₹} 1,48,594.7396$

Rounding to two decimal places for currency:

$\text{A} \approx \textsf{₹} 1,48,594.74$

Therefore, the accumulated amount after 5 years, compounded quarterly, is $\textsf{₹} 1,48,594.74$.

Question 3. An investment promises to pay $\textsf{₹} 1,50,000$ in 5 years. If the required rate of return is $12\%$ per annum compounded annually, what is the present value of this amount?

Answer:

Problem Statement:

Calculate the present value of $\textsf{₹} 1,50,000$ to be received in 5 years, given a required rate of return of $12\%$ per annum compounded annually.


Given:

Future Value (FV) $= \textsf{₹} 1,50,000$

Number of years (n) $= 5$ years

Annual Rate of Return (r) $= 12\%$ per annum $= 0.12$

Compounding Frequency: Annually


To Find:

The Present Value (PV) of the amount.


Formula Used:

The formula for the Present Value (PV) of a single future amount, compounded annually, is:

$\text{PV} = \frac{\text{FV}}{(1 + r)^n}$

... (1)

Where:

  • $PV$ = Present Value
  • $FV$ = Future Value
  • $r$ = Annual Interest Rate (as a decimal)
  • $n$ = Number of compounding periods (years in this case)

Solution:

Substitute the given values into formula (1):

$\text{PV} = \frac{1,50,000}{(1 + 0.12)^5}$

$\text{PV} = \frac{1,50,000}{(1.12)^5}$

First, calculate $(1.12)^5$:

$(1.12)^5 \approx 1.7623416832$

Now, perform the division:

$\text{PV} = \frac{1,50,000}{1.7623416832}$

$\text{PV} \approx \textsf{₹} 85,119.576$

Rounding to two decimal places for currency:

$\text{PV} \approx \textsf{₹} 85,119.58$

Therefore, the present value of $\textsf{₹} 1,50,000$ received in 5 years, at a $12\%$ annual compound rate, is approximately $\textsf{₹} 85,119.58$.

Question 4. A person invests $\textsf{₹} 5,000$ at the end of each year for 3 years in a scheme that offers interest at $7\%$ per annum compounded annually. Calculate the accumulated value of this annuity at the end of the 3rd year.

Answer:

Problem Statement:

Calculate the accumulated value of an annuity where $\textsf{₹} 5,000$ is invested at the end of each year for 3 years, with an interest rate of $7\%$ per annum compounded annually.


Given:

Periodic Payment (Pmt) $= \textsf{₹} 5,000$ (invested at the end of each year, indicating a Regular Annuity)

Number of Periods (n) $= 3$ years

Annual Interest Rate (r) $= 7\%$ per annum $= 0.07$

Compounding Frequency: Annually


To Find:

The Accumulated Value (Future Value) of this annuity at the end of the 3rd year.


Formula Used:

The formula for the Future Value of a Regular Annuity ($FV_{Reg}$) is:

$\text{FV}_{Reg} = Pmt \times \left[ \frac{(1+r)^n - 1}{r} \right]$

... (1)

Where:

  • $FV_{Reg}$ = Future Value of the Regular Annuity
  • $Pmt$ = Payment amount per period
  • $r$ = Interest rate per period (as a decimal)
  • $n$ = Total number of periods

Solution:

Substitute the given values into formula (1):

$\text{FV}_{Reg} = 5000 \times \left[ \frac{(1+0.07)^3 - 1}{0.07} \right]$

$\text{FV}_{Reg} = 5000 \times \left[ \frac{(1.07)^3 - 1}{0.07} \right]$

First, calculate $(1.07)^3$:

$1.07 \times 1.07 = 1.1449$

$1.1449 \times 1.07 = 1.225043$

So, $(1.07)^3 = 1.225043$

Now, substitute this value back into the formula:

$\text{FV}_{Reg} = 5000 \times \left[ \frac{1.225043 - 1}{0.07} \right]$

$\text{FV}_{Reg} = 5000 \times \left[ \frac{0.225043}{0.07} \right]$

$\text{FV}_{Reg} = 5000 \times 3.2149$ (approximately)

$\text{FV}_{Reg} = \textsf{₹} 16074.5$ (approximately)

Let's use a more precise value for the fraction:

$\frac{0.225043}{0.07} \approx 3.2149$

$\text{FV}_{Reg} = 5000 \times 3.2149$

$\text{FV}_{Reg} = \textsf{₹} 16074.50$

Therefore, the accumulated value of this annuity at the end of the 3rd year is approximately $\textsf{₹} 16,074.50$.

Question 5. A company is considering a project that requires an initial investment of $\textsf{₹} 50,000$. It is expected to generate cash flows of $\textsf{₹} 20,000$ at the end of year 1, $\textsf{₹} 25,000$ at the end of year 2, and $\textsf{₹} 30,000$ at the end of year 3. If the discount rate is $10\%$ per annum, calculate the Net Present Value (NPV) of the project. Should the company accept or reject the project based on NPV?

Answer:

Problem Statement:

Calculate the Net Present Value (NPV) of a project requiring an initial investment of $\textsf{₹} 50,000$ and generating specified cash flows over 3 years, with a discount rate of $10\%$ per annum. Based on the NPV, determine whether the company should accept or reject the project.


Given:

Initial Investment ($CF_0$) $= \textsf{₹} 50,000$ (This is an outflow, so it will be treated as negative)

Cash Flow at end of Year 1 ($CF_1$) $= \textsf{₹} 20,000$

Cash Flow at end of Year 2 ($CF_2$) $= \textsf{₹} 25,000$

Cash Flow at end of Year 3 ($CF_3$) $= \textsf{₹} 30,000$

Discount Rate (r) $= 10\%$ per annum $= 0.10$


To Find:

1. The Net Present Value (NPV) of the project.

2. Whether to Accept or Reject the project based on NPV.


Formula Used:

The Net Present Value (NPV) is calculated as the sum of the present values of all future cash flows minus the initial investment. The formula for NPV is:

$\text{NPV} = CF_0 + \frac{CF_1}{(1+r)^1} + \frac{CF_2}{(1+r)^2} + \frac{CF_3}{(1+r)^3} + ... + \frac{CF_n}{(1+r)^n}$

... (1)

Where:

  • $CF_0$ = Initial Investment (at time 0, usually negative)
  • $CF_t$ = Cash flow at the end of year $t$
  • $r$ = Discount rate
  • $n$ = Number of years

Solution:

Step 1: Calculate the Present Value (PV) of each cash inflow.

PV of Cash Flow at Year 1 ($CF_1$):

$PV_1 = \frac{\textsf{₹} 20,000}{(1+0.10)^1}$

$PV_1 = \frac{\textsf{₹} 20,000}{1.10}$

$\text{PV}_1 \approx \textsf{₹} 18,181.82$

(Present Value of Year 1 Cash Flow)

PV of Cash Flow at Year 2 ($CF_2$):

$PV_2 = \frac{\textsf{₹} 25,000}{(1+0.10)^2}$

$PV_2 = \frac{\textsf{₹} 25,000}{(1.10)^2}$

$PV_2 = \frac{\textsf{₹} 25,000}{1.21}$

$\text{PV}_2 \approx \textsf{₹} 20,661.16$

(Present Value of Year 2 Cash Flow)

PV of Cash Flow at Year 3 ($CF_3$):

$PV_3 = \frac{\textsf{₹} 30,000}{(1+0.10)^3}$

$PV_3 = \frac{\textsf{₹} 30,000}{(1.10)^3}$

$PV_3 = \frac{\textsf{₹} 30,000}{1.331}$

$\text{PV}_3 \approx \textsf{₹} 22,539.44$

(Present Value of Year 3 Cash Flow)

Step 2: Calculate the Net Present Value (NPV).

$\text{NPV} = \text{Initial Investment} + \text{PV of CF}_1 + \text{PV of CF}_2 + \text{PV of CF}_3$

$\text{NPV} = -\textsf{₹} 50,000 + \textsf{₹} 18,181.82 + \textsf{₹} 20,661.16 + \textsf{₹} 22,539.44$

$\text{NPV} = -\textsf{₹} 50,000 + \textsf{₹} (18,181.82 + 20,661.16 + 22,539.44)$

$\text{NPV} = -\textsf{₹} 50,000 + \textsf{₹} 61,382.42$

$\text{NPV} = \textsf{₹} 11,382.42$


Decision Rule for NPV:

  • If NPV > 0: Accept the project. The project is expected to add value to the firm.
  • If NPV < 0: Reject the project. The project is expected to diminish firm value.
  • If NPV = 0: Be indifferent. The project is expected to just cover the cost of capital.

Conclusion:

The calculated Net Present Value (NPV) of the project is $\textsf{₹} 11,382.42$.

Since the NPV is positive ($\textsf{₹} 11,382.42 > 0$), the project is expected to generate a return higher than the required rate of $10\%$. Therefore, the company should accept the project.

Question 6. Calculate the effective annual rate of interest for the following nominal rates:

i) $6\%$ p.a. compounded monthly

ii) $7\%$ p.a. compounded half-yearly

Which rate is more beneficial for an investor?

Answer:

Problem Statement:

Calculate the effective annual rate of interest for two given nominal rates with different compounding frequencies and determine which rate is more beneficial for an investor.


Given:

i) Nominal Rate ($R_1$) $= 6\%$ p.a. compounded monthly

ii) Nominal Rate ($R_2$) $= 7\%$ p.a. compounded half-yearly


To Find:

1. The Effective Annual Rate (EAR) for both nominal rates.

2. Which rate is more beneficial for an investor.


Formula Used:

The formula for the Effective Annual Rate (EAR) is:

$\text{EAR} = \left(1 + \frac{R}{m}\right)^m - 1$

... (1)

Where:

  • $R$ = Nominal annual interest rate (as a decimal)
  • $m$ = Number of times interest is compounded per year

Solution:

Part i) Calculate EAR for $6\%$ p.a. compounded monthly

Here, $R = 0.06$ and $m = 12$ (monthly compounding).

Using formula (1):

$\text{EAR}_1 = \left(1 + \frac{0.06}{12}\right)^{12} - 1$

$\text{EAR}_1 = \left(1 + 0.005\right)^{12} - 1$

$\text{EAR}_1 = (1.005)^{12} - 1$

Calculating $(1.005)^{12} \approx 1.06167781186$

$\text{EAR}_1 = 1.06167781186 - 1$

$\text{EAR}_1 = 0.06167781186$

Converting to percentage:

$\text{EAR}_1 \approx 6.1678\%$

... (2)

Part ii) Calculate EAR for $7\%$ p.a. compounded half-yearly

Here, $R = 0.07$ and $m = 2$ (half-yearly compounding).

Using formula (1):

$\text{EAR}_2 = \left(1 + \frac{0.07}{2}\right)^2 - 1$

$\text{EAR}_2 = \left(1 + 0.035\right)^2 - 1$

$\text{EAR}_2 = (1.035)^2 - 1$

Calculating $(1.035)^2 = 1.035 \times 1.035 = 1.071225$

$\text{EAR}_2 = 1.071225 - 1$

$\text{EAR}_2 = 0.071225$

Converting to percentage:

$\text{EAR}_2 = 7.1225\%$

... (3)


Comparison and Conclusion:

Comparing the two effective annual rates:

$\text{EAR}_1 \approx 6.1678\%$

$\text{EAR}_2 = 7.1225\%$

Since $7.1225\% > 6.1678\%$, the effective annual rate of $7\%$ p.a. compounded half-yearly is higher than the effective annual rate of $6\%$ p.a. compounded monthly.

For an investor, a higher effective annual rate means that their investment will grow more over a year. Therefore, the rate of $7\%$ p.a. compounded half-yearly is more beneficial for an investor.

Question 7. Find the Present Value of an annuity of $\textsf{₹} 3,000$ per year for 3 years, with payments made at the end of each year, if the interest rate is $9\%$ per annum compounded annually.

Answer:

Problem Statement:

Calculate the Present Value of an annuity of $\textsf{₹} 3,000$ per year for 3 years, with payments made at the end of each year, if the interest rate is $9\%$ per annum compounded annually.


Given:

Periodic Payment (Pmt) $= \textsf{₹} 3,000$ (payments made at the end of each year, indicating a Regular Annuity)

Number of Periods (n) $= 3$ years

Annual Interest Rate (r) $= 9\%$ per annum $= 0.09$

Compounding Frequency: Annually


To Find:

The Present Value (PV) of this annuity.


Formula Used:

The formula for the Present Value of a Regular Annuity ($PVA_{Reg}$) is:

$\text{PVA}_{Reg} = Pmt \times \left[ \frac{1 - (1+r)^{-n}}{r} \right]$

... (1)

Where:

  • $PVA_{Reg}$ = Present Value of the Regular Annuity
  • $Pmt$ = Payment amount per period
  • $r$ = Interest rate per period (as a decimal)
  • $n$ = Total number of periods

Solution:

Substitute the given values into formula (1):

$\text{PVA}_{Reg} = 3000 \times \left[ \frac{1 - (1+0.09)^{-3}}{0.09} \right]$

$\text{PVA}_{Reg} = 3000 \times \left[ \frac{1 - (1.09)^{-3}}{0.09} \right]$

First, calculate $(1.09)^{-3}$:

$(1.09)^{-3} = \frac{1}{(1.09)^3}$

$(1.09)^3 = 1.09 \times 1.09 \times 1.09 = 1.1881 \times 1.09 = 1.295029$

So, $(1.09)^{-3} = \frac{1}{1.295029} \approx 0.772183498$

Now, substitute this value back into the formula:

$\text{PVA}_{Reg} = 3000 \times \left[ \frac{1 - 0.772183498}{0.09} \right]$

$\text{PVA}_{Reg} = 3000 \times \left[ \frac{0.227816502}{0.09} \right]$

$\text{PVA}_{Reg} = 3000 \times 2.531294466$

$\text{PVA}_{Reg} = \textsf{₹} 7593.883398$

Rounding to two decimal places for currency:

$\text{PVA}_{Reg} \approx \textsf{₹} 7,593.88$

Therefore, the present value of this annuity is approximately $\textsf{₹} 7,593.88$.

Question 8. An individual's taxable income is $\textsf{₹} 8,50,000$. The income tax rates are as follows:

Taxable IncomeRate
Up to $\textsf{₹} 2,50,000$Nil
$\textsf{₹} 2,50,001$ to $\textsf{₹} 5,00,000$$5\%$
$\textsf{₹} 5,00,001$ to $\textsf{₹} 10,00,000$$20\%$
Above $\textsf{₹} 10,00,000$$30\%$
Calculate the total income tax payable by the individual (ignore cess and surcharge for simplicity).

Answer:

Problem Statement:

Calculate the total income tax payable by an individual with a taxable income of $\textsf{₹} 8,50,000$ based on the provided progressive tax slab rates (ignoring cess and surcharge).


Given:

Taxable Income $= \textsf{₹} 8,50,000$

Income Tax Rates:

Taxable Income Rate
Up to $\textsf{₹} 2,50,000$Nil
$\textsf{₹} 2,50,001$ to $\textsf{₹} 5,00,000$$5\%$
$\textsf{₹} 5,00,001$ to $\textsf{₹} 10,00,000$$20\%$
Above $\textsf{₹} 10,00,000$$30\%$

To Find:

The Total Income Tax Payable.


Solution:

We need to calculate the tax for each slab that the individual's income falls into. The taxable income is $\textsf{₹} 8,50,000$.

Step 1: Tax on the first slab (Up to $\textsf{₹} 2,50,000$)

Taxable amount in this slab $= \textsf{₹} 2,50,000$

$\text{Tax}_1 = \textsf{₹} 2,50,000 \times 0\%$

(Nil rate)

$\text{Tax}_1 = \textsf{₹} 0$

Step 2: Tax on the second slab ($\textsf{₹} 2,50,001$ to $\textsf{₹} 5,00,000$)

This slab covers income from $\textsf{₹} 2,50,001$ up to $\textsf{₹} 5,00,000$.

Amount taxable in this slab $= \textsf{₹} 5,00,000 - \textsf{₹} 2,50,000 = \textsf{₹} 2,50,000$

$\text{Tax}_2 = \textsf{₹} 2,50,000 \times 5\%$

(Tax rate for this slab)

$\text{Tax}_2 = \textsf{₹} 2,50,000 \times \frac{5}{100}$

$\text{Tax}_2 = \textsf{₹} 12,500$

Step 3: Tax on the third slab ($\textsf{₹} 5,00,001$ to $\textsf{₹} 10,00,000$)

The individual's total income is $\textsf{₹} 8,50,000$. The income falling into this slab is the portion above $\textsf{₹} 5,00,000$ up to $\textsf{₹} 8,50,000$.

Amount taxable in this slab $= \textsf{₹} 8,50,000 - \textsf{₹} 5,00,000 = \textsf{₹} 3,50,000$

$\text{Tax}_3 = \textsf{₹} 3,50,000 \times 20\%$

(Tax rate for this slab)

$\text{Tax}_3 = \textsf{₹} 3,50,000 \times \frac{20}{100}$

$\text{Tax}_3 = \textsf{₹} 70,000$

Step 4: Tax on the fourth slab (Above $\textsf{₹} 10,00,000$)

The individual's income of $\textsf{₹} 8,50,000$ does not exceed $\textsf{₹} 10,00,000$.

Taxable amount in this slab $= \textsf{₹} 0$

$\text{Tax}_4 = \textsf{₹} 0$

Step 5: Calculate the Total Income Tax Payable

Total Tax Payable = $\text{Tax}_1 + \text{Tax}_2 + \text{Tax}_3 + \text{Tax}_4$

Total Tax Payable = $\textsf{₹} 0 + \textsf{₹} 12,500 + \textsf{₹} 70,000 + \textsf{₹} 0$

Total Tax Payable = $\textsf{₹} 82,500$


Conclusion:

The total income tax payable by the individual is $\textsf{₹} 82,500$.

Question 9. A shopkeeper buys goods worth $\textsf{₹} 1,00,000$ and pays $18\%$ GST (Input GST). He sells these goods for $\textsf{₹} 1,50,000$ and collects $18\%$ GST from the customer (Output GST). Calculate the net GST payable by the shopkeeper to the government after claiming Input Tax Credit.

Answer:

Problem Statement:

Calculate the net GST payable by a shopkeeper who buys goods for $\textsf{₹} 1,00,000$ and sells them for $\textsf{₹} 1,50,000$, with an $18\%$ GST rate applicable on both transactions.


Given:

Purchase Price of Goods $= \textsf{₹} 1,00,000$

Selling Price of Goods $= \textsf{₹} 1,50,000$

GST Rate $= 18\%$


To Find:

The Net GST Payable by the shopkeeper to the government.


Concept Used:

Input Tax Credit (ITC): This is the credit a taxpayer receives for the GST paid on purchases. When a business pays GST on inputs (goods or services bought), they can use this credit to reduce the GST they have to pay on outputs (goods or services sold) to the government.

Net GST Payable = Output GST - Input GST Credit


Solution:

Step 1: Calculate the Input GST (GST paid on purchases).

Input GST = Purchase Price $\times$ GST Rate

Input GST = $\textsf{₹} 1,00,000 \times 18\%$

Input GST = $\textsf{₹} 1,00,000 \times \frac{18}{100}$

$\text{Input GST} = \textsf{₹} 18,000$

... (i)

This $\textsf{₹} 18,000$ is the Input Tax Credit available to the shopkeeper.

Step 2: Calculate the Output GST (GST collected on sales).

Output GST = Selling Price $\times$ GST Rate

Output GST = $\textsf{₹} 1,50,000 \times 18\%$

Output GST = $\textsf{₹} 1,50,000 \times \frac{18}{100}$

$\text{Output GST} = \textsf{₹} 27,000$

... (ii)

Step 3: Calculate the Net GST Payable to the government.

Net GST Payable = Output GST - Input GST (Input Tax Credit)

Net GST Payable = $\textsf{₹} 27,000 - \textsf{₹} 18,000$

Net GST Payable = $\textsf{₹} 9,000$


Conclusion:

The net GST payable by the shopkeeper to the government after claiming Input Tax Credit is $\textsf{₹} 9,000$.

Question 10. An electricity bill shows the following details:

Fixed Charge: $\textsf{₹} 120$

Energy Charge: First 100 units at $\textsf{₹} 4$ per unit, Next 200 units at $\textsf{₹} 6$ per unit, Above 300 units at $\textsf{₹} 8$ per unit.

Surcharge: $10\%$ on total energy charge.

Other Charges: $\textsf{₹} 50$ (Service Charge, etc.)

Total units consumed: 350 units.

Calculate the total electricity bill amount.

Answer:

Problem Statement:

Calculate the total electricity bill amount based on tiered energy charges, fixed charge, a surcharge on energy charge, and other charges, for a total consumption of 350 units.


Given:

  • Fixed Charge: $\textsf{₹} 120$
  • Energy Charge Slabs:
    • First 100 units at $\textsf{₹} 4$ per unit
    • Next 200 units at $\textsf{₹} 6$ per unit
    • Above 300 units at $\textsf{₹} 8$ per unit
  • Surcharge: $10\%$ on total energy charge
  • Other Charges: $\textsf{₹} 50$ (Service Charge, etc.)
  • Total Units Consumed: 350 units

To Find:

The Total Electricity Bill Amount.


Solution:

Step 1: Calculate the Energy Charge based on units consumed in each slab.

Total units consumed are 350 units.

a) First 100 units:

Charge for first 100 units = $100 \text{ units} \times \textsf{₹} 4/\text{unit}$

$\text{Charge}_1 = \textsf{₹} 400$

... (i)

Units remaining to be charged = $350 - 100 = 250$ units.

b) Next 200 units (from the remaining 250 units):

Charge for next 200 units = $200 \text{ units} \times \textsf{₹} 6/\text{unit}$

$\text{Charge}_2 = \textsf{₹} 1,200$

... (ii)

Units remaining to be charged = $250 - 200 = 50$ units.

c) Units above 300 (the remaining 50 units):

These 50 units fall into the "Above 300 units" slab (since $100 + 200 = 300$ units have already been accounted for).

Charge for remaining 50 units = $50 \text{ units} \times \textsf{₹} 8/\text{unit}$

$\text{Charge}_3 = \textsf{₹} 400$

... (iii)

d) Total Energy Charge:

Total Energy Charge = $\text{Charge}_1 + \text{Charge}_2 + \text{Charge}_3$

Total Energy Charge = $\textsf{₹} 400 + \textsf{₹} 1,200 + \textsf{₹} 400$

$\text{Total Energy Charge} = \textsf{₹} 2,000$

... (iv)

Step 2: Calculate the Surcharge.

Surcharge is $10\%$ on the total energy charge.

Surcharge Amount = $10\% \text{ of } \textsf{₹} 2,000$

Surcharge Amount = $\frac{10}{100} \times \textsf{₹} 2,000$

Surcharge Amount = $0.10 \times \textsf{₹} 2,000$

$\text{Surcharge Amount} = \textsf{₹} 200$

... (v)

Step 3: Calculate the Total Electricity Bill.

Total Electricity Bill = Fixed Charge + Total Energy Charge + Surcharge Amount + Other Charges

Total Electricity Bill = $\textsf{₹} 120 + \textsf{₹} 2,000 + \textsf{₹} 200 + \textsf{₹} 50$

Total Electricity Bill = $\textsf{₹} 2,370$


Conclusion:

The total electricity bill amount is $\textsf{₹} 2,370$.

Question 11. An investor has two options: Option A offers $9\%$ per annum simple interest for 5 years. Option B offers $8\%$ per annum compounded annually for 5 years. If the initial investment is $\textsf{₹} 40,000$, which option yields a higher return at the end of 5 years and by how much?

Answer:

Problem Statement:

Compare two investment options: Option A with simple interest and Option B with compound interest, for an initial investment of $\textsf{₹} 40,000$ over 5 years. Determine which option yields a higher return and by what amount.


Given:

Initial Investment (Principal, P) $= \textsf{₹} 40,000$

Time (T or n) $= 5$ years

Option A: Rate (R) $= 9\%$ p.a. Simple Interest

Option B: Rate (r) $= 8\%$ p.a. Compounded Annually


To Find:

1. Accumulated Amount for Option A ($A_A$).

2. Accumulated Amount for Option B ($A_B$).

3. Which option yields a higher return.

4. The difference in returns.


Formulae Used:

1. Simple Interest (S.I.):

$\text{S.I.} = \frac{P \times R \times T}{100}$

... (1)

2. Amount (A) with Simple Interest:

$\text{A} = P + \text{S.I.}$

... (2)

3. Amount (A) with Compound Interest:

$\text{A} = P \left(1 + \frac{r}{100}\right)^n$

... (3)


Solution:

Calculation for Option A (Simple Interest):

Using formula (1) to find Simple Interest:

$\text{S.I.} = \frac{40000 \times 9 \times 5}{100}$

$\text{S.I.} = 400 \times 9 \times 5$

$\text{S.I.} = 400 \times 45$

$\text{S.I.} = \textsf{₹} 18,000$

Using formula (2) to find the Accumulated Amount ($A_A$):

$A_A = P + \text{S.I.}$

$A_A = \textsf{₹} 40,000 + \textsf{₹} 18,000$

$\text{A}_A = \textsf{₹} 58,000$

... (i)

Calculation for Option B (Compound Interest):

Using formula (3) to find the Accumulated Amount ($A_B$):

$A_B = 40000 \left(1 + \frac{8}{100}\right)^5$

$A_B = 40000 (1 + 0.08)^5$

$A_B = 40000 (1.08)^5$

Calculate $(1.08)^5$:

$(1.08)^5 \approx 1.4693280768$

Now, substitute this value back into the amount calculation:

$A_B = 40000 \times 1.4693280768$

$A_B = \textsf{₹} 58,773.123072$

Rounding to two decimal places:

$\text{A}_B \approx \textsf{₹} 58,773.12$

... (ii)


Comparison of Returns:

Amount from Option A ($A_A$) = $\textsf{₹} 58,000$

Amount from Option B ($A_B$) = $\textsf{₹} 58,773.12$

Since $\textsf{₹} 58,773.12 > \textsf{₹} 58,000$, Option B yields a higher return.

Difference in Returns:

Difference = $A_B - A_A$

Difference = $\textsf{₹} 58,773.12 - \textsf{₹} 58,000$

Difference = $\textsf{₹} 773.12$


Conclusion:

Option B (8% p.a. compounded annually) yields a higher return at the end of 5 years. The difference in interest earned, favoring Option B, is $\textsf{₹} 773.12$.

Question 12. Find the Present Value of an investment that pays $\textsf{₹} 10,000$ at the end of year 1, $\textsf{₹} 15,000$ at the end of year 2, and $\textsf{₹} 20,000$ at the end of year 3. The discount rate is $11\%$ per annum compounded annually.

Answer:

Problem Statement:

Calculate the Present Value of an investment that promises three future cash flows: $\textsf{₹} 10,000$ at the end of year 1, $\textsf{₹} 15,000$ at the end of year 2, and $\textsf{₹} 20,000$ at the end of year 3. The applicable discount rate is $11\%$ per annum compounded annually.


Given:

  • Cash Flow at end of Year 1 ($CF_1$) $= \textsf{₹} 10,000$
  • Cash Flow at end of Year 2 ($CF_2$) $= \textsf{₹} 15,000$
  • Cash Flow at end of Year 3 ($CF_3$) $= \textsf{₹} 20,000$
  • Discount Rate (r) $= 11\%$ per annum $= 0.11$
  • Compounding Frequency: Annually

To Find:

The Total Present Value (PV) of the investment.


Formula Used:

The formula for the Present Value (PV) of a single future cash flow is:

$\text{PV} = \frac{\text{FV}}{(1 + r)^n}$

... (1)

Where:

  • $PV$ = Present Value
  • $FV$ = Future Value (the cash flow at time $n$)
  • $r$ = Discount rate per period (as a decimal)
  • $n$ = Number of periods (years in this case)

To find the total present value of multiple uneven cash flows, we sum the present values of each individual cash flow.

$\text{Total PV} = \frac{CF_1}{(1+r)^1} + \frac{CF_2}{(1+r)^2} + \frac{CF_3}{(1+r)^3}$

... (2)


Solution:

Step 1: Calculate the Present Value (PV) of each individual cash flow.

a) PV of Cash Flow at Year 1 ($CF_1 = \textsf{₹} 10,000$):

$PV_1 = \frac{\textsf{₹} 10,000}{(1+0.11)^1}$

$PV_1 = \frac{\textsf{₹} 10,000}{1.11}$

$\text{PV}_1 \approx \textsf{₹} 9,009.01$

(Rounded to two decimal places)

b) PV of Cash Flow at Year 2 ($CF_2 = \textsf{₹} 15,000$):

$PV_2 = \frac{\textsf{₹} 15,000}{(1+0.11)^2}$

$PV_2 = \frac{\textsf{₹} 15,000}{(1.11)^2}$

$PV_2 = \frac{\textsf{₹} 15,000}{1.2321}$

$\text{PV}_2 \approx \textsf{₹} 12,174.34$

(Rounded to two decimal places)

c) PV of Cash Flow at Year 3 ($CF_3 = \textsf{₹} 20,000$):

$PV_3 = \frac{\textsf{₹} 20,000}{(1+0.11)^3}$

$PV_3 = \frac{\textsf{₹} 20,000}{(1.11)^3}$

$PV_3 = \frac{\textsf{₹} 20,000}{1.367631}$

$\text{PV}_3 \approx \textsf{₹} 14,624.50$

(Rounded to two decimal places)

Step 2: Sum the Present Values to get the Total Present Value.

Total PV = $PV_1 + PV_2 + PV_3$

Total PV = $\textsf{₹} 9,009.01 + \textsf{₹} 12,174.34 + \textsf{₹} 14,624.50$

Total PV = $\textsf{₹} 35,807.85$


Conclusion:

The Present Value of the investment is approximately $\textsf{₹} 35,807.85$.

Question 13. A person deposits $\textsf{₹} 2,000$ at the beginning of each year for 3 years in a recurring deposit scheme. The interest rate is $6\%$ per annum compounded annually. Calculate the total amount accumulated at the end of 3 years. (Annuity Due application).

Answer:

Problem Statement:

Calculate the total amount accumulated at the end of 3 years in a recurring deposit scheme where $\textsf{₹} 2,000$ is deposited at the beginning of each year, and the interest rate is $6\%$ per annum compounded annually.


Given:

Periodic Payment (Pmt) $= \textsf{₹} 2,000$ (deposited at the beginning of each year, indicating an Annuity Due)

Number of Periods (n) $= 3$ years

Annual Interest Rate (r) $= 6\%$ per annum $= 0.06$

Compounding Frequency: Annually


To Find:

The Total Amount Accumulated (Future Value) of this annuity at the end of 3 years.


Formula Used:

The formula for the Future Value of an Annuity Due ($FV_{Due}$) is:

$\text{FV}_{Due} = Pmt \times \left[ \frac{(1+r)^n - 1}{r} \right] \times (1+r)$

... (1)

Where:

  • $FV_{Due}$ = Future Value of the Annuity Due
  • $Pmt$ = Payment amount per period
  • $r$ = Interest rate per period (as a decimal)
  • $n$ = Total number of periods

Solution:

Substitute the given values into formula (1):

$\text{FV}_{Due} = 2000 \times \left[ \frac{(1+0.06)^3 - 1}{0.06} \right] \times (1+0.06)$

$\text{FV}_{Due} = 2000 \times \left[ \frac{(1.06)^3 - 1}{0.06} \right] \times (1.06)$

First, calculate $(1.06)^3$:

$(1.06)^2 = 1.06 \times 1.06 = 1.1236$

$(1.06)^3 = 1.1236 \times 1.06 = 1.191016$

Now, substitute this value back into the formula:

$\text{FV}_{Due} = 2000 \times \left[ \frac{1.191016 - 1}{0.06} \right] \times 1.06$

$\text{FV}_{Due} = 2000 \times \left[ \frac{0.191016}{0.06} \right] \times 1.06$

$\text{FV}_{Due} = 2000 \times 3.1836 \times 1.06$

$\text{FV}_{Due} = 2000 \times 3.374616$

$\text{FV}_{Due} = \textsf{₹} 6,749.232$

Rounding to two decimal places for currency:

$\text{FV}_{Due} \approx \textsf{₹} 6,749.23$


Conclusion:

The total amount accumulated at the end of 3 years is approximately $\textsf{₹} 6,749.23$.

Question 14. Compare the effective annual rates of interest for $10\%$ per annum compounded semi-annually and $9.8\%$ per annum compounded monthly. Which option is better for borrowing money?

Answer:

Problem Statement:

Compare the effective annual rates of interest for two nominal rates: $10\%$ per annum compounded semi-annually and $9.8\%$ per annum compounded monthly. Determine which option is better for borrowing money.


Given:

Option 1: Nominal Rate ($R_1$) $= 10\%$ p.a. $= 0.10$, Compounded Semi-annually ($m_1 = 2$)

Option 2: Nominal Rate ($R_2$) $= 9.8\%$ p.a. $= 0.098$, Compounded Monthly ($m_2 = 12$)


To Find:

1. The Effective Annual Rate (EAR) for both options.

2. Which option is better for borrowing money.


Formula Used:

The formula for the Effective Annual Rate (EAR) is:

$\text{EAR} = \left(1 + \frac{R}{m}\right)^m - 1$

... (1)

Where:

  • $R$ = Nominal annual interest rate (as a decimal)
  • $m$ = Number of times interest is compounded per year

Solution:

Part 1: Calculate EAR for Option 1 ($10\%$ p.a. compounded semi-annually)

Here, $R_1 = 0.10$ and $m_1 = 2$.

Using formula (1):

$\text{EAR}_1 = \left(1 + \frac{0.10}{2}\right)^2 - 1$

$\text{EAR}_1 = \left(1 + 0.05\right)^2 - 1$

$\text{EAR}_1 = (1.05)^2 - 1$

$\text{EAR}_1 = 1.1025 - 1$

$\text{EAR}_1 = 0.1025$

Converting to percentage:

$\text{EAR}_1 = 10.25\%$

... (i)

Part 2: Calculate EAR for Option 2 ($9.8\%$ p.a. compounded monthly)

Here, $R_2 = 0.098$ and $m_2 = 12$.

Using formula (1):

$\text{EAR}_2 = \left(1 + \frac{0.098}{12}\right)^{12} - 1$

$\text{EAR}_2 = \left(1 + 0.0081666667\right)^{12} - 1$

$\text{EAR}_2 = (1.0081666667)^{12} - 1$

Calculating $(1.0081666667)^{12} \approx 1.102568$

$\text{EAR}_2 = 1.102568 - 1$

$\text{EAR}_2 = 0.102568$

Converting to percentage:

$\text{EAR}_2 \approx 10.2568\%$

... (ii)


Comparison and Conclusion:

Comparing the two effective annual rates:

$\text{EAR}_1 = 10.25\%$

$\text{EAR}_2 \approx 10.2568\%$

For borrowing money, an individual wants to pay the least amount of interest. This means the option with the lower effective annual rate is more beneficial for a borrower.

In this case, $10.25\% < 10.2568\%$.

Therefore, Option 1 ($10\%$ p.a. compounded semi-annually) is better for borrowing money as it results in a slightly lower effective annual interest rate.

Question 15. Calculate the Present Value of the cash flows for Project B from Question 5, using a discount rate of $8\%$ per annum. Compare the NPV of Project A (from Question 5) with the NPV of Project B and recommend which project should be chosen.

Answer:

Problem Statement:

Calculate the Present Value of cash flows for Project B using a $8\%$ per annum discount rate. Then, compare its Net Present Value (NPV) with Project A (from Question 5, which had an NPV of $\textsf{₹} 11,382.42$ with a $10\%$ discount rate) and recommend which project should be chosen.


Given:

For Project B (using cash flows from Question 5):

  • Initial Investment ($CF_0$) $= \textsf{₹} 50,000$ (outflow)
  • Cash Flow at end of Year 1 ($CF_1$) $= \textsf{₹} 20,000$
  • Cash Flow at end of Year 2 ($CF_2$) $= \textsf{₹} 25,000$
  • Cash Flow at end of Year 3 ($CF_3$) $= \textsf{₹} 30,000$
  • Discount Rate (r) $= 8\%$ per annum $= 0.08$

From Question 5:

  • NPV of Project A $= \textsf{₹} 11,382.42$ (calculated with a $10\%$ discount rate for the same cash flows)

To Find:

1. The Net Present Value (NPV) of Project B.

2. Which project (Project A or Project B) should be chosen.


Formula Used:

The Net Present Value (NPV) is calculated as the sum of the present values of all future cash flows minus the initial investment. The formula for NPV is:

$\text{NPV} = CF_0 + \sum_{t=1}^{n} \frac{CF_t}{(1+r)^t}$

... (1)

For this specific project:

$\text{NPV} = - \text{Initial Investment} + \frac{CF_1}{(1+r)^1} + \frac{CF_2}{(1+r)^2} + \frac{CF_3}{(1+r)^3}$

... (2)

Where:

  • $CF_0$ = Initial Investment (negative as it's an outflow)
  • $CF_t$ = Cash flow at the end of year $t$
  • $r$ = Discount rate

Solution:

Step 1: Calculate the Present Value (PV) of each cash inflow for Project B.

a) PV of Cash Flow at Year 1 ($CF_1 = \textsf{₹} 20,000$):

$PV_1 = \frac{\textsf{₹} 20,000}{(1+0.08)^1}$

$PV_1 = \frac{\textsf{₹} 20,000}{1.08}$

$\text{PV}_1 \approx \textsf{₹} 18,518.52$

(Rounded to two decimal places)

b) PV of Cash Flow at Year 2 ($CF_2 = \textsf{₹} 25,000$):

$PV_2 = \frac{\textsf{₹} 25,000}{(1+0.08)^2}$

$PV_2 = \frac{\textsf{₹} 25,000}{(1.08)^2}$

$PV_2 = \frac{\textsf{₹} 25,000}{1.1664}$

$\text{PV}_2 \approx \textsf{₹} 21,433.47$

(Rounded to two decimal places)

c) PV of Cash Flow at Year 3 ($CF_3 = \textsf{₹} 30,000$):

$PV_3 = \frac{\textsf{₹} 30,000}{(1+0.08)^3}$

$PV_3 = \frac{\textsf{₹} 30,000}{(1.08)^3}$

$PV_3 = \frac{\textsf{₹} 30,000}{1.259712}$

$\text{PV}_3 \approx \textsf{₹} 23,814.73$

(Rounded to two decimal places)

Step 2: Calculate the Net Present Value (NPV) of Project B.

$\text{NPV}_B = - \textsf{₹} 50,000 + \textsf{₹} 18,518.52 + \textsf{₹} 21,433.47 + \textsf{₹} 23,814.73$

$\text{NPV}_B = - \textsf{₹} 50,000 + \textsf{₹} 63,766.72$

$\text{NPV}_B = \textsf{₹} 13,766.72$

(Net Present Value of Project B)


Comparison and Recommendation:

NPV of Project A (from Question 5) $= \textsf{₹} 11,382.42$

NPV of Project B $= \textsf{₹} 13,766.72$

Both projects have a positive NPV, which generally means both are acceptable. However, when choosing between mutually exclusive projects (assuming a company can only undertake one), the project with the higher positive NPV is preferred as it is expected to add more value to the firm.

Comparing the two NPVs:

$\textsf{₹} 13,766.72$ (Project B) $> \textsf{₹} 11,382.42$ (Project A)

Therefore, Project B yields a higher Net Present Value.


Conclusion:

The Net Present Value (NPV) of Project B is $\textsf{₹} 13,766.72$. Since Project B has a higher positive NPV compared to Project A ($\textsf{₹} 11,382.42$), the company should choose Project B.

Question 16. A television set is priced at $\textsf{₹} 30,000$ plus $18\%$ GST. A customer pays a down payment of $\textsf{₹} 10,000$ and decides to pay the remaining amount in 3 equal monthly instalments. Assuming $0\%$ interest for simplicity on the instalments, calculate the amount of each instalment. What is the total amount paid by the customer for the television?

Answer:

Problem Statement:

Calculate the amount of each monthly installment and the total amount paid by a customer for a television set, given its price, GST rate, a down payment, and the number of interest-free installments.


Given:

Television Set Price (before GST) $= \textsf{₹} 30,000$

GST Rate $= 18\%$

Down Payment $= \textsf{₹} 10,000$

Number of Installments $= 3$ equal monthly installments

Interest on Installments $= 0\%$


To Find:

1. The Amount of each Installment.

2. The Total Amount Paid by the customer for the television.


Solution:

Step 1: Calculate the Total Price of the Television including GST.

First, calculate the GST amount:

$\text{GST Amount} = \text{Television Set Price} \times \text{GST Rate}$

$\text{GST Amount} = \textsf{₹} 30,000 \times 18\%$

$\text{GST Amount} = \textsf{₹} 30,000 \times \frac{18}{100}$

$\text{GST Amount} = \textsf{₹} 5,400$

Now, calculate the Total Price:

$\text{Total Price} = \text{Television Set Price} + \text{GST Amount}$

$\text{Total Price} = \textsf{₹} 30,000 + \textsf{₹} 5,400$

$\text{Total Price} = \textsf{₹} 35,400$

... (i)

Step 2: Calculate the Remaining Amount to be Paid after the Down Payment.

$\text{Remaining Amount} = \text{Total Price} - \text{Down Payment}$

$\text{Remaining Amount} = \textsf{₹} 35,400 - \textsf{₹} 10,000$

$\text{Remaining Amount} = \textsf{₹} 25,400$

... (ii)

Step 3: Calculate the Amount of each Installment.

Since the installments are interest-free and equal, the remaining amount is simply divided by the number of installments.

$\text{Installment Amount} = \frac{\text{Remaining Amount}}{\text{Number of Installments}}$

$\text{Installment Amount} = \frac{\textsf{₹} 25,400}{3}$

$\text{Installment Amount} \approx \textsf{₹} 8,466.666...$

$\text{Installment Amount} \approx \textsf{₹} 8,466.67$

... (iii)

Step 4: Calculate the Total Amount Paid by the Customer.

The total amount paid is the sum of the down payment and the total of all installments.

$\text{Total Amount Paid} = \text{Down Payment} + (\text{Installment Amount} \times \text{Number of Installments})$

$\text{Total Amount Paid} = \textsf{₹} 10,000 + (\textsf{₹} 8,466.67 \times 3)$

$\text{Total Amount Paid} = \textsf{₹} 10,000 + \textsf{₹} 25,400.01$ (Due to rounding in installment calculation, this is slightly off)

Alternatively, since there's $0\%$ interest, the total amount paid should simply be the Total Price calculated in Step 1.

$\text{Total Amount Paid} = \text{Total Price (including GST)}$

$\text{Total Amount Paid} = \textsf{₹} 35,400$


Conclusion:

The amount of each monthly installment is approximately $\textsf{₹} 8,466.67$.

The total amount paid by the customer for the television is $\textsf{₹} 35,400$.

Question 17. An individual's total income for the year is $\textsf{₹} 12,00,000$. The tax slabs are given in Question 8. Calculate the total income tax payable, including a health and education cess of $4\%$ on the tax amount.

Answer:

Problem Statement:

Calculate the total income tax payable by an individual with a total income of $\textsf{₹} 12,00,000$, using the provided tax slab rates and including a $4\%$ health and education cess on the calculated tax amount.


Given:

Total Income $= \textsf{₹} 12,00,000$

Income Tax Rates:

Taxable Income Rate
Up to $\textsf{₹} 2,50,000$Nil
$\textsf{₹} 2,50,001$ to $\textsf{₹} 5,00,000$$5\%$
$\textsf{₹} 5,00,001$ to $\textsf{₹} 10,00,000$$20\%$
Above $\textsf{₹} 10,00,000$$30\%$

Health and Education Cess $= 4\%$ on the tax amount.


To Find:

The Total Income Tax Payable by the individual (including cess).


Solution:

We will calculate the tax liability for each applicable slab based on the individual's total income of $\textsf{₹} 12,00,000$.

Step 1: Calculate Tax for the First Slab (Nil Rate).

Income up to $\textsf{₹} 2,50,000$ is exempt from tax.

Taxable amount in this slab $= \textsf{₹} 2,50,000$

$\text{Tax}_1 = \textsf{₹} 2,50,000 \times 0\% = \textsf{₹} 0$

... (i)

Step 2: Calculate Tax for the Second Slab ($5\%$).

This slab covers income from $\textsf{₹} 2,50,001$ to $\textsf{₹} 5,00,000$.

Amount taxable in this slab $= \textsf{₹} 5,00,000 - \textsf{₹} 2,50,000 = \textsf{₹} 2,50,000$

$\text{Tax}_2 = \textsf{₹} 2,50,000 \times 5\%$

(Tax rate for this slab)

$\text{Tax}_2 = \textsf{₹} 2,50,000 \times \frac{5}{100}$

$\text{Tax}_2 = \textsf{₹} 12,500$

... (ii)

Step 3: Calculate Tax for the Third Slab ($20\%$).

This slab covers income from $\textsf{₹} 5,00,001$ to $\textsf{₹} 10,00,000$.

Amount taxable in this slab $= \textsf{₹} 10,00,000 - \textsf{₹} 5,00,000 = \textsf{₹} 5,00,000$

$\text{Tax}_3 = \textsf{₹} 5,00,000 \times 20\%$

(Tax rate for this slab)

$\text{Tax}_3 = \textsf{₹} 5,00,000 \times \frac{20}{100}$

$\text{Tax}_3 = \textsf{₹} 1,00,000$

... (iii)

Step 4: Calculate Tax for the Fourth Slab ($30\%$).

This slab covers income above $\textsf{₹} 10,00,000$. The individual's total income is $\textsf{₹} 12,00,000$.

Amount taxable in this slab $= \textsf{₹} 12,00,000 - \textsf{₹} 10,00,000 = \textsf{₹} 2,00,000$

$\text{Tax}_4 = \textsf{₹} 2,00,000 \times 30\%$

(Tax rate for this slab)

$\text{Tax}_4 = \textsf{₹} 2,00,000 \times \frac{30}{100}$

$\text{Tax}_4 = \textsf{₹} 60,000$

... (iv)

Step 5: Calculate the Total Income Tax (before Cess).

Total Income Tax = $\text{Tax}_1 + \text{Tax}_2 + \text{Tax}_3 + \text{Tax}_4$

Total Income Tax = $\textsf{₹} 0 + \textsf{₹} 12,500 + \textsf{₹} 1,00,000 + \textsf{₹} 60,000$

$\text{Total Income Tax} = \textsf{₹} 1,72,500$

... (v)

Step 6: Calculate Health and Education Cess.

Cess is $4\%$ on the Total Income Tax calculated in Step 5.

$\text{Cess Amount} = \textsf{₹} 1,72,500 \times 4\%$

$\text{Cess Amount} = \textsf{₹} 1,72,500 \times \frac{4}{100}$

$\text{Cess Amount} = \textsf{₹} 6,900$

... (vi)

Step 7: Calculate the Total Income Tax Payable (including Cess).

Total Tax Payable = Total Income Tax + Cess Amount

Total Tax Payable = $\textsf{₹} 1,72,500 + \textsf{₹} 6,900$

Total Tax Payable = $\textsf{₹} 1,79,400$


Conclusion:

The total income tax payable by the individual, including health and education cess, is $\textsf{₹} 1,79,400$.

Question 18. A water supply bill has the following structure:

Fixed Charge: $\textsf{₹} 80$ per month.

Consumption Charge: First 5 kilolitres at $\textsf{₹} 10$ per kL, Next 10 kL at $\textsf{₹} 18$ per kL, Above 15 kL at $\textsf{₹} 25$ per kL.

Sanitation Surcharge: $15\%$ of the consumption charge.

Meter Rent: $\textsf{₹} 30$ per month.

Total consumption in a month: 22 kilolitres.

Calculate the total water bill for the month.

Answer:

Problem Statement:

Calculate the total water bill for a month, considering fixed charges, tiered consumption charges, a sanitation surcharge on consumption, and meter rent, for a total consumption of 22 kilolitres.


Given:

  • Fixed Charge: $\textsf{₹} 80$ per month
  • Consumption Charge Slabs:
    • First 5 kilolitres (kL) at $\textsf{₹} 10$ per kL
    • Next 10 kL at $\textsf{₹} 18$ per kL
    • Above 15 kL at $\textsf{₹} 25$ per kL
  • Sanitation Surcharge: $15\%$ of the consumption charge
  • Meter Rent: $\textsf{₹} 30$ per month
  • Total Consumption in a month: 22 kilolitres

To Find:

The Total Water Bill for the month.


Solution:

Step 1: Calculate the Consumption Charge based on units consumed in each slab.

Total units consumed are 22 kilolitres.

a) First 5 kilolitres:

Charge for first 5 kL = $5 \text{ kL} \times \textsf{₹} 10/\text{kL}$

$\text{Charge}_1 = \textsf{₹} 50$

... (i)

Remaining units to be charged = $22 - 5 = 17$ kL.

b) Next 10 kilolitres (from the remaining 17 kL):

Charge for next 10 kL = $10 \text{ kL} \times \textsf{₹} 18/\text{kL}$

$\text{Charge}_2 = \textsf{₹} 180$

... (ii)

Units remaining to be charged = $17 - 10 = 7$ kL.

c) Units above 15 kL (the remaining 7 kL):

These 7 kL fall into the "Above 15 kL" slab (since $5 + 10 = 15$ kL have already been accounted for).

Charge for remaining 7 kL = $7 \text{ kL} \times \textsf{₹} 25/\text{kL}$

$\text{Charge}_3 = \textsf{₹} 175$

... (iii)

d) Total Consumption Charge:

Total Consumption Charge = $\text{Charge}_1 + \text{Charge}_2 + \text{Charge}_3$

Total Consumption Charge = $\textsf{₹} 50 + \textsf{₹} 180 + \textsf{₹} 175$

$\text{Total Consumption Charge} = \textsf{₹} 405$

... (iv)

Step 2: Calculate the Sanitation Surcharge.

Surcharge is $15\%$ of the total consumption charge.

Surcharge Amount = $15\% \text{ of } \textsf{₹} 405$

Surcharge Amount = $\frac{15}{100} \times \textsf{₹} 405$

Surcharge Amount = $0.15 \times \textsf{₹} 405$

$\text{Surcharge Amount} = \textsf{₹} 60.75$

... (v)

Step 3: Calculate the Total Water Bill.

Total Water Bill = Fixed Charge + Total Consumption Charge + Sanitation Surcharge + Meter Rent

Total Water Bill = $\textsf{₹} 80 + \textsf{₹} 405 + \textsf{₹} 60.75 + \textsf{₹} 30$

Total Water Bill = $\textsf{₹} 575.75$


Conclusion:

The total water bill for the month is $\textsf{₹} 575.75$.

Question 19. Calculate the Present Value of an investment that pays $\textsf{₹} 5,000$ at the beginning of each year for 3 years, if the discount rate is $7\%$ per annum compounded annually. (Annuity Due PV).

Answer:

Problem Statement:

Calculate the Present Value of an investment that pays $\textsf{₹} 5,000$ at the beginning of each year for 3 years, if the discount rate is $7\%$ per annum compounded annually.


Given:

Periodic Payment (Pmt) $= \textsf{₹} 5,000$ (payments made at the beginning of each year, indicating an Annuity Due)

Number of Periods (n) $= 3$ years

Annual Discount Rate (r) $= 7\%$ per annum $= 0.07$

Compounding Frequency: Annually


To Find:

The Present Value (PV) of this Annuity Due.


Formula Used:

The formula for the Present Value of an Annuity Due ($PVA_{Due}$) is:

$\text{PVA}_{Due} = Pmt \times \left[ \frac{1 - (1+r)^{-n}}{r} \right] \times (1+r)$

... (1)

Alternatively, since an annuity due's first payment occurs immediately (at time 0), its PV is the sum of the initial payment and the PV of a regular annuity for $(n-1)$ periods:

$\text{PVA}_{Due} = Pmt + Pmt \times \left[ \frac{1 - (1+r)^{-(n-1)}}{r} \right]$

... (2)

Where:

  • $PVA_{Due}$ = Present Value of the Annuity Due
  • $Pmt$ = Payment amount per period
  • $r$ = Discount rate per period (as a decimal)
  • $n$ = Total number of periods

Solution:

We will use formula (1) for this calculation:

$\text{PVA}_{Due} = 5000 \times \left[ \frac{1 - (1+0.07)^{-3}}{0.07} \right] \times (1+0.07)$

$\text{PVA}_{Due} = 5000 \times \left[ \frac{1 - (1.07)^{-3}}{0.07} \right] \times (1.07)$

First, calculate $(1.07)^{-3}$:

$(1.07)^{-3} = \frac{1}{(1.07)^3}$

$(1.07)^3 = 1.07 \times 1.07 \times 1.07 = 1.1449 \times 1.07 = 1.225043$

So, $(1.07)^{-3} = \frac{1}{1.225043} \approx 0.81629787$

Now, substitute this value back into the formula:

$\text{PVA}_{Due} = 5000 \times \left[ \frac{1 - 0.81629787}{0.07} \right] \times 1.07$

$\text{PVA}_{Due} = 5000 \times \left[ \frac{0.18370213}{0.07} \right] \times 1.07$

$\text{PVA}_{Due} = 5000 \times 2.62431614 \times 1.07$

$\text{PVA}_{Due} = 5000 \times 2.80801827$

$\text{PVA}_{Due} = \textsf{₹} 14040.09135$

Rounding to two decimal places for currency:

$\text{PVA}_{Due} \approx \textsf{₹} 14,040.09$


Conclusion:

The Present Value of this annuity due is approximately $\textsf{₹} 14,040.09$.

Question 20. An investment of $\textsf{₹} P$ is made at $r\%$ per annum compounded annually. Derive the formula for the accumulated amount after $n$ years, $A = P(1 + r)^n$. Explain each term in the formula.

Answer:

Problem Statement:

Derive the formula for the accumulated amount after $n$ years for an investment of $\textsf{₹} P$ at $r\%$ per annum compounded annually, which is $A = P(1 + r)^n$. Explain each term in the formula.


Derivation of the Formula:

Let:

  • P = Principal amount (initial investment)
  • r = Annual interest rate (as a decimal, e.g., for $10\%$, $r=0.10$)
  • n = Number of years (compounding periods)
  • A = Accumulated amount (future value) after $n$ years

When interest is compounded annually, it means that the interest earned in each year is added to the principal, and then the interest for the next year is calculated on this new, larger principal (principal + accumulated interest).

Year 1:

Interest earned in Year 1 ($I_1$) = $P \times r$

Amount at the end of Year 1 ($A_1$) = Principal + Interest in Year 1

$A_1 = P + P \times r$

$\text{A}_1 = P(1 + r)$

(Factoring out P)

Year 2:

The principal for Year 2 is the amount accumulated at the end of Year 1, i.e., $A_1 = P(1+r)$.

Interest earned in Year 2 ($I_2$) = $A_1 \times r = P(1+r) \times r$

Amount at the end of Year 2 ($A_2$) = Amount at end of Year 1 + Interest in Year 2

$A_2 = A_1 + A_1 \times r$

$A_2 = A_1 (1 + r)$

Substitute $A_1 = P(1+r)$ into the equation:

$A_2 = P(1 + r) (1 + r)$

$\text{A}_2 = P(1 + r)^2$

(Amount after 2 years)

Year 3:

The principal for Year 3 is the amount accumulated at the end of Year 2, i.e., $A_2 = P(1+r)^2$.

Interest earned in Year 3 ($I_3$) = $A_2 \times r = P(1+r)^2 \times r$

Amount at the end of Year 3 ($A_3$) = Amount at end of Year 2 + Interest in Year 3

$A_3 = A_2 + A_2 \times r$

$A_3 = A_2 (1 + r)$

Substitute $A_2 = P(1+r)^2$ into the equation:

$A_3 = P(1 + r)^2 (1 + r)$

$\text{A}_3 = P(1 + r)^3$

(Amount after 3 years)

Generalization for n Years:

Observing the pattern:

  • After 1 year, the amount is $P(1+r)^1$
  • After 2 years, the amount is $P(1+r)^2$
  • After 3 years, the amount is $P(1+r)^3$

We can generalize this pattern. For $n$ years, the accumulated amount (A) will be:

$\textbf{A} = \textbf{P}(\textbf{1} + \textbf{r})^\textbf{n}$

... (Final Formula)

This formula applies when the interest is compounded annually. If compounded $m$ times a year, the rate becomes $r/m$ and the periods become $n \times m$, leading to $A = P(1 + r/m)^{nm}$.


Explanation of Each Term in the Formula $A = P(1 + r)^n$:

1. A (Accumulated Amount / Future Value):

  • This is the final value of the investment at the end of the specified period ($n$ years).
  • It represents the initial principal plus all the compound interest earned over the investment term. This is what the investor will receive back or what the loan will amount to.

2. P (Principal / Initial Investment):

  • This is the original sum of money invested or borrowed.
  • It is the starting amount upon which interest is calculated.

3. r (Annual Interest Rate):

  • This is the nominal annual rate of interest.
  • It must be expressed as a decimal in the formula (e.g., if the rate is $10\%$, then $r = 0.10$).
  • This rate is applied to the principal (and accumulated interest) each year.

4. n (Number of Years):

  • This represents the total number of compounding periods. Since the interest is compounded annually in this derivation, $n$ directly corresponds to the number of years the money is invested or borrowed.
  • The exponent $n$ signifies that the factor $(1+r)$ is multiplied by itself $n$ times, reflecting the compounding effect of interest over the years.

5. (1 + r):

  • This term is the growth factor per period.
  • It signifies that for every $\textsf{₹} 1$ invested, it grows to $\textsf{₹} (1+r)$ by the end of one year (or one compounding period). For example, if $r=0.10$, $\textsf{₹} 1$ becomes $\textsf{₹} 1.10$.

Question 21. A company produces two items, A and B. Item A costs $\textsf{₹} 1,000$ (exclusive of tax) and falls under $12\%$ GST. Item B costs $\textsf{₹} 1,500$ (exclusive of tax) and falls under $18\%$ GST. A customer buys 5 units of Item A and 3 units of Item B. Calculate the total bill amount including GST.

Answer:

Problem Statement:

Calculate the total bill amount for a customer who buys 5 units of Item A (costing $\textsf{₹} 1,000$ each with $12\%$ GST) and 3 units of Item B (costing $\textsf{₹} 1,500$ each with $18\%$ GST).


Given:

  • Item A:
    • Unit Cost (exclusive of tax) $= \textsf{₹} 1,000$
    • GST Rate $= 12\%$
    • Quantity Purchased $= 5$ units
  • Item B:
    • Unit Cost (exclusive of tax) $= \textsf{₹} 1,500$
    • GST Rate $= 18\%$
    • Quantity Purchased $= 3$ units

To Find:

The Total Bill Amount including GST.


Solution:

Part 1: Calculate the Total Cost for Item A (including GST).

a) Cost of 5 units of Item A (exclusive of tax):

Cost of Item A (base) $= \text{Unit Cost} \times \text{Quantity}$

Cost of Item A (base) $= \textsf{₹} 1,000 \times 5 = \textsf{₹} 5,000$

b) GST on 5 units of Item A:

GST_A $= \text{Cost of Item A (base)} \times \text{GST Rate}_A$

GST_A $= \textsf{₹} 5,000 \times 12\%$

GST_A $= \textsf{₹} 5,000 \times \frac{12}{100} = \textsf{₹} 600$

c) Total Cost of 5 units of Item A (including GST):

Total Cost_A $= \text{Cost of Item A (base)} + \text{GST}_A$

Total Cost_A $= \textsf{₹} 5,000 + \textsf{₹} 600$

$\text{Total Cost}_A = \textsf{₹} 5,600$

... (i)

Part 2: Calculate the Total Cost for Item B (including GST).

a) Cost of 3 units of Item B (exclusive of tax):

Cost of Item B (base) $= \text{Unit Cost} \times \text{Quantity}$

Cost of Item B (base) $= \textsf{₹} 1,500 \times 3 = \textsf{₹} 4,500$

b) GST on 3 units of Item B:

GST_B $= \text{Cost of Item B (base)} \times \text{GST Rate}_B$

GST_B $= \textsf{₹} 4,500 \times 18\%$

GST_B $= \textsf{₹} 4,500 \times \frac{18}{100} = \textsf{₹} 810$

c) Total Cost of 3 units of Item B (including GST):

Total Cost_B $= \text{Cost of Item B (base)} + \text{GST}_B$

Total Cost_B $= \textsf{₹} 4,500 + \textsf{₹} 810$

$\text{Total Cost}_B = \textsf{₹} 5,310$

... (ii)

Part 3: Calculate the Total Bill Amount.

Total Bill Amount = Total Cost of Item A + Total Cost of Item B

Total Bill Amount = $\textsf{₹} 5,600 + \textsf{₹} 5,310$

Total Bill Amount = $\textsf{₹} 10,910$


Conclusion:

The total bill amount including GST is $\textsf{₹} 10,910$.

Question 22. A person takes a loan of $\textsf{₹} 30,000$ from a friend at a simple interest rate of $10\%$ per annum. They agree to repay $\textsf{₹} 10,000$ at the end of the first year, $\textsf{₹} 10,000$ at the end of the second year, and the remaining amount at the end of the third year. Calculate the amount paid at the end of the third year and the total interest paid. (Simple interest calculation with partial payments).

Answer:

Problem Statement:

Calculate the amount paid at the end of the third year and the total interest paid for a loan of $\textsf{₹} 30,000$ at $10\%$ per annum simple interest, with partial repayments of $\textsf{₹} 10,000$ at the end of the first and second years.


Given:

Initial Loan Amount (Principal, P) $= \textsf{₹} 30,000$

Simple Interest Rate (R) $= 10\%$ per annum

Repayment at End of Year 1 $= \textsf{₹} 10,000$

Repayment at End of Year 2 $= \textsf{₹} 10,000$

Repayment at End of Year 3 $= \text{Remaining amount}$


To Find:

1. The Amount Paid at the End of the Third Year.

2. The Total Interest Paid.


Concept Used:

For simple interest with partial payments, the interest for each period is typically calculated on the outstanding principal at the beginning of that period. Payments reduce the principal, thereby reducing the base for future interest calculations.


Solution:

We will track the outstanding principal and interest year by year.

Year 1:

  • Principal at beginning of Year 1 ($P_0$) = $\textsf{₹} 30,000$
  • Interest for Year 1 ($I_1$) = $P_0 \times R \times 1 \text{ year}$
  • $\text{I}_1 = \textsf{₹} 30,000 \times 0.10 = \textsf{₹} 3,000$

    ... (i)

  • Amount Due at end of Year 1 = $P_0 + I_1 = \textsf{₹} 30,000 + \textsf{₹} 3,000 = \textsf{₹} 33,000$
  • Payment Made at end of Year 1 = $\textsf{₹} 10,000$
  • Outstanding Principal at end of Year 1 ($P_1$) = $\textsf{₹} 33,000 - \textsf{₹} 10,000 = \textsf{₹} 23,000$

Year 2:

  • Principal at beginning of Year 2 ($P_1$) = $\textsf{₹} 23,000$
  • Interest for Year 2 ($I_2$) = $P_1 \times R \times 1 \text{ year}$
  • $\text{I}_2 = \textsf{₹} 23,000 \times 0.10 = \textsf{₹} 2,300$

    ... (ii)

  • Amount Due at end of Year 2 = $P_1 + I_2 = \textsf{₹} 23,000 + \textsf{₹} 2,300 = \textsf{₹} 25,300$
  • Payment Made at end of Year 2 = $\textsf{₹} 10,000$
  • Outstanding Principal at end of Year 2 ($P_2$) = $\textsf{₹} 25,300 - \textsf{₹} 10,000 = \textsf{₹} 15,300$

Year 3:

  • Principal at beginning of Year 3 ($P_2$) = $\textsf{₹} 15,300$
  • Interest for Year 3 ($I_3$) = $P_2 \times R \times 1 \text{ year}$
  • $\text{I}_3 = \textsf{₹} 15,300 \times 0.10 = \textsf{₹} 1,530$

    ... (iii)

  • Amount Paid at end of Year 3 (Remaining Amount) = $P_2 + I_3$
  • Amount Paid at end of Year 3 = $\textsf{₹} 15,300 + \textsf{₹} 1,530 = \textsf{₹} 16,830$

Total Interest Paid:

Total Interest Paid = Interest for Year 1 + Interest for Year 2 + Interest for Year 3

Total Interest Paid = $\textsf{₹} 3,000 + \textsf{₹} 2,300 + \textsf{₹} 1,530$

Total Interest Paid = $\textsf{₹} 6,830$


Conclusion:

The amount paid at the end of the third year is $\textsf{₹} 16,830$.

The total interest paid is $\textsf{₹} 6,830$.

Question 23. Calculate the compound interest on $\textsf{₹} 60,000$ for 2 years at $9\%$ per annum, compounded half-yearly.

Answer:

Problem Statement:

Calculate the compound interest on $\textsf{₹} 60,000$ for 2 years at $9\%$ per annum, compounded half-yearly.


Given:

Principal (P) $= \textsf{₹} 60,000$

Time (t) $= 2$ years

Annual Rate (R) $= 9\%$ per annum $= 0.09$

Compounding Frequency (m) $= \text{half-yearly} = 2$ times a year


To Find:

The Compound Interest (C.I.).


Formulae Used:

1. Amount (A) with Compound Interest:

$\text{A} = P \left(1 + \frac{R}{m}\right)^{mt}$

... (1)

2. Compound Interest (C.I.):

$\text{C.I.} = A - P$

... (2)

Where:

  • $P$ = Principal Amount
  • $R$ = Annual Interest Rate (as a decimal)
  • $m$ = Number of times interest is compounded per year
  • $t$ = Time in years

Solution:

Step 1: Determine the rate per period and total number of periods.

Rate per compounding period ($\frac{R}{m}$) = $\frac{0.09}{2} = 0.045$

Total number of compounding periods ($mt$) = $2 \times 2 = 4$ periods

Step 2: Calculate the Accumulated Amount (A) using formula (1).

$\text{A} = 60000 \left(1 + 0.045\right)^{4}$

$\text{A} = 60000 (1.045)^4$

Calculate $(1.045)^4$:

$(1.045)^2 = 1.045 \times 1.045 = 1.092025$

$(1.045)^4 = (1.092025)^2 = 1.092025 \times 1.092025 \approx 1.1925186$

Substitute this value back into the amount calculation:

$\text{A} = 60000 \times 1.1925186$

$\text{A} = \textsf{₹} 71,551.116$

Rounding to two decimal places for currency:

$\text{A} \approx \textsf{₹} 71,551.12$

... (i)

Step 3: Calculate the Compound Interest (C.I.) using formula (2).

$\text{C.I.} = \text{A} - P$

$\text{C.I.} = \textsf{₹} 71,551.12 - \textsf{₹} 60,000$

$\text{C.I.} = \textsf{₹} 11,551.12$


Conclusion:

The compound interest on $\textsf{₹} 60,000$ for 2 years at $9\%$ per annum, compounded half-yearly, is $\textsf{₹} 11,551.12$.

Question 24. Find the accumulated value of an annuity of $\textsf{₹} 1,500$ paid at the end of each quarter for 1 year, if the interest rate is $8\%$ per annum compounded quarterly.

Answer:

Problem Statement:

Calculate the accumulated value of an annuity of $\textsf{₹} 1,500$ paid at the end of each quarter for 1 year, if the interest rate is $8\%$ per annum compounded quarterly.


Given:

Periodic Payment (Pmt) $= \textsf{₹} 1,500$ (paid at the end of each quarter, indicating a Regular Annuity)

Time (t) $= 1$ year

Annual Interest Rate (R) $= 8\%$ per annum $= 0.08$

Compounding Frequency (m) $= \text{quarterly} = 4$ times a year


To Find:

The Accumulated Value (Future Value) of this annuity.


Formula Used:

The formula for the Future Value of a Regular Annuity ($FV_{Reg}$) is:

$\text{FV}_{Reg} = Pmt \times \left[ \frac{(1+r')^n - 1}{r'} \right]$

... (1)

Where:

  • $Pmt$ = Payment amount per period
  • $r'$ = Interest rate per period (calculated as $R/m$)
  • $n$ = Total number of periods (calculated as $t \times m$)

Solution:

Step 1: Determine the interest rate per period and total number of periods.

Interest rate per period ($r'$):

$r' = \frac{R}{m} = \frac{0.08}{4} = 0.02$

Total number of periods ($n$):

$n = t \times m = 1 \text{ year} \times 4 \text{ quarters/year} = 4 \text{ periods}$

Step 2: Calculate the Future Value of the Regular Annuity using formula (1).

$\text{FV}_{Reg} = 1500 \times \left[ \frac{(1+0.02)^4 - 1}{0.02} \right]$

$\text{FV}_{Reg} = 1500 \times \left[ \frac{(1.02)^4 - 1}{0.02} \right]$

First, calculate $(1.02)^4$:

$(1.02)^2 = 1.02 \times 1.02 = 1.0404$

$(1.02)^4 = (1.0404)^2 = 1.0404 \times 1.0404 = 1.08243216$

Now, substitute this value back into the formula:

$\text{FV}_{Reg} = 1500 \times \left[ \frac{1.08243216 - 1}{0.02} \right]$

$\text{FV}_{Reg} = 1500 \times \left[ \frac{0.08243216}{0.02} \right]$

$\text{FV}_{Reg} = 1500 \times 4.121608$

$\text{FV}_{Reg} = \textsf{₹} 6182.412$

Rounding to two decimal places for currency:

$\text{FV}_{Reg} \approx \textsf{₹} 6,182.41$


Conclusion:

The accumulated value of the annuity at the end of 1 year is approximately $\textsf{₹} 6,182.41$.

Question 25. The electricity bill tariff has a fixed charge of $\textsf{₹} 150$. Energy charges are $\textsf{₹} 5$ per unit for the first 200 units and $\textsf{₹} 7$ per unit for consumption above 200 units. A consumer used 280 units. A regulatory charge of $5\%$ on the total energy charge is added. Calculate the total bill amount.

Answer:

Problem Statement:

Calculate the total electricity bill amount based on a fixed charge, tiered energy charges, a regulatory surcharge on energy charges, for a total consumption of 280 units.


Given:

  • Fixed Charge: $\textsf{₹} 150$
  • Energy Charge Slabs:
    • First 200 units at $\textsf{₹} 5$ per unit
    • Above 200 units at $\textsf{₹} 7$ per unit
  • Total Units Consumed: 280 units
  • Regulatory Charge: $5\%$ on total energy charge

To Find:

The Total Electricity Bill Amount.


Solution:

Step 1: Calculate the Energy Charge based on units consumed in each slab.

Total units consumed are 280 units.

a) First 200 units:

Charge for first 200 units = $200 \text{ units} \times \textsf{₹} 5/\text{unit}$

$\text{Charge}_1 = \textsf{₹} 1,000$

... (i)

Units remaining to be charged = $280 - 200 = 80$ units.

b) Remaining 80 units (above 200 units):

Charge for remaining 80 units = $80 \text{ units} \times \textsf{₹} 7/\text{unit}$

$\text{Charge}_2 = \textsf{₹} 560$

... (ii)

c) Total Energy Charge:

Total Energy Charge = $\text{Charge}_1 + \text{Charge}_2$

Total Energy Charge = $\textsf{₹} 1,000 + \textsf{₹} 560$

$\text{Total Energy Charge} = \textsf{₹} 1,560$

... (iii)

Step 2: Calculate the Regulatory Charge.

Regulatory Charge is $5\%$ on the total energy charge.

Regulatory Charge Amount = $5\% \text{ of } \textsf{₹} 1,560$

Regulatory Charge Amount = $\frac{5}{100} \times \textsf{₹} 1,560$

Regulatory Charge Amount = $0.05 \times \textsf{₹} 1,560$

$\text{Regulatory Charge Amount} = \textsf{₹} 78$

... (iv)

Step 3: Calculate the Total Electricity Bill.

Total Electricity Bill = Fixed Charge + Total Energy Charge + Regulatory Charge Amount

Total Electricity Bill = $\textsf{₹} 150 + \textsf{₹} 1,560 + \textsf{₹} 78$

Total Electricity Bill = $\textsf{₹} 1,788$


Conclusion:

The total electricity bill amount is $\textsf{₹} 1,788$.

Question 26. Explain the concept of Net Present Value (NPV) as a criterion for investment decisions. Discuss its advantages and disadvantages compared to other methods (like payback period - if introduced, else just keep general).

Answer:

Problem Statement:

Explain the concept of Net Present Value (NPV) as a criterion for investment decisions. Discuss its advantages and disadvantages compared to other methods.


Concept of Net Present Value (NPV):

The Net Present Value (NPV) is a capital budgeting technique used to determine the profitability of a project or investment. It measures the difference between the present value of future cash inflows and the present value of the initial investment (cash outflows) over a specified period.

The core idea behind NPV is the time value of money, which states that a rupee received today is worth more than a rupee received in the future due to its potential earning capacity. Therefore, all future cash flows (both inflows and outflows) associated with a project are discounted back to their present value using a specified discount rate (often the cost of capital or required rate of return).

The general formula for NPV is:

$\text{NPV} = \sum_{t=0}^{n} \frac{CF_t}{(1+r)^t}$

... (i)

Where:

  • $CF_t$ = Net cash flow at time $t$. ($CF_0$ is usually the initial investment, which is a cash outflow and thus a negative value).
  • $r$ = Discount rate (cost of capital or required rate of return).
  • $t$ = Time period (ranging from 0 to $n$).
  • $n$ = Total number of periods.

If the initial investment is $I_0$ (at time $t=0$) and subsequent cash inflows are $CF_1, CF_2, ..., CF_n$:

$\text{NPV} = -I_0 + \frac{CF_1}{(1+r)^1} + \frac{CF_2}{(1+r)^2} + ... + \frac{CF_n}{(1+r)^n}$

... (ii)

NPV Decision Rule:

  • If NPV > 0: Accept the project. The project is expected to generate a return greater than the cost of capital, thereby adding value to the firm and increasing shareholder wealth.
  • If NPV < 0: Reject the project. The project is expected to generate a return less than the cost of capital, leading to a decrease in firm value.
  • If NPV = 0: Indifferent. The project is expected to generate a return exactly equal to the cost of capital, neither adding nor detracting from firm value. In practice, such projects are often rejected due to the inherent risks and uncertainties not captured in the calculation.

For mutually exclusive projects (where only one can be chosen), the project with the highest positive NPV is typically selected.


Advantages of NPV:

1. Considers Time Value of Money: This is its fundamental strength. NPV explicitly discounts future cash flows back to their present value, making it a theoretically sound method as it accounts for the opportunity cost of capital.

2. Considers All Cash Flows: Unlike some other methods (e.g., payback period), NPV takes into account all cash flows generated by the project over its entire life, providing a comprehensive view of profitability.

3. Provides a Direct Measure of Value Added: A positive NPV indicates the actual increase in the wealth of the shareholders (or the value added to the firm) by undertaking the project. It directly answers how much value a project will create.

4. Consistent with Shareholder Wealth Maximization: The primary objective of a firm is to maximize shareholder wealth. By accepting projects with positive NPVs, the firm aligns its investment decisions directly with this objective.

5. Absolute Measure: NPV provides an absolute monetary value of a project's profitability, which can be easily understood and compared (for mutually exclusive projects).

6. Reinvestment Assumption: NPV implicitly assumes that intermediate cash flows are reinvested at the discount rate (cost of capital), which is generally considered a more realistic assumption than assuming reinvestment at the project's internal rate of return (IRR).


Disadvantages of NPV:

1. Requires Estimation of Discount Rate: Determining the appropriate discount rate (cost of capital) can be challenging and subjective. An incorrect discount rate can lead to flawed NPV results and incorrect decisions.

2. Requires Accurate Cash Flow Forecasts: NPV heavily relies on accurate forecasts of future cash flows, which are often difficult to predict with certainty, especially for long-term projects. Errors in forecasting can significantly impact the NPV.

3. Does Not Indicate Relative Profitability/Efficiency: NPV is an absolute measure (in currency units), not a relative one. It might not be suitable for comparing projects of different sizes or initial investments. A small project with a small positive NPV might be more capital-efficient than a large project with a large positive NPV. (This limitation is sometimes addressed by using the Profitability Index, which is a ratio).

4. Complexity: For non-finance professionals, the concept of discounting and present value might be less intuitive than simpler methods like payback period or accounting rate of return.

5. Difficulty with Mutually Exclusive Projects (of different lives): While useful for mutually exclusive projects, comparing projects with significantly different lifespans using NPV can sometimes be misleading, although methods like Equivalent Annual Annuity (EAA) can help address this.

Question 27. A car is bought for $\textsf{₹} 10,00,000$. Its value depreciates at $15\%$ per annum on a reducing balance basis (compound depreciation). Find the value of the car at the end of 3 years and the total depreciation over these 3 years.

Answer:

Problem Statement:

Calculate the value of a car at the end of 3 years and the total depreciation over these 3 years, given its initial cost of $\textsf{₹} 10,00,000$ and a depreciation rate of $15\%$ per annum on a reducing balance basis.


Given:

Initial Cost (P) $= \textsf{₹} 10,00,000$

Depreciation Rate (r) $= 15\%$ per annum $= 0.15$

Time (n) $= 3$ years

Depreciation Method: Reducing Balance Basis (Compound Depreciation)


To Find:

1. The Value of the Car at the end of 3 years (A).

2. The Total Depreciation over these 3 years.


Formulae Used:

1. Value after $n$ years (A) with Reducing Balance Depreciation:

$\text{A} = P(1 - r)^n$

... (1)

2. Total Depreciation:

$\text{Total Depreciation} = \text{Initial Cost} - \text{Value after n years}$

... (2)

Where:

  • $P$ = Initial Cost
  • $r$ = Depreciation Rate (as a decimal)
  • $n$ = Number of years
  • $A$ = Value after $n$ years

Solution:

Step 1: Calculate the Value of the Car at the end of 3 years (A).

Using formula (1):

$\text{A} = 10,00,000 (1 - 0.15)^3$

$\text{A} = 10,00,000 (0.85)^3$

Calculate $(0.85)^3$:

$(0.85)^2 = 0.85 \times 0.85 = 0.7225$

$(0.85)^3 = 0.7225 \times 0.85 = 0.614125$

Now, substitute this value back into the amount calculation:

$\text{A} = 10,00,000 \times 0.614125$

$\text{A} = \textsf{₹} 6,14,125$

... (i)

The value of the car at the end of 3 years is $\textsf{₹} 6,14,125$.

Step 2: Calculate the Total Depreciation over these 3 years.

Using formula (2):

$\text{Total Depreciation} = \text{Initial Cost} - \text{Value after 3 years}$

$\text{Total Depreciation} = \textsf{₹} 10,00,000 - \textsf{₹} 6,14,125$

$\text{Total Depreciation} = \textsf{₹} 3,85,875$


Conclusion:

The value of the car at the end of 3 years is $\textsf{₹} 6,14,125$.

The total depreciation over these 3 years is $\textsf{₹} 3,85,875$.